Posts

Daily Prelims CA Quiz

UPSC Quiz – 2024 : IASbaba’s Daily Current Affairs Quiz 30th April 2024

The Current Affairs questions are based on sources like ‘The Hindu’, ‘Indian Express’ and ‘PIB’, which are very important sources for UPSC Prelims Exam. The questions are focused on both the concepts and facts. The topics covered here are generally different from what is being covered under ‘Daily Current Affairs/Daily News Analysis (DNA) and Daily Static Quiz’ to avoid duplication. The questions would be published from Monday to Saturday before 2 PM. One should not spend more than 10 minutes on this initiative. Gear up and Make the Best Use of this initiative. Do remember that, “the difference between Ordinary and EXTRA-Ordinary is PRACTICE!!” Important Note: Don’t forget to post your marks in the comment section. Also, let us know if you enjoyed today’s test 🙂After completing the 5 questions, click on ‘View Questions’ to check your score, time taken, and solutions.To take the Test Click Here

DAILY CURRENT AFFAIRS IAS | UPSC Prelims and Mains Exam – 29th April 2024

Archives (PRELIMS & MAINS Focus)   Candidate elected unopposed Syllabus Prelims – Polity Context: On April 22, the ruling Bharatiya Janata Party opened its tally in the Lok Sabha when its nominee for the Surat constituency in the western State of Gujarat, Mukesh Dalal, was elected unopposed. Background:- This happened because of the rejection of the nomination papers of the Congress candidates [main and substitute nominees] the previous day and the withdrawal of other nominees. How is a candidate declared elected before polling? Section 53 (3) of the Representation of the People Act, 1951 deals with the procedure in uncontested elections. According to this proviso, if the number of such candidates is less than the number of seats to be filled, the returning officer (RO) shall forthwith declare all such candidates to be elected. The RO’s actions are governed by Section 33 of the Act which pertains to the presentation of nomination papers and requirements for a valid nomination.The sub-section 4 says: “On the presentation of a nomination paper, the returning officer shall satisfy himself that the names and electoral roll numbers of the candidate and his proposer as entered in the nomination paper are the same as those entered in the electoral rolls”. In the given instance, three proposers of the Congress’ candidate for Surat, Nilesh Kumbhani, claimed in an affidavit to the district election officer (DEO) that they had not signed his nomination form. They did not also turn up before the DEO on April 21 to support the candidate’s nomination form. What is the scope for negative voting in the election system? There is ample scope in the system. While the NOTA (none of the above) option has been in force since 2013, the Conduct of Election Rules, 1961, allows electors to decide not to vote through Rule 49-O. A remark to the effect that the elector has decided not to record his or her vote would have to be made in the “remarks column” against the entry relating to the elector in the Register of Voters by the presiding officer, after which the signature or thumb impression of the elector would have to be obtained against such a remark. The option of NOTA, introduced through the Supreme Court’s intervention, has been available on electronic voting machines (EVMs) since November 2013. This became a reality in the wake of a verdict given by a Bench of the Supreme Court comprising Chief Justice P. Sathasivam and Justices Ranjana Desai and Ranjan Gogoi in September 2013, while allowing a petition filed by the People’s Union for Civil Liberties. The then CJI, who wrote the judgment, said: “Giving right to a voter not to vote for any candidate while protecting his right of secrecy is extremely important in a democracy. Such an option gives the voter the right to express his disapproval of the kind of candidates being put up by the parties.” He expressed the hope that “gradually, there will be a systemic change and the parties will be forced to accept the will of the people and field candidates who are known for their integrity.” There is a difference between an elector exercising Rule 49-O and one using the NOTA option. In the case of the former, the likelihood of such an elector compromising his or her secrecy is high, as there is a procedure to be followed manually at a polling booth. However, in the case of the latter, there is no such issue. Source: Hindu Success of the INS Vikrant Syllabus Prelims & Mains – Current Event Context: On March 5, both aircraft carriers of the Indian Navy, INS Vikramaditya and INS Vikrant, showcased “twin carrier operations” with MiG-29K fighter jets taking off simultaneously from both and landing cross deck as Defence Minister Rajnath Singh looked on from onboard one of them. Background: An aircraft carrier is fundamental to command, control and coordination of operations from the sea and to project combat power ashore, over the seas or in the air. What does INS Vikrant signify? : The design work on the Indigenous Aircraft Carrier (IAC)-I, later christened Vikrant, began in 1999; however 2005-2006 were probably the most crucial years for the carrier and for India’s war shipbuilding. The crucial decision was on the warship grade steel, which till then was procured from Russia. After much brainstorming, it was decided that it would be developed and produced in India, a collaborative effort between the Steel Authority of India, the Defence Research Development Organisation (DRDO) and the Indian Navy.DMR grade steel was developed. Vikrant is an engineering marvel. It has a total area in excess of 12,450 m2 which equals to about two and a half hockey fields. The 262m long and 62m wide ship is powered by four General Electric LM2500 engines generating 88 MW of power giving it a maximum speed of 28 Knots and an endurance of 7,500 nautical miles. Built at an overall cost of around ₹20,000 crore and 76% indigenous content, the ship has around 2,200 compartments, for a crew of around 1,600 that include specialised cabins to accommodate women officers and sailors. Vikrant can operate an air wing of 30 aircraft comprising MiG-29K fighter jets, Kamov-31, MH-60R multi-role helicopters, in addition to indigenous Advanced Light Helicopters and Light Combat Aircraft (Navy). It uses the STOBAR (Short Take-Off but Arrested Recovery) method to launch and recover aircraft for which it is equipped with a ski- jump to launch aircraft, and three ‘arrester wires’ for their recovery. Noting that among manufacturing activities, shipbuilding has one of the highest employment multipliers of 6.48, the economic Survey 2022-23 said that Vikrant alone engaged approximately 500 MSMEs, 12,000 employees from ancillary industries, and 2,000 CSL employees. While the present Vikrant was the first carrier built in the country, India has had a long history of operating carriers. The erstwhile 19,500 tonne Vikrant was India’s first carrier purchased from the U.K., which arrived in 1961 and played a vital role in the 1971 war. Then came the 28,700 tonne INS Viraat commissioned in 1987, formerly HMS Hermes, also from the U.K. INS Vikramaditya procured from Russia and commissioned in 2013 is India’s third carrier. Source: Hindu NATIONAL DISASTER RESPONSE FUND (NDRF) Syllabus Prelims – Current Event Context: The Union government releases disaster relief funds for Karnataka and Tamil Nadu from the National Disaster Response Fund (NDRF). Background: Move comes weeks after the two State governments approached the Supreme Court seeking relief funds for calamities such as Cyclone Michaung, floods and drought in 2023. About NATIONAL DISASTER RESPONSE FUND (NDRF) The National Disaster Response Fund (NDRF) is a crucial component of disaster management in India. It is a fund managed by the Central Government to meet expenses related to emergency response, relief, and rehabilitation during threatening disaster situations or actual disasters. The fund aims to provide immediate assistance when natural or man-made calamities strike. Decisions on NDRF expenses are made by the National Executive Committee (NEC) of the National Disaster Management Authority. The NDRF accounts are audited annually by the Comptroller and Auditor General (CAG). Supplementing State Disaster Response Funds (SDRF): The NDRF operates alongside the State Disaster Response Funds (SDRF). It provides additional financial assistance for natural calamities of severe nature. When a state faces a disaster that exceeds the available funds in its own SDRF, the NDRF steps in to offer support. Eligibility and Coverage – NDRF guidelines cover a range of disasters: Natural Calamities: Cyclones, drought, earthquakes, fires, floods, tsunamis, hailstorms, landslides, avalanches, cloud bursts, pest attacks, and cold waves. Man-Made Disasters: These include terrorist attacks, chemical or biological incidents, and nuclear disasters as notified by the Central Government. States seeking NDRF funds must submit a memorandum detailing the damage and funding requirements. The NDRF provides immediate relief assistance, not compensation for property or crop loss. Sources of Financing: The NDRF is financed through a cess on certain items, chargeable to excise and customs duty, approved annually through the Finance Bill. If additional funds are needed beyond what the NDRF provides, they come from general budgetary resources. Source: The Wire WTO INFORMATION TECHNOLOGY PRODUCTS AGREEMENT (ITA) Syllabus Prelims – Current Event Context: India and Chinese Taipei have jointly requested the World Trade Organization (WTO) not to adopt any ruling regarding New Delhi’s import duties on specific information and technology (ICT) products until July 26. Background: ICT products fall under the WTO’s Information Technology Products Agreement (ITA). India’s involvement with the ITA has led to disputes due to the imposition of customs duties on certain ICT products. INFORMATION TECHNOLOGY PRODUCTS AGREEMENT (ITA) : The Information Technology Agreement (ITA) is a significant trade agreement under the World Trade Organization (WTO) that was adopted in 1996. The ITA aims to eliminate all import duties and other charges on specified information technology products. The commitments under the ITA become binding on a country under Articles II of the General Agreement on Tariffs and Trade (GATT) only if they are incorporated into the Goods Schedule. Key aspects of the ITA include: Trade Liberalization: The ITA promotes trade liberalization by eliminating duties on IT products among participating WTO member countries. Product Coverage: The agreement covers a wide range of IT products, including computers, telecommunication equipment, semiconductors, software, and scientific instruments. Expansion: The ITA has been expanded since its inception to include more products and participants, reflecting the evolving nature of technology and global trade. Source: Business Standard National Research Foundation Syllabus Prelims & Mains – Education Context: The provisions of the Anusandhan National Research Foundation (ANRF) Act have been brought into force on February 5, 2024 in line with the commitment of the Government to boost Research and Innovation as the fulcrum of growth and development of the country. Background: The Anusandhan National Research Foundation Act of 2023 provides for the establishment of the Anusandhan National Research Foundation (NRF). National Research Foundation (NRF): It is an apex body to provide high-level strategic direction of scientific research in the country in accordance with the recommendations of the National Education Policy (NEP) and will be established at a cost of 50,000 crore over the period 2023-28. It aims to increase private sector contributions to research in India and to ensure that a larger portion of government funds go to state universities and colleges. The Department of Science and Technology (DST) will be NRF’s administrative department. Functions of the National Research Foundation: Preparing the roadmap for short-, medium-, and long-term research and development. Facilitating and funding the expansion of Research and Development and related infrastructure in universities, colleges, and research institutions. Funding competitive peer-reviewed grant proposals to eligible individuals. Assisting in the transformation of research into capital-intensive technology. Enhancing India’s role and engagement in significant national and global areas. Encouraging private and public sector entities to invest in the foundation. Carry out annual surveys of scientific research, outcomes, and spending. Need for National Research Foundation: India’s Gross Expenditure on Research and Development (GERD) stagnated at about 0.7 percent of GDP, which is less as compared to the 2% of the global benchmark. A significant infusion of funding, steered by a clear implementation strategy, is required for India to become a science leader. 52% of GERD in India is by the government which is substantially higher than in the US (10%), Germany (13%), and China (15%). To bridge this gap, the ANRF-2023 Act plans to seek significant private-sector contributions. NRF can be a viable tool to address challenges to India’s research and development capacity. For example, a share of the funds should be targeted toward building scientific research capacity in universities and semi-urban areas. Lower spending on R&D and less innovative opportunities may force young talents to move to better terrains. India must strive to optimally use its pool of talent. Significance of the National Research Foundation: NRF will prioritize funding for projects in outlying, rural, and semi-urban areas, which are underserved and rarely receive funding for science projects. The NRF would support research in areas other than natural sciences and engineering, such as social sciences, arts, and humanities. For the implementation of missions such as the supercomputer mission or the quantum mission. The NRF will establish collaborations between business, academia, government agencies, and research institutions, and will dig into the financial resources of the private sector to ensure a financially viable research ecosystem. Limitations of the National Research Foundation: An amount of Rs 50,000 crore has been earmarked for priority areas in research over 2023-2028. It is stated that approximately 70% will be raised from the private sector. There is no mechanism in the NRF Act to ensure this support. It does not follow the principle of cooperative federalism and has no place for the representatives of state governments in the decision-making structure. The act does not enunciate the role and contribution of the relevant actors operating under the economic and social ministries in the NRF decision-making structures. The CPSEs should play a pivotal role in being vehicles for the transfer of R&D benefits from academic institutions to application to strategic areas of manufacturing. Source: Times of India CLIMATE TECHNOLOGY CENTRE AND NETWORK (CTCN) Syllabus Prelims – Current Event Context: The Climate Technology Centre and Network (CTCN) celebrates its 10-year anniversary as a key player in advancing climate technology and innovation. Background: The CTCN’s commitment to climate resilience and emissions reduction remains unwavering as it continues to drive technological advancements for a sustainable future. About CLIMATE TECHNOLOGY CENTRE AND NETWORK (CTCN) : The Climate Technology Centre and Network (CTCN) is a vital initiative that connects countries to climate solutions. The Climate Technology Centre and Network (CTCN) is the operational arm of the UNFCCC Technology Mechanism, hosted by the UN Environment Programme and the UN Industrial Development Organization (UNIDO). It was established under the United Nations Framework Convention on Climate Change (UNFCCC) to facilitate the transfer of environmentally sound technologies to developing countries for low carbon and climate-resilient development. The primary goal is to facilitate the transfer of environmentally sound technologies to developing countries. Key Functions: Technology Transfer: The CTCN accelerates the adoption of climate-friendly technologies by responding to country-driven requests. Capacity Building: It provides expertise, policy advice, and capacity-building support tailored to individual countries. Global Network: The CTCN collaborates with a global network of technology companies and institutions. Source: UNEP Practice MCQs Daily Practice MCQs Q1.) With reference to the Climate Technology Centre and Network (CTCN), consider the following statements: The CTCN serves as the implementation arm of the Technology Mechanism under the UNFCCC. The primary goal is to facilitate the transfer of environmentally sound technologies to developing countries. Which of the statements given above is/are correct? 1 only 2 only Both 1 and 2 Neither 1 nor 2 Q2.) The Information Technology Agreement (ITA), recently seen in news is related to IMF WTO ASEAN EU Q3.) With reference to the National Disaster Response Fund (NDRF), consider the following statements: It is a fund managed by the Central Government to meet expenses related to emergency response, relief, and rehabilitation during threatening disaster situations or actual disasters. Decisions on NDRF expenses are made by the National Executive Committee of the National Disaster Management Authority. The NDRF accounts are audited annually by the Comptroller and Auditor General. How many of the above statements are correct? Only one Only two All three None Comment the answers to the above questions in the comment section below!! ANSWERS FOR ’  29th April  2024 – Daily Practice MCQs’ will be updated along with tomorrow’s Daily Current Affairs.st ANSWERS FOR  27th April – Daily Practice MCQs Answers- Daily Practice MCQs Q.1) – b Q.2) – d Q.3) – c

Daily Prelims CA Quiz

UPSC Quiz – 2024 : IASbaba’s Daily Current Affairs Quiz 29th April 2024

The Current Affairs questions are based on sources like ‘The Hindu’, ‘Indian Express’ and ‘PIB’, which are very important sources for UPSC Prelims Exam. The questions are focused on both the concepts and facts. The topics covered here are generally different from what is being covered under ‘Daily Current Affairs/Daily News Analysis (DNA) and Daily Static Quiz’ to avoid duplication. The questions would be published from Monday to Saturday before 2 PM. One should not spend more than 10 minutes on this initiative. Gear up and Make the Best Use of this initiative. Do remember that, “the difference between Ordinary and EXTRA-Ordinary is PRACTICE!!” Important Note: Don’t forget to post your marks in the comment section. Also, let us know if you enjoyed today’s test 🙂After completing the 5 questions, click on ‘View Questions’ to check your score, time taken, and solutions.To take the Test Click Here

DAILY CURRENT AFFAIRS IAS | UPSC Prelims and Mains Exam – 26th April 2024

Archives (PRELIMS & MAINS Focus)   Effect of rising temperature on health Syllabus Prelims & Mains – Health & Climate Change Context: A study of 266 studies worldwide,  revealed that heat waves were associated with a 11.7 per cent increase in mortality. The highest specific risk was for stroke and coronary heart disease. Background:- Even as we must try energetically to mitigate the drivers of climate change and global warming, we must prepare adaptation plans. Heat Waves and Health The consequences of climate change, mostly mediated through global warming, adversely affect human, animal and plant health in several ways: Direct effects of heat exposure on the body; extreme weather events; water scarcity; vector-borne and water-borne infections; non-communicable diseases (strokes; heart attacks; diabetes; respiratory disease; cancers); mental health disorders; food and nutrition insecurity due to reduced yield and nutrient quality of crops. Health systems will face increased demand due to these and the movement of displaced climate refugees. They will also be affected by the adverse impact of extreme weather events on healthcare infrastructure, supply chains and personnel. Of these myriad threats, heat waves are of immediate concern as summer advances across the Indian Subcontinent.A heat wave represents a combination of heat and humidity. At any level of heat, its impact on human body is amplified if humidity is high in the ambient air. The India Meteorological Department (IMD) says that such “moist heat” stress has increased by 30 per cent between 1980 and 2020. Heat acts through dehydration, inability of the skin to cool the body through perspiration, dilatation of blood vessels and thickening of blood with increased risk of clot formation (thrombosis). Infants and young children are more vulnerable than adults, since their bodies have higher fluid content than grownups. Among adults, vulnerability is high among the elderly, persons with disabilities and in those with serious comorbidities. Women are more vulnerable than men. Non-communicable diseases (NCDs) contribute to 65 per cent of deaths in India. The heat effects of climate change will only exacerbate the problem. As our population ages and cardiovascular risk factors rise in our population, every 1 degree centigrade rise in ambient temperature will compound the risk of serious cardiovascular events. Wildfires triggered by excessive heat release particulate matter (PM 2.5) and toxic chemicals (ozone, carbon monoxide, polycyclic aromatic compounds and nitrogen dioxide) can cause extensive inflammation, increasing the risk of cardiovascular disease (heart attacks, strokes, heart rhythm abnormalities, pulmonary embolism, heart failure), respiratory disease, diabetes and pre-diabetes. Chemicals like benzene and formaldehyde (also present in wildfire emissions) increase the risk of cancer. Less immediate, but with a long lasting impact on health, is the impact on food systems which threatens nutrition security. Countries in South Asia and sub-Saharan Africa currently grow staples (like rice and wheat) at the highest levels of heat tolerance. A further increase of 1 degree centigrade will lower their yield by 10 per cent. The crops will also be more deficient in zinc, protein and iron. Rising ocean temperatures will flood coastal agriculture while reducing fish yields. Loss of biodiversity will reduce access to nutritious “wild foods”. Heat will kill us not only through its effect on our skin but also our stomachs. There is a need to prepare for adaptation plans. These will include heat action climate-smart and climate resilient food and healthcare systems, and education of public and healthcare providers. Source: Indian Express Rise of a ‘new’ Japan Syllabus Mains – GS 2 Context: Japan’s Prime Minister Fumio Kishida’s address to the United States Congress earlier this month, and the developments from his summit meeting with President Joe Biden, announced the arrival of a new, assertive Japan to the world. Background: Japan has given up its post-War pacifism and is working to become a major military power. The turn is historic, and promises to transform the geopolitics of Asia and the world. A politically resolute and militarily strong Japan contributes to India’s objective of building a multipolar Asia in a multipolar world. Key Takeaways: After World War II, a defeated and chastened Japan adopted a policy of pacifism — it avoided building significant armed capability, limited its defence expenditure, and refused to participate in military conflicts anywhere.This pacifism was rooted in Japan’s determination to expiate its militarism and imperialism in the pre-War decades. Japan’s pacifism was compensated by its bilateral military alliance with the United States. Being a protectorate of the US in Asia, Japan was free to focus its energies on rebuilding its economy. Today, Japan is set to become a major military power, transform its famed civilian industrial capability into a military industrial complex, and turn from being a US protectorate into an American partner and a significant contributor to Asian and Indo-Pacific security. This is the second Japanese transition over the last decade. In the mid-2000s, Japan began to articulate ideas for a new security architecture in Asia. Take for instance the concept of the Indo-Pacific. The late Prime Minister Shinzo Abe first spoke about the strategic unity of the two oceans (Indian and Pacific) during his visit to India in 2007. Since then, Japan has invested enormous diplomatic, political, and financial resources to popularise this geopolitical construct that was eventually adopted by Australia, Indonesia, India, and the United States. In the second transition unfolding today, Japan is matching its strategic ideas with military resources and the political will to actively reshape the regional security order. Reasons behind the radical change in Japan’s reorientation in defence: A mix of external and internal factors have contributed to its Japan’s reorientation in the 21st century. On the external front, four elements have come together: the rise of China and its military assertion, especially on the territorial disputes with Japan; the deepening military bonds between Beijing and Moscow and the coordination of their policies in North East Asia; the growing military capabilities of North Korea; and the fears triggered by the Trump Administration that the US could withdraw its security protection to Japan and other Asian allies. Internally, this situation strengthened conservatives in Tokyo, who wanted to see Japan becoming a ‘normal power’. They argued that Japan has done enough to prove its credentials as a responsible citizen of world affairs, and the time has come for it to take responsibility for its own security, and to contribute to the regional order. What Japan is doing to become a geopolitical power? First, Japan has done away with the historical cap on defence expenditure. According to data from the World Bank, in 2020, Japan’s military expenditure touched 1% of GDP for the first time in six decades. In 2022, it touched 1.1%. Kishida’s government has pledged to double annual defence spending to around 10 trillion yen ($68 billion) by 2027. Second, Japan has acquired, and is in the process of further acquiring, its own counter-strike capability in the form of cruise missiles. Third, late last year, Japan’s cabinet eased its self-imposed ban on exports of lethal weapons to friendly countries. This has paved the way for the leveraging of Japan’s immense manufacturing and technological base to aid its allies at a time when their own production capacities cannot keep up with the likes of China and Russia. Fourth, Japan and the US are in the process of revising the command-and-control framework governing their defence forces, with plans to eventually significantly integrate the command structure. Currently, the US operates military bases in mainland Japan and Okinawa island, but its command structure is in Hawai’i, the headquarters of the US Indo-Pacific Command. A joint operations command will be set up in Japan going forward. Source: Indian Express SPICES BOARD OF INDIA Syllabus Prelims – Current Event Context: In response to the recent ban on certain Indian spice brands in Singapore and Hong Kong due to quality concerns, the Spices Board of India has taken proactive measures. Background: The Spices Board will now conduct mandatory testing for ethylene oxide in spice consignments destined for Singapore and Hong Kong. This decision comes after the ban on products from Indian brands MDH and Everest in these countries. About SPICES BOARD OF INDIA The Spices Board of India is a statutory body established under the Spices Board Act of 1986. It resulted from the merger of two entities: the erstwhile Cardamom Board and the Spices Export Promotion Council. The Spices Board operates under the Ministry of Commerce & Industry, Government of India. Its primary purpose is to facilitate spice trade, promote exports, and safeguard the interests of Indian spice growers and exporters. The Board serves as an international link between Indian spice exporters and importers abroad. Spices Production and Export: India is the largest producer, consumer, and exporter of spices globally. India produces about 75 out of 109 varieties listed by the International Organization for Standardization (ISO). The country’s diverse climatic conditions allow for the cultivation of a variety of spices, including: Black Pepper: Often referred to as the “King of Spices,” native to the Western Ghats of India. Cardamom: Known as the “Queen of Spices,” produced in Kerala, Tamil Nadu, and Karnataka. Turmeric: Key ingredient in Indian cooking, known for its vibrant colour and anti-inflammatory properties. Major spices-producing states in India include Madhya Pradesh, Rajasthan, Gujarat, Andhra Pradesh, Telangana, Karnataka, Maharashtra, Assam, and others. Source: Times of India Municipal Elections Syllabus Prelims & Mains – Polity Context: The Supreme Court judgment on the Chandigarh mayoral election resurfaces the issues associated with the electoral processes of local urban bodies. Background: Bench of DY Chandrachud, CJI, JB Pardiwala and Manoj Misra, JJ. set aside the Mayor election results and declared the appellant Kuldeep Kumar to be the winner for Mayor post since the 8 wrongly invalidated votes were cast in his favour, giving him the majority over BJP candidate. Status of Municipal Elections in India: According to the Annual Survey of India’s City-Systems 2023 by Janaagraha (a non-profit institution), over 1,400 municipalities in India did not have elected councils in place as of September 2021 which indicates a significant and widespread issue across the country. The Comptroller and Auditor-General of India (CAG)’s audit revealed that, between 2015 and 2021, over 1,500 municipalities didn’t have elected councils. Even major cities like Chennai, Delhi, Mumbai, and Bengaluru faced delays ranging from months to years in holding elections. Issues/Challenges associated with Local Bodies’ Elections: Due to the ambiguous constitutional safeguards, government officials like SECs currently possess discretionary powers when it comes to scheduling elections. This flexibility can sometimes lead to inconsistent or delayed election timelines, which may undermine the democratic process’s transparency and fairness. There is a concern about the potential undue influence exerted by state governments to delay elections for political or strategic reasons. Such interference can compromise the integrity of the electoral process and erode public trust in democratic institutions. The continued reliance on manual ballot paper-based processes introduces vulnerabilities, such as errors in counting, the potential for tampering, and delays in declaring election results. This traditional approach may not be as efficient or secure as modern Electronic Voting Machine (EVM) and Voter Verifiable Paper Audit Trail (VVPAT), which could enhance the transparency and credibility of electoral outcomes. Even after elections, municipal councils in urban local governments were not promptly constituted. For example: In Karnataka, a delay of 12-24 months was observed post-elections. Way Forward: To deal with the challenges effectively, the State Election Commissions (SECs) need to play a more significant role in overseeing the electoral process using the powers mentioned in Articles 243K and 243ZA of the Constitution. Only 11 out of 35 states and union territories have empowered SECs to conduct ward delimitation. Ward delimitation is crucial for ensuring fair and equitable representation in municipal elections. SECs should be granted greater authority, including the power to conduct ward delimitation. Holding election officials and authorities accountable for any delays or irregularities in the conduct of municipal elections. This can be done through transparent investigation processes and appropriate disciplinary action. Addressing the challenges highlighted, from scheduling elections to ensuring fair processes, requires comprehensive policy reforms. The idea of ‘One Nation, One Election’ can be explored with a major focus on the efficient and timely election of the local bodies. Source: SCC Online MOUNT EREBUS Syllabus Prelims – Geography Context: According to experts, Mount Erebus, one of the most active volcanoes on Earth, emits approximately 80 grams of gold every day, valued at about £5,000 (or $6,000, approximately). Background: Mt Erebus gets its name from Captain Sir James Clark Ross who first discovered it in 1841.   About MOUNT EREBUS : Mount Erebus is located on Ross Island in Antarctica. Mount Erebus is the second-highest volcano in Antarctica, following Mount Sidley. Mount Erebus holds the distinction of being the world’s southernmost active volcano. It stands at 3,794 meters (12,448 feet) above sea level. Among all the islands on Earth, it ranks as the sixth-highest point. After Mount Vinson, it is the second most prominent mountain in Antarctica. Geological Significance: Mount Erebus is classified as a polygenetic stratovolcano. Its lower half is a shield, while the upper half forms a stratocone. The volcano has been active for around 1.3 million years. Within its inner summit crater, there exists a persistent convecting phonolitic lava lake—one of only five such long-lasting lava lakes on Earth. Source: Times of India EXERCISE POORVI LEHAR (XPOL) Syllabus Prelims – Current Event Context: Recently, the Indian Navy conducted extensive exercise Poorvi Lehar. Background: Exercise Poorvi Lehar plays a crucial role in ensuring the Indian Navy’s capability to safeguard our coastal waters and maintain security in the East Coast region. About EXERCISE POORVI LEHAR (XPOL): Exercise Poorvi Lehar (XPOL) is a significant maritime exercise conducted by the Indian Navy along the East Coast. Under the operational direction of the Flag Officer Commanding-in-Chief, Eastern Naval Command, this exercise aims to validate procedures and assess the Indian Navy’s preparedness to meet Maritime Security challenges in the region. Key Points : Objective: The exercise focuses on enhancing the Indian Navy’s readiness to address security threats in the maritime domain. Participating Assets: It involves Ships, Submarines, Aircrafts, and Special Forces. Phases: XPOL is conducted in multiple phases, including combat training during the Tactical Phase and successful firings during the Weapon Phase. Interoperability: The exercise demonstrates a high degree of interoperability among different services, with participation from assets of the Indian Air Force (IAF), Andaman & Nicobar Command, and Coast Guard. Maritime Domain Awareness: Continuous awareness of the maritime area of operations is maintained through the operation of aircraft from diverse locations. Source: PIB Practice MCQs Daily Practice MCQs Q1.) With reference to Exercise Poorvi Lehar (XPOL), consider the following statements: It is a maritime exercise conducted by the Indian Navy along the East Coast. The exercise focuses on enhancing the Indian Navy’s readiness to address security threats in the maritime domain. How many of the statements given above are correct? 1 only 2 only Both 1 and 2 Neither 1 nor 2 Q2.) With reference to the Spices Board of India, consider the following statements: The Spices Board of India is a statutory body. The Spices Board operates under the Ministry of Commerce & Industry. The Board serves as an international link between Indian spice exporters and importers abroad. How many of the statements given above are correct? Only one Only two All three None Q3.) Mount Erebus, an active volcano is located in Indonesia Italy Australia Antarctica  

[DAY 49] 60 DAY RAPID REVISION (RaRe) SERIES for UPSC Prelims 2024 – POLITY, CURRENT AFFAIRS & CSAT TEST SERIES!

Archives Hello Friends The 60 Days Rapid Revision (RaRe) Series is IASbaba’s Flagship Initiative recommended by Toppers and loved by the aspirants’ community every year. It is the most comprehensive program which will help you complete the syllabus, revise and practice tests on a daily basis. The Programme on a daily basis includes Daily Prelims MCQs from Static (Monday – Saturday) Daily Static Quiz will cover all the topics of static subjects – Polity, History, Geography, Economics, Environment and Science and technology. 20 questions will be posted daily and these questions are framed from the topics mentioned in the schedule. It will ensure timely and streamlined revision of your static subjects. Daily Current Affairs MCQs (Monday – Saturday) Daily 5 Current Affairs questions, based on sources like ‘The Hindu’, ‘Indian Express’ and ‘PIB’, would be published from Monday to Saturday according to the schedule. Daily CSAT Quiz (Monday – Friday) CSAT has been an Achilles heel for many aspirants. Daily 5 CSAT Questions will be published. Note – Daily Test of 20 static questions, 10 current affairs, and 5 CSAT questions. (35 Prelims Questions) in QUIZ FORMAT will be updated on a daily basis. To Know More about 60 Days Rapid Revision (RaRe) Series – CLICK HERE   60 Day Rapid Revision (RaRe) Series Schedule – CLICK HERE  Important Note Comment your Scores in the Comment Section. This will keep you accountable, responsible and sincere in days to come. It will help us come out with the Cut-Off on a Daily Basis. Let us know if you enjoyed today’s test 🙂  You can post your comments in the given format  (1) Your Score (2) Matrix Meter (3) New Learning from the Test Time limit: 0 Test-summary 0 of 35 questions completed Questions: 1 2 3 4 5 6 7 8 9 10 11 12 13 14 15 16 17 18 19 20 21 22 23 24 25 26 27 28 29 30 31 32 33 34 35 Information The following Test is based on the syllabus of 60 Days Plan-2023 for UPSC IAS Prelims 2022. To view Solutions, follow these instructions: Click on – ‘Start Test’ button Solve Questions Click on ‘Test Summary’ button Click on ‘Finish Test’ button Now click on ‘View Questions’ button – here you will see solutions and links. You have already completed the test before. Hence you can not start it again. Test is loading... You must sign in or sign up to start the test. You have to finish following test, to start this test: Results 0 of 35 questions answered correctly Your time: Time has elapsed You have scored 0 points out of 0 points, (0) Average score     Your score     Categories Not categorized 0% Your result has been entered into leaderboard Loading Name: E-Mail: Captcha: maximum of 70 points Pos. Name Entered on Points Result Table is loading No data available 1 2 3 4 5 6 7 8 9 10 11 12 13 14 15 16 17 18 19 20 21 22 23 24 25 26 27 28 29 30 31 32 33 34 35 Answered Review Question 1 of 35 1. Question Which of the following is/are among the items provided under 12th Schedule for devolution of powers to Municipalities? Regulation of slaughter houses Fire services Prevention of cruelty to animals Safeguarding the interests of the weaker sections Select the correct answer using the code given below: a) 1 only b) 3 only c) 2 and 4 only d) All of the above Correct Solution (d) Items provided under 12th Schedule for devolution of powers to Municipalities: Urban planning including town Regulation of land-use and construction of Planning for economic and social Roads and Water supply for domestic, industrial and commercial Public health, sanitation conservancy and solid waste Fire Urban forestry, protection of the environment and promotion of ecological aspects Safeguarding the interests of weaker sections of society, including the handicapped and mentally retarded. Slum improvement and Urban poverty Provision of urban amenities and facilities such as parks, gardens, Promotion of cultural, educational and aesthetic Burials and burial grounds; cremations, cremation grounds; and electric Cattle pounds; prevention of cruelty to Vital statistics including registration of births and Public amenities including street lighting, parking lots, bus stops and public Regulation of slaughter houses and Incorrect Solution (d) Items provided under 12th Schedule for devolution of powers to Municipalities: Urban planning including town Regulation of land-use and construction of Planning for economic and social Roads and Water supply for domestic, industrial and commercial Public health, sanitation conservancy and solid waste Fire Urban forestry, protection of the environment and promotion of ecological aspects Safeguarding the interests of weaker sections of society, including the handicapped and mentally retarded. Slum improvement and Urban poverty Provision of urban amenities and facilities such as parks, gardens, Promotion of cultural, educational and aesthetic Burials and burial grounds; cremations, cremation grounds; and electric Cattle pounds; prevention of cruelty to Vital statistics including registration of births and Public amenities including street lighting, parking lots, bus stops and public Regulation of slaughter houses and Question 2 of 35 2. Question Which of the following is/are the compulsory provisions of Part IX of the Constitution of India? Direct elections to all seats in panchayats at the village, intermediate and district 25 years to be the minimum age for contesting elections to panchayats. Granting financial powers to the How many of the above statements are correct? a) Only one b) Only two c) All three d) None Correct Solution (a) Statement 1 Statement 2 Statement 3 Correct Incorrect Incorrect Direct elections to all seats in panchayats at the village, intermediate and district levels. It is a compulsory provision under 73rd Constitutional amendment Act 1992. 21 years to be the minimum age for contesting elections to panchayats. Granting financial powers to the panchayats, that is, authorizing them to levy, collect and appropriate taxes, duties, tolls and fees is under voluntary provision of 73rd Constitutional amendment act 1992. Incorrect Solution (a) Statement 1 Statement 2 Statement 3 Correct Incorrect Incorrect Direct elections to all seats in panchayats at the village, intermediate and district levels. It is a compulsory provision under 73rd Constitutional amendment Act 1992. 21 years to be the minimum age for contesting elections to panchayats. Granting financial powers to the panchayats, that is, authorizing them to levy, collect and appropriate taxes, duties, tolls and fees is under voluntary provision of 73rd Constitutional amendment act 1992. Question 3 of 35 3. Question Consider the following statements with regards to the Panchayats? Elections to constitute panchayat shall be completed before the expiry of its duration of 5 In case of premature dissolution new panchayat remains in the office only for the remaining It shall not be necessary to hold elections if the remaining period for dissolved panchayat is less than 6 How many of the above statements are correct? a) Only one b) Only two c) All three d) None Correct Solution (c) Statement 1 Statement 2 Statement 3 Correct Correct Correct The fresh elections to constitute a panchayat shall be completed before the expiry of its duration of five years or in case of dissolution the expiry of a period of six months from the date of its dissolution. After        the premature dissolution of the panchayat, the reconstituted panchayat does not enjoy a full period of 5years. It shall continue only for the remainder period for which dissolved Panchayat would have continued had it not been so dissolved. It is not necessary to hold elections where the remaining period is less than 6 months (and not five years) for constituting a new panchayat for such period. Incorrect Solution (c) Statement 1 Statement 2 Statement 3 Correct Correct Correct The fresh elections to constitute a panchayat shall be completed before the expiry of its duration of five years or in case of dissolution the expiry of a period of six months from the date of its dissolution. After        the premature dissolution of the panchayat, the reconstituted panchayat does not enjoy a full period of 5years. It shall continue only for the remainder period for which dissolved Panchayat would have continued had it not been so dissolved. It is not necessary to hold elections where the remaining period is less than 6 months (and not five years) for constituting a new panchayat for such period. Question 4 of 35 4. Question Consider the following statements about the reservation of seats for Scheduled Caste (SCs) and Scheduled Tribes (STs) in the Panchayati Raj Institution: Not less than one-third of the total number of seats reserved for SC/ST shall be reserved for women belonging to the respective Reservation for SC and STs extends to the position of Chairperson at all three Which of the statements given above is/are correct? a) 1 only b) 2 only c) Both 1 and 2 d) Neither 1 nor 2 Correct Solution (c) Statement 1 Statement 2 Correct Correct Article 243D (2) states that not less than one- third of the total number of seats reserved under clause (1) shall be reserved for women belonging to the Scheduled Castes or the Scheduled Tribes, as the case may be. Article 243D (1) provides for the reservation of seats for Scheduled Castes (SCs) and Scheduled tribes (STs) at all three levels in proportion to the total population in the panchayat area. Incorrect Solution (c) Statement 1 Statement 2 Correct Correct Article 243D (2) states that not less than one- third of the total number of seats reserved under clause (1) shall be reserved for women belonging to the Scheduled Castes or the Scheduled Tribes, as the case may be. Article 243D (1) provides for the reservation of seats for Scheduled Castes (SCs) and Scheduled tribes (STs) at all three levels in proportion to the total population in the panchayat area. Question 5 of 35 5. Question Consider the following statements about revenue generation/income of urban local bodies: They receive grants from the Centre as well as State Governments for developmental programmes. The devolution of funds from state governments to urban local bodies based on the recommendation of the State Finance Commission. They can borrow from financial institutions with the prior approval of the Governor. How many of the above statements are correct? a) Only one b) Only two c) All three d) None Correct Solution (b) Statement 1 Statement 2 Statement 3 Correct Correct Incorrect Urban Local bodies receive various grants by central and state governments for several development programs, Infrastructure schemes, initiatives in reforms in urban governance, etc. The Devolution of funds consists of the transfer of funds from the state government to urban local bodies on the recommendation of the State Finance Commission. Municipalities raise loans from state government and financial institutions to meet their capital expenditure. however, they required prior approval of the state government (and not the Governor) to raise or borrow the loans from financial institutions.   Incorrect Solution (b) Statement 1 Statement 2 Statement 3 Correct Correct Incorrect Urban Local bodies receive various grants by central and state governments for several development programs, Infrastructure schemes, initiatives in reforms in urban governance, etc. The Devolution of funds consists of the transfer of funds from the state government to urban local bodies on the recommendation of the State Finance Commission. Municipalities raise loans from state government and financial institutions to meet their capital expenditure. however, they required prior approval of the state government (and not the Governor) to raise or borrow the loans from financial institutions.   Question 6 of 35 6. Question The Panchayats (Extension to Scheduled Areas) Act, 1996 was enacted in order to give greater power to grass root democracy in the scheduled areas. Consider the following statements regarding the PESA Act, 1996: It aims to provide for extension of the provisions of Part-IX of the Constitution to areas under Schedule 5 and Schedule 6 of the Constitution. It provides that at least half of the seats for the election of chairpersons of panchayats at each level to be reserved for Scheduled Tribes. The state legislature is to provide for reservation of different communities for election to panchayats in the Scheduled Areas. The state government may nominate members of unrepresented Scheduled Tribes to Panchayats at the Intermediate and District levels only, subject to a limit of one-tenth of total members. How many of the above statements are correct? a) Only one b) Only two c) Only three d) All four Correct Solution (a) Statement 1 Statement 2 Statement 3 Statement 4 Incorrect Incorrect Incorrect Correct It aims to provide for extension of the provisions of Part-IX of the Constitution to areas under Schedule 5 of the Constitution. Schedule 6 areas come under Autonomous District Councils. According to PESA Act, the reservation  of seats in the Scheduled Areas at every Panchayat shall be in proportion  to the population  of the communities in that Panchayat for whom reservation is sought to be given under Part IX of the Constitution;   Provided that the reservation    for the Scheduled Tribes shall not be less than one- half of the total number of seats;   Provided further that all seats of Chairpersons of Panchayats at all levels shall be reserved for the Scheduled Tribes. Reservation of different communities for election to panchayats in the Scheduled Areas is to be according to the proportion of the population of the communities for whom reservation is sought under Part-IX of the Constitution. The State government may nominate members of unrepresented Scheduled Tribes to Panchayats at Intermediate            and District levels            only, subject to a limit of one-tenth of total members. Incorrect Solution (a) Statement 1 Statement 2 Statement 3 Statement 4 Incorrect Incorrect Incorrect Correct It aims to provide for extension of the provisions of Part-IX of the Constitution to areas under Schedule 5 of the Constitution. Schedule 6 areas come under Autonomous District Councils. According to PESA Act, the reservation  of seats in the Scheduled Areas at every Panchayat shall be in proportion  to the population  of the communities in that Panchayat for whom reservation is sought to be given under Part IX of the Constitution;   Provided that the reservation    for the Scheduled Tribes shall not be less than one- half of the total number of seats;   Provided further that all seats of Chairpersons of Panchayats at all levels shall be reserved for the Scheduled Tribes. Reservation of different communities for election to panchayats in the Scheduled Areas is to be according to the proportion of the population of the communities for whom reservation is sought under Part-IX of the Constitution. The State government may nominate members of unrepresented Scheduled Tribes to Panchayats at Intermediate            and District levels            only, subject to a limit of one-tenth of total members. Question 7 of 35 7. Question Which of the following are the features of the 73rd Constitutional Amendment Act, 1992? A three-tier system of Panchayati Raj for all states having a population of over 20 lakhs. Reservation of the seats for the Scheduled Castes, the Scheduled Tribes and the women only. To appoint State Finance Commission to make recommendations regarding the financial powers of the Panchayats. To constitute District Planning Committee to prepare a draft development plan for the district as a whole. How many of the above statements are correct? a) Only one b) Only two c) Only three d) All four Correct Solution (c) Statement 1 Statement 2 Statement 3 Statement 4 Correct Incorrect Correct Correct A three-tier system of Panchayati Raj for all states having a population of over 20 lakhs. Reservation of the seats for the Scheduled Castes, the Scheduled Tribes, the OBCs, the women (33%)  and  general seats. To      appoint State Finance Commission to make recommendations regarding the financial powers of the Panchayats. To constitute District Planning Committee to prepare a draft development plan for the district as a whole. Incorrect Solution (c) Statement 1 Statement 2 Statement 3 Statement 4 Correct Incorrect Correct Correct A three-tier system of Panchayati Raj for all states having a population of over 20 lakhs. Reservation of the seats for the Scheduled Castes, the Scheduled Tribes, the OBCs, the women (33%)  and  general seats. To      appoint State Finance Commission to make recommendations regarding the financial powers of the Panchayats. To constitute District Planning Committee to prepare a draft development plan for the district as a whole. Question 8 of 35 8. Question With reference to ‘Union Public Service Commission (UPSC)’, consider the following statements: Part XIV of the Constitution deals with the appointment powers and functions of the UPSC. The strength of the UPSC is determined by the Ministry of Personnel, Public Grievances and Pensions, Government of India. The Chairman and Members of the UPSC hold office for a term of six years or until they attain the age of 65 years. How many of the above statements are correct? a) Only one b) Only two c) All three d) None Correct Solution (b) Statement 1 Statement 2 Statement 3 Correct Incorrect Correct The Union Public Service Commission (UPSC) is an independent Constitutional body. It has been directly created by the Constitution. Articles 315 to 323 in Part XIV of the Constitution contain elaborate provisions regarding the composition, appointment and removal of members along with the independence, powers and functions of the UPSC. The UPSC consists of a Chairman and other Members appointed by the President of India. The Constitution, without specifying the strength of the Commission has left the matter to the discretion of the President, who determines its composition. Usually, the Commission consists of nine to eleven members including the Chairman. Further, no qualifications are prescribed for the Commission’s membership except that one-half of the members of the Commission should be such persons who have held office for at least ten years either under the Government of India or under the Government of a State. The Chairman and Members of the Commission hold office for a term of six years or until they attain the age of 65 years, whichever is earlier. However, they can relinquish their offices at any time by addressing their resignation to the President. They can also be removed before the expiry of their term by the President in the manner as provided in the Constitution. Incorrect Solution (b) Statement 1 Statement 2 Statement 3 Correct Incorrect Correct The Union Public Service Commission (UPSC) is an independent Constitutional body. It has been directly created by the Constitution. Articles 315 to 323 in Part XIV of the Constitution contain elaborate provisions regarding the composition, appointment and removal of members along with the independence, powers and functions of the UPSC. The UPSC consists of a Chairman and other Members appointed by the President of India. The Constitution, without specifying the strength of the Commission has left the matter to the discretion of the President, who determines its composition. Usually, the Commission consists of nine to eleven members including the Chairman. Further, no qualifications are prescribed for the Commission’s membership except that one-half of the members of the Commission should be such persons who have held office for at least ten years either under the Government of India or under the Government of a State. The Chairman and Members of the Commission hold office for a term of six years or until they attain the age of 65 years, whichever is earlier. However, they can relinquish their offices at any time by addressing their resignation to the President. They can also be removed before the expiry of their term by the President in the manner as provided in the Constitution. Question 9 of 35 9. Question With reference to the Joint State Public Service Commission (JSPSC) in India, consider the following statements: It is a statutory body created by an Act of Parliament. The Chairman and Members of a JSPSC are appointed by the President of India. The Government of India Act of 1919 envisaged establishment of Joint State Public Service Commission. How many of the above statements are correct? a) Only one b) Only two c) All three d) None Correct Solution (b) Statement 1 Statement 2 Statement 3 Correct Correct Incorrect The Constitution makes a provision for the establishment of a Joint State Public Service Commission (JSPSC) for two or more States. While the Union Public Service Commission (UPSC) and the State Public Service Commission (SPSC) are created directly by the Constitution, a JSPSC can be created by an act of Parliament on the request of the State legislatures concerned. Thus, a JSPSC is a statutory and non-constitutional body. The Chairman and Members of a JSPSC are appointed by the President of India. They hold office for a term of six years or until they attain the age of 62 years, whichever is earlier. They can be suspended or removed by the President. They can also resign from their offices at any time by submitting their resignation letters to the President. The number of Members of a JSPSC and their conditions of service are determined by the President. The Government of India Act of 1935 provided for the establishment of not only a Federal Public Service Commission but also a Provincial Public Service Commission and Joint Public Service Commission for two or more Provinces. Incorrect Solution (b) Statement 1 Statement 2 Statement 3 Correct Correct Incorrect The Constitution makes a provision for the establishment of a Joint State Public Service Commission (JSPSC) for two or more States. While the Union Public Service Commission (UPSC) and the State Public Service Commission (SPSC) are created directly by the Constitution, a JSPSC can be created by an act of Parliament on the request of the State legislatures concerned. Thus, a JSPSC is a statutory and non-constitutional body. The Chairman and Members of a JSPSC are appointed by the President of India. They hold office for a term of six years or until they attain the age of 62 years, whichever is earlier. They can be suspended or removed by the President. They can also resign from their offices at any time by submitting their resignation letters to the President. The number of Members of a JSPSC and their conditions of service are determined by the President. The Government of India Act of 1935 provided for the establishment of not only a Federal Public Service Commission but also a Provincial Public Service Commission and Joint Public Service Commission for two or more Provinces. Question 10 of 35 10. Question Consider the following statements regarding the Finance Commission of India: The Members of the Finance Commission are not eligible for any employment after completion of their tenure. The Constitution of India authorises the President to determine the qualifications of Members of the Finance Commission. Which of the statements given above is/are not correct? a) 1 only b) 2 only c) Both 1 and 2 d) Neither 1 nor 2 Correct Solution (c) Statement 1 Statement 2 Incorrect Incorrect The Finance Commission consists of a Chairman and four other Members to be appointed by the President. They hold office for such a period as specified by the President in his order. They are eligible for reappointment. The Indian Constitution authorises the Parliament to determine the qualifications of Members of the commission and the manner in which they should be selected. Accordingly, the Parliament has specified the qualifications of the Chairman and Members of the commission. Incorrect Solution (c) Statement 1 Statement 2 Incorrect Incorrect The Finance Commission consists of a Chairman and four other Members to be appointed by the President. They hold office for such a period as specified by the President in his order. They are eligible for reappointment. The Indian Constitution authorises the Parliament to determine the qualifications of Members of the commission and the manner in which they should be selected. Accordingly, the Parliament has specified the qualifications of the Chairman and Members of the commission. Question 11 of 35 11. Question With reference to the National Commission for Scheduled Castes in India, consider the following statements: Article 338A of the Indian Constitution envisages the establishment of a National Commission for Scheduled Castes. The conditions of service and tenure of office of Chairperson of National Commission for Scheduled Castes is determined by the President of India. Which of the statements given above is/are correct? a) 1 only b) 2 only c) Both 1 and 2 d) Neither 1 nor 2 Correct Solution (b) Statement 1 Statement 2 Incorrect Correct National Commission for Scheduled Castes (NCSC) is a constitutional body established directly by Article 338 of the Constitution. The 89th Constitutional Amendment Act of 2003 bifurcated the combined National Commission for SCs and STs (Scheduled Castes and Scheduled Tribes) into two separate bodies, namely, National Commission for Scheduled Castes (under Article 338) and National Commission for Scheduled Tribes (under Article 338A). The NCSC consists of a Chairperson, a Vice-Chairperson and three other Members. They are appointed by the President by warrant under his hand and seal. Their conditions of service and tenure of office are also determined by the President. The commission presents an annual report to the President. The President places all such reports before the Parliament, along with a memorandum explaining the action taken on the recommendations made by the Commission. Incorrect Solution (b) Statement 1 Statement 2 Incorrect Correct National Commission for Scheduled Castes (NCSC) is a constitutional body established directly by Article 338 of the Constitution. The 89th Constitutional Amendment Act of 2003 bifurcated the combined National Commission for SCs and STs (Scheduled Castes and Scheduled Tribes) into two separate bodies, namely, National Commission for Scheduled Castes (under Article 338) and National Commission for Scheduled Tribes (under Article 338A). The NCSC consists of a Chairperson, a Vice-Chairperson and three other Members. They are appointed by the President by warrant under his hand and seal. Their conditions of service and tenure of office are also determined by the President. The commission presents an annual report to the President. The President places all such reports before the Parliament, along with a memorandum explaining the action taken on the recommendations made by the Commission. Question 12 of 35 12. Question Consider the following statements regarding the National Commission for Scheduled Tribes in India: A Separate National Commission for Scheduled Tribes came into existence in 2003. The National Commission for Scheduled Tribes, while investigating any matter or inquiring into any complaint, is vested with all the powers of a Civil Court. Which of the statements given above is/are correct? a) 1 only b) 2 only c) Both 1 and 2 d) Neither 1 nor 2 Correct Solution (c) Statement 1 Statement 2 Correct Correct The 89th Constitutional Amendment Act of 2003 bifurcated the National Commission for SCs and STs (Scheduled Castes and Scheduled Tribes). It established the National Commission for Scheduled Tribes in 2004 as a separate Constitutional body. The National Commission for Scheduled Tribes consists of a Chairperson, a Vice-Chairperson and three other Members. They are appointed by the President by warrant under his hand and seal. Their conditions of service and tenure of office are also determined by the President.   The National Commission for Scheduled Tribes, while investigating any matter or inquiring into any complaint, is vested with all the powers of a Civil Court trying a suit. Incorrect Solution (c) Statement 1 Statement 2 Correct Correct The 89th Constitutional Amendment Act of 2003 bifurcated the National Commission for SCs and STs (Scheduled Castes and Scheduled Tribes). It established the National Commission for Scheduled Tribes in 2004 as a separate Constitutional body. The National Commission for Scheduled Tribes consists of a Chairperson, a Vice-Chairperson and three other Members. They are appointed by the President by warrant under his hand and seal. Their conditions of service and tenure of office are also determined by the President.   The National Commission for Scheduled Tribes, while investigating any matter or inquiring into any complaint, is vested with all the powers of a Civil Court trying a suit. Question 13 of 35 13. Question Consider the following statements regarding the Special Officer for Linguistic Minorities in India: The Special Officer for Linguistic Minorities was mentioned at the commencement of the Constitution of India. Only a person qualified to be Judge of a High Court can be appointed as the Special Officer for Linguistic Minorities. Which of the statements given above is/are not correct? a) 1 only b) 2 only c) Both 1 and 2 d) Neither 1 nor 2 Correct Solution (c) Statement 1 Statement 2 Incorrect Incorrect Originally, the Constitution of India did not make any provision with respect to the Special Officer for Linguistic Minorities. Later, the States Reorganisation Commission (1953-55) made a recommendation in this regard. Accordingly, the Seventh Constitutional Amendment Act of 1956 inserted a new Article 350B in Part XVII of the Indian Constitution which recommended that there should be a Special Officer for Linguistic Minorities. He is to be appointed by the President of India. The Indian Constitution does not specify the qualifications, tenure, salaries and allowances, service conditions and procedure for removal of the Special Officer for Linguistic Minorities. He investigates all matters related to safeguards provided to the linguistic minorities. Incorrect Solution (c) Statement 1 Statement 2 Incorrect Incorrect Originally, the Constitution of India did not make any provision with respect to the Special Officer for Linguistic Minorities. Later, the States Reorganisation Commission (1953-55) made a recommendation in this regard. Accordingly, the Seventh Constitutional Amendment Act of 1956 inserted a new Article 350B in Part XVII of the Indian Constitution which recommended that there should be a Special Officer for Linguistic Minorities. He is to be appointed by the President of India. The Indian Constitution does not specify the qualifications, tenure, salaries and allowances, service conditions and procedure for removal of the Special Officer for Linguistic Minorities. He investigates all matters related to safeguards provided to the linguistic minorities. Question 14 of 35 14. Question Which of the following statements is/are correct about the Comptroller and Auditor General (CAG) of India? He is the guardian of the public purse and controls the entire financial system of the country at the Centre level only. He acts as a guide, friend, and philosopher of the Estimates Committee of the Parliament. Select the correct answer using the code given below: a) 1 only b) 2 only c) Both 1 and 2 d) Neither 1 nor 2 Correct Solution (d) Statement 1 Statement 2 Incorrect Incorrect The Constitution of India (Article 148) provides for an independent office of the Comptroller and Auditor General of India (CAG). He is the Head of the Indian Audit and Accounts Department. He is the guardian of the public purse and controls the entire financial system of the country at both the levels—the Centre and the State. His duty is to uphold the Constitution of India and laws of Parliament in the field of financial administration. He acts as a guide, friend, and philosopher of the Public Accounts Committee (PAC) of the Parliament. He audits the receipts and expenditure of the Centre and each State to satisfy himself that the rules and procedures in that behalf are designed to secure an effective check on the assessment, collection, and proper allocation of revenue. Incorrect Solution (d) Statement 1 Statement 2 Incorrect Incorrect The Constitution of India (Article 148) provides for an independent office of the Comptroller and Auditor General of India (CAG). He is the Head of the Indian Audit and Accounts Department. He is the guardian of the public purse and controls the entire financial system of the country at both the levels—the Centre and the State. His duty is to uphold the Constitution of India and laws of Parliament in the field of financial administration. He acts as a guide, friend, and philosopher of the Public Accounts Committee (PAC) of the Parliament. He audits the receipts and expenditure of the Centre and each State to satisfy himself that the rules and procedures in that behalf are designed to secure an effective check on the assessment, collection, and proper allocation of revenue. Question 15 of 35 15. Question Consider the following statements about the historical background of the rural self-government in India: The formal structure of the rural self-government was introduced by Ripon’s Resolution. According to the recommendations of the Royal Commission of Decentralization, the Panchayats were placed under the control of the local boards. The Rural Self-Government Bill, 1925, provided for a 9-member village authority, elected on the basis of the universal adult franchise. How many of the above statements are correct? a) Only one b) Only two c) All three d) None Correct Solution (a) Statement 1 Statement 2 Statement 3 Correct Incorrect Incorrect The formal structure of the rural self-government was introduced in 1882, according to Ripon’s Resolution. Its main purpose was to provide the institutional backing of the local Indian elites to the colonial administration. Contemporary local self-government in India can be deemed as more a continuation of the system introduced by the British, rather than in the pre-British era. According to the recommendations of the Royal Commission of Decentralization, the Panchayats were not to be placed under the control of the local boards, but the Deputy Commissioner. The village Panchayat enjoyed certain judicial and administrative powers. It was also entitled to a portion of land cesses and special grants. The Rural Self-Government Bill, 1925 (The Commonwealth of India Bill 1925), provided for a 9-member village authority, elected on the basis of the restricted adult franchise. A successful village authority was to be given more power. A Panchayat could include more than one village. Incorrect Solution (a) Statement 1 Statement 2 Statement 3 Correct Incorrect Incorrect The formal structure of the rural self-government was introduced in 1882, according to Ripon’s Resolution. Its main purpose was to provide the institutional backing of the local Indian elites to the colonial administration. Contemporary local self-government in India can be deemed as more a continuation of the system introduced by the British, rather than in the pre-British era. According to the recommendations of the Royal Commission of Decentralization, the Panchayats were not to be placed under the control of the local boards, but the Deputy Commissioner. The village Panchayat enjoyed certain judicial and administrative powers. It was also entitled to a portion of land cesses and special grants. The Rural Self-Government Bill, 1925 (The Commonwealth of India Bill 1925), provided for a 9-member village authority, elected on the basis of the restricted adult franchise. A successful village authority was to be given more power. A Panchayat could include more than one village. Question 16 of 35 16. Question Consider the following statements about the Gram Panchayat: The Gram Panchayat consists of the Gram Sabha and the members of the village Panchayats. The village Pradhans are directly elected by the members of the village Panchayat. The Gram Panchayat’s jurisdiction covers all 29 subjects mentioned in the Eleventh Schedule. How many of the above statements are correct? a) Only one b) Only two c) All three d) None Correct Solution (b) Statement 1 Statement 2 Statement 3 Correct Incorrect Correct The Gram Panchayat consists of the Gram Sabha and the members of the village Panchayats, who are directly elected by the electorate and headed by the Pradhans. The village Pradhans are elected indirectly by the members of the village Panchayat. The Gram Panchayat’s jurisdiction covers all 29 subjects mentioned in the Eleventh Schedule. The Panchayats are supposed to consist of several committees to help them in performing various duties. Incorrect Solution (b) Statement 1 Statement 2 Statement 3 Correct Incorrect Correct The Gram Panchayat consists of the Gram Sabha and the members of the village Panchayats, who are directly elected by the electorate and headed by the Pradhans. The village Pradhans are elected indirectly by the members of the village Panchayat. The Gram Panchayat’s jurisdiction covers all 29 subjects mentioned in the Eleventh Schedule. The Panchayats are supposed to consist of several committees to help them in performing various duties. Question 17 of 35 17. Question Which of the following statements are correct about the Balwant Rai Mehta Committee? This was the first initiative to introduce the Panchayati Raj Institute in the post-Independence India. It recommended the measures for ‘democratic decentralization’. It suggested that the power for development should be located in the intermediate level – the Panchayat Samiti. The first village Panchayat in India was elected in Maharashtra. How many of the above statements are correct? a) Only one b) Only two c) Only three d) All four Correct Solution (c) Statement 1 Statement 2 Statement 3 Statement 4 Correct Correct Correct Incorrect The first initiative to introduce the Panchayat Raj Institute (PRI) in the post-Independence India was taken in January, 1957. At that time, the Planning Commission appointed a Committee on Plan Projects. The Committee was known as the Mehta Committee, which was named after its Chairman, Balwant Rai G. Mehta. It recommended the measures for ‘democratic decentralization’, in order to meet the deficiency of the Community Development Programmes and Extension Services Programmes. It suggested that the power for development should be located in the intermediate level – the Panchayat Samiti. The Mehta Committee report made the Village Level Worker (VLWs) or the Gram Sewaks as a link between the Panchayat Samiti and the village level Panchayat. The Mehta Committee Report became the basis of the extension of the PRIs all over India. The first village Panchayat in India was elected in 1957, in Nagaur district of Rajasthan. Incorrect Solution (c) Statement 1 Statement 2 Statement 3 Statement 4 Correct Correct Correct Incorrect The first initiative to introduce the Panchayat Raj Institute (PRI) in the post-Independence India was taken in January, 1957. At that time, the Planning Commission appointed a Committee on Plan Projects. The Committee was known as the Mehta Committee, which was named after its Chairman, Balwant Rai G. Mehta. It recommended the measures for ‘democratic decentralization’, in order to meet the deficiency of the Community Development Programmes and Extension Services Programmes. It suggested that the power for development should be located in the intermediate level – the Panchayat Samiti. The Mehta Committee report made the Village Level Worker (VLWs) or the Gram Sewaks as a link between the Panchayat Samiti and the village level Panchayat. The Mehta Committee Report became the basis of the extension of the PRIs all over India. The first village Panchayat in India was elected in 1957, in Nagaur district of Rajasthan. Question 18 of 35 18. Question Consider the following statements regarding Municipalities: All the members of a municipality shall be elected indirectly by the people of the municipal area. The state legislature may provide for the manner of reservation of offices of chairpersons in the municipalities for SCs, STs and women. Which of the statements given above is/are correct? a) 1 only b) 2 only c) Both 1 and 2 d) Neither 1 nor 2 Correct Solution (b) Statement 1 Statement 2 Incorrect Correct All the members of a municipality shall be elected directly by the people of the municipal area. The state legislature may provide for the manner of reservation of offices of chairpersons in the municipalities for SCs, STs and women.   Incorrect Solution (b) Statement 1 Statement 2 Incorrect Correct All the members of a municipality shall be elected directly by the people of the municipal area. The state legislature may provide for the manner of reservation of offices of chairpersons in the municipalities for SCs, STs and women.   Question 19 of 35 19. Question With reference to the State Public Service Commission (SPSC), consider the following statements: The President of India can remove the Chairman of SPSC in the same manner as he can remove a Chairman of the Union Public Service Commission (UPSC). The Constitution of India has mentioned the term ‘misbehaviour’ in the context of the removal of a Chairman of SPSC. Which of the statements given above is/are correct? a) 1 only b) 2 only c) Both 1 and 2 d) Neither 1 nor 2 Correct Solution (c) Statement 1 Statement 2 Correct Correct The Chairman and members of an SPSC are appointed by the Governor; they can be removed only by the President of India (and not by the Governor). The President can remove them on the same grounds and in the same manner as he can remove a Chairman or a member of the UPSC. The Constitution has Mentioned the term ‘misbehaviour’ in the context of removal of member or Chairman of SPSC. Incorrect Solution (c) Statement 1 Statement 2 Correct Correct The Chairman and members of an SPSC are appointed by the Governor; they can be removed only by the President of India (and not by the Governor). The President can remove them on the same grounds and in the same manner as he can remove a Chairman or a member of the UPSC. The Constitution has Mentioned the term ‘misbehaviour’ in the context of removal of member or Chairman of SPSC. Question 20 of 35 20. Question With reference to the Attorney General of India, consider the following statements: A person who is qualified to be appointed a Judge of the Supreme Court is eligible for the post of the Attorney General of India. The term of office of the Attorney General of India is not fixed by the Indian Constitution. Which of the statements given above is/are correct? a) 1 only b) 2 only c) Both 1 and 2 d) Neither 1 nor 2 Correct Solution (c) Statement 1 Statement 2 Correct Correct Article 76 has provided for the office of the Attorney General for India. He is the highest law officer in the country. The Attorney General (AG) is appointed by the President. He must be a person who is qualified to be appointed a Judge of the Supreme Court. In other words, he must be a Citizen of India and he must have been a Judge of some High Court for five years or an advocate of some High Court for ten years or an Eminent jurist, in the opinion of the President. The term of office of the Attorney General of India is not fixed by the Constitution. Further, the Constitution does not contain the procedure and grounds for his removal. He holds office during the pleasure of the President. This means that he may be removed by the President at any time. He may also quit his office by submitting his resignation to the President. Conventionally, he resigns when the Government (Council of Ministers) resigns or is replaced, as he is appointed on its advice. Incorrect Solution (c) Statement 1 Statement 2 Correct Correct Article 76 has provided for the office of the Attorney General for India. He is the highest law officer in the country. The Attorney General (AG) is appointed by the President. He must be a person who is qualified to be appointed a Judge of the Supreme Court. In other words, he must be a Citizen of India and he must have been a Judge of some High Court for five years or an advocate of some High Court for ten years or an Eminent jurist, in the opinion of the President. The term of office of the Attorney General of India is not fixed by the Constitution. Further, the Constitution does not contain the procedure and grounds for his removal. He holds office during the pleasure of the President. This means that he may be removed by the President at any time. He may also quit his office by submitting his resignation to the President. Conventionally, he resigns when the Government (Council of Ministers) resigns or is replaced, as he is appointed on its advice. Question 21 of 35 21. Question Consider the following pairs: Glacier Country 1.     Jostedalsbreen Glacier Norway 2.     Stuorrajekna Glacier Antarctica 3.     Thwaites Glacier Sweden 4.     Bering Glacier United States of America How many pairs are correctly matched? a) One pair b) Two pairs c) Three pairs d) Four pairs Correct Solution (b) Glacier Country 1.     Jostedalsbreen Glacier Norway 2.     Stuorrajekna Glacier Sweden 3.     Thwaites Glacier Antarctica 4.     Bering Glacier United States of America Hence option b is correct. Note: Thwaites Glacier, also known as the “Doomsday Glacier,” is located in the remote Amundsen Sea in West Antarctica. It’s a wide and fast-flowing glacier, roughly the size of Florida or Great Britain, and it has been a significant focus of scientific study due to its sensitivity to climate change. It is one of the most vulnerable and important glaciers in the world in terms of future global sea-level rise. Satellite measurements have shown that the glacier is losing an enormous amount of ice each year, nearly 50 billion tons annually, contributing to rising global sea levels. The glacier already contributes 4% of global sea level rise. If the entire Thwaites Glacier were to melt, it could raise the world’s oceans by about 65 centimeters (over 2 feet). It also acts as a buffer, holding back neighboring glaciers that contain around three meters of potential sea level rise. Incorrect Solution (b) Glacier Country 1.     Jostedalsbreen Glacier Norway 2.     Stuorrajekna Glacier Sweden 3.     Thwaites Glacier Antarctica 4.     Bering Glacier United States of America Hence option b is correct. Note: Thwaites Glacier, also known as the “Doomsday Glacier,” is located in the remote Amundsen Sea in West Antarctica. It’s a wide and fast-flowing glacier, roughly the size of Florida or Great Britain, and it has been a significant focus of scientific study due to its sensitivity to climate change. It is one of the most vulnerable and important glaciers in the world in terms of future global sea-level rise. Satellite measurements have shown that the glacier is losing an enormous amount of ice each year, nearly 50 billion tons annually, contributing to rising global sea levels. The glacier already contributes 4% of global sea level rise. If the entire Thwaites Glacier were to melt, it could raise the world’s oceans by about 65 centimeters (over 2 feet). It also acts as a buffer, holding back neighboring glaciers that contain around three meters of potential sea level rise. Question 22 of 35 22. Question Consider the following statements regarding Vikramaditya Vedic Clock: It is the world’s first Vedic Clock designed to display time according to the ancient Indian traditional Panchang. It has been positioned on a tower in Uttar Pradesh. It provides information on planetary positions, muhurat, astrological calculations, and predictions. It indicates Indian Standard Time (IST) and Greenwich Mean Time (GMT). How many of the above statements are correct? a) Only one b) Only two c) Only three d) All four Correct Solution (c) Vikramaditya Vedic Clock is the world’s first Vedic Clock designed to display time according to the ancient Indian traditional Panchang. Hence statement 1 is correct. The clock will calculate time from one sunrise to another. The period between the two sunrises will be divided into 30 parts, whose one hour consists of 48 minutes, according to ISD.  The reading will start from 0:00 with the sunrise functions for 30 hours (an hour of 48 minutes). It has been positioned on an 85-foot tower within Jantar Mantar in Ujjain, Madhya Pradesh. Hence statement 2 is incorrect. Ujjain’s rich heritage in timekeeping dates back centuries, with the city playing a pivotal role in determining India’s time zones and time differences. Ujjain is located at the precise point of interaction with the zero meridian and the Tropic of Cancer. Before 82.5E longitude was adopted for IST, Ujjain (75.78E) was considered as Bharat’s time meridian. It provides information on planetary positions, muhurat, astrological calculations, and predictions. Hence statement 3 is correct. It has been developed by Lucknow-based Sanstha Arohan, using digital interventions, enabling it to be connected to the internet and provide a wide range of features through a mobile app named after the clock. It indicates Indian Standard Time (IST) and Greenwich Mean Time (GMT). Hence statement 4 is correct. Incorrect Solution (c) Vikramaditya Vedic Clock is the world’s first Vedic Clock designed to display time according to the ancient Indian traditional Panchang. Hence statement 1 is correct. The clock will calculate time from one sunrise to another. The period between the two sunrises will be divided into 30 parts, whose one hour consists of 48 minutes, according to ISD.  The reading will start from 0:00 with the sunrise functions for 30 hours (an hour of 48 minutes). It has been positioned on an 85-foot tower within Jantar Mantar in Ujjain, Madhya Pradesh. Hence statement 2 is incorrect. Ujjain’s rich heritage in timekeeping dates back centuries, with the city playing a pivotal role in determining India’s time zones and time differences. Ujjain is located at the precise point of interaction with the zero meridian and the Tropic of Cancer. Before 82.5E longitude was adopted for IST, Ujjain (75.78E) was considered as Bharat’s time meridian. It provides information on planetary positions, muhurat, astrological calculations, and predictions. Hence statement 3 is correct. It has been developed by Lucknow-based Sanstha Arohan, using digital interventions, enabling it to be connected to the internet and provide a wide range of features through a mobile app named after the clock. It indicates Indian Standard Time (IST) and Greenwich Mean Time (GMT). Hence statement 4 is correct. Question 23 of 35 23. Question Consider the following statements regarding Nano Urea: It is a nanotechnology-based revolutionary Agri-input that provides phosphorous to plants. It is developed and patented by the Indian Farmers Fertiliser Cooperative Limited (IFFCO). Choose the correct code: a) 1 only b) 2 only c) Both 1 and 2 d) Neither 1 nor 2 Correct Solution (b) Nano Urea is a nanotechnology-based revolutionary Agri-input that provides nitrogen to plants. Hence statement 1 is incorrect. When compared to conventional urea prill, Nano Urea has a desirable particle size of about 20-50 nm,  more surface area (10,000 times over 1 mm urea prill), and several particles (55,000 nitrogen particles over 1 mm urea prill). It contains 4.0 % total nitrogen (w/v). It is produced by an energy-efficient, environment-friendly production process with less carbon footprints. It is expected to improve crop productivity, soil health, and nutritional quality of produce and address the “imbalanced and excessive use” of conventional fertilizer. It is developed and patented by the Indian Farmers Fertiliser Cooperative Limited (IFFCO). Hence statement 2 is correct. IFFCO Nano Urea is the only nano fertilizer approved by the Government of India and included in the Fertilizer Control Order (FCO). Incorrect Solution (b) Nano Urea is a nanotechnology-based revolutionary Agri-input that provides nitrogen to plants. Hence statement 1 is incorrect. When compared to conventional urea prill, Nano Urea has a desirable particle size of about 20-50 nm,  more surface area (10,000 times over 1 mm urea prill), and several particles (55,000 nitrogen particles over 1 mm urea prill). It contains 4.0 % total nitrogen (w/v). It is produced by an energy-efficient, environment-friendly production process with less carbon footprints. It is expected to improve crop productivity, soil health, and nutritional quality of produce and address the “imbalanced and excessive use” of conventional fertilizer. It is developed and patented by the Indian Farmers Fertiliser Cooperative Limited (IFFCO). Hence statement 2 is correct. IFFCO Nano Urea is the only nano fertilizer approved by the Government of India and included in the Fertilizer Control Order (FCO). Question 24 of 35 24. Question Consider the following statements regarding the functions of the Narcotics Control Bureau (NCB): It is a central authority to effectively prevent and combat the abuse of narcotic drugs and psychotropic substances and their illicit trade. It works to improve awareness, educate the public on the dangers of drugs and drug abuse, and provide support to those affected by the illegal drug trade. It is responsible for maintaining and updating the National Data Bank on Narcotic Drugs and Psychotropic Substances. It maintains and updates the National Register of Treatment Providers and Narcotic Drug Dependents. How many of the above statements are correct? a) Only one b) Only two c) Only three d) All four Correct Solution (d) The functions of the Narcotics Control Bureau (NCB): It is a central authority to effectively prevent and combat the abuse of narcotic drugs and psychotropic substances and their illicit trade. Hence statement 1 is correct. It is the apex coordinating agency. It also functions as an enforcement agency through its zones and sub-zones. It works to improve awareness, educate the public on the dangers of drugs and drug abuse, and provide support to those affected by the illegal drug trade. Hence statement 2 is correct. It assists concerned authorities in foreign countries and concerned international organisations to facilitate coordination and universal action for the prevention and suppression of illicit traffic in these drugs and substances. It is responsible for maintaining and updating the National Data Bank on Narcotic Drugs and Psychotropic Substances. Hence statement 3 is correct. It provides coordination of actions by various offices, State Governments, and other authorities under the N.D.P.S. Act, Customs Act, Drugs and Cosmetics Act, and any other law for the time being in force in connection with the enforcement provisions of the NDPS Act, 1985. It maintains and updates the National Register of Treatment Providers and Narcotic Drug Dependents. Hence statement 4 is correct. It provides expert advice to the central government on narcotics, psychotropic substances, and related matters. Note: The NCB was created in March 1986 in terms of Section 4(3) of the Narcotic Drugs and Psychotropic Substances Act of 1985. Incorrect Solution (d) The functions of the Narcotics Control Bureau (NCB): It is a central authority to effectively prevent and combat the abuse of narcotic drugs and psychotropic substances and their illicit trade. Hence statement 1 is correct. It is the apex coordinating agency. It also functions as an enforcement agency through its zones and sub-zones. It works to improve awareness, educate the public on the dangers of drugs and drug abuse, and provide support to those affected by the illegal drug trade. Hence statement 2 is correct. It assists concerned authorities in foreign countries and concerned international organisations to facilitate coordination and universal action for the prevention and suppression of illicit traffic in these drugs and substances. It is responsible for maintaining and updating the National Data Bank on Narcotic Drugs and Psychotropic Substances. Hence statement 3 is correct. It provides coordination of actions by various offices, State Governments, and other authorities under the N.D.P.S. Act, Customs Act, Drugs and Cosmetics Act, and any other law for the time being in force in connection with the enforcement provisions of the NDPS Act, 1985. It maintains and updates the National Register of Treatment Providers and Narcotic Drug Dependents. Hence statement 4 is correct. It provides expert advice to the central government on narcotics, psychotropic substances, and related matters. Note: The NCB was created in March 1986 in terms of Section 4(3) of the Narcotic Drugs and Psychotropic Substances Act of 1985. Question 25 of 35 25. Question Which of the following constitutes India’s foreign exchange reserves? Foreign currency assets (FCAs) Gold Special Drawing Rights (SDR) Reserve Tranche Position (RTP) Choose the correct code: a) Only one b) Only two c) Only three d) All four Correct Solution (d) Foreign Exchange Reserves (also called Forex Reserves) are reserve assets held by a central bank in foreign currencies. Reserves are denominated and expressed in the US dollar, which is the international numeraire for the purpose. RBI is the custodian of the foreign exchange reserves in India. India’s foreign exchange reserves comprise: Foreign currency assets (FCAs) are maintained in currencies like the US dollar, euro, pound sterling, Australian dollar, and Japanese yen. Gold SDR (Special Drawing Rights) is the reserve currency of the IMF. RTP (Reserve Tranche Position) is the reserve capital with the IMF. Hence option d is correct. The biggest contributor to India’s Forex reserves is foreign currency assets, followed by gold. Incorrect Solution (d) Foreign Exchange Reserves (also called Forex Reserves) are reserve assets held by a central bank in foreign currencies. Reserves are denominated and expressed in the US dollar, which is the international numeraire for the purpose. RBI is the custodian of the foreign exchange reserves in India. India’s foreign exchange reserves comprise: Foreign currency assets (FCAs) are maintained in currencies like the US dollar, euro, pound sterling, Australian dollar, and Japanese yen. Gold SDR (Special Drawing Rights) is the reserve currency of the IMF. RTP (Reserve Tranche Position) is the reserve capital with the IMF. Hence option d is correct. The biggest contributor to India’s Forex reserves is foreign currency assets, followed by gold. Question 26 of 35 26. Question Consider the following statements regarding the Bureau of Energy Efficiency: It was established under the provisions of the Energy Conservation Act of 2001. It develops minimum energy performance standards for equipment and appliances under standards and labelling. It works under the Ministry of New and Renewable Energy (MNRE). How many of the above statements are correct? a) Only one b) Only two c) All three d) None Correct Solution (b) The Bureau of Energy Efficiency was established under the provisions of the Energy Conservation Act of 2001. Hence statement 1 is correct. The primary objective of BEE is to reduce energy intensity in the Indian economy. It coordinates with designated consumers, designated agencies, and other organizations; recognizes, identifies, and utilizes the existing resources and infrastructure, in performing the functions assigned to it under the Energy Conservation Act. The EC Act provides for regulatory and promotional functions which are assigned to the organisation. It develops minimum energy performance standards for equipment and appliances under standards and labelling. Hence statement 2 is correct. It develops minimum energy performance standards for commercial buildings. It develops energy consumption norms for designated consumers. It works under the Ministry of Power. Hence statement 3 is incorrect. Note: State Energy Efficiency Index 2023 is the fifth edition of The State Energy Efficiency Index (SEEI), initiated by the Bureau of Energy Efficiency (BEE), in association with the Alliance for an Energy-Efficient Economy. It is to evaluate the annual progress of energy efficiency implementation in the states. Incorrect Solution (b) The Bureau of Energy Efficiency was established under the provisions of the Energy Conservation Act of 2001. Hence statement 1 is correct. The primary objective of BEE is to reduce energy intensity in the Indian economy. It coordinates with designated consumers, designated agencies, and other organizations; recognizes, identifies, and utilizes the existing resources and infrastructure, in performing the functions assigned to it under the Energy Conservation Act. The EC Act provides for regulatory and promotional functions which are assigned to the organisation. It develops minimum energy performance standards for equipment and appliances under standards and labelling. Hence statement 2 is correct. It develops minimum energy performance standards for commercial buildings. It develops energy consumption norms for designated consumers. It works under the Ministry of Power. Hence statement 3 is incorrect. Note: State Energy Efficiency Index 2023 is the fifth edition of The State Energy Efficiency Index (SEEI), initiated by the Bureau of Energy Efficiency (BEE), in association with the Alliance for an Energy-Efficient Economy. It is to evaluate the annual progress of energy efficiency implementation in the states. Question 27 of 35 27. Question Consider the following statements regarding Very Short-Range Air Defence System (VSHORADS): It is a long-range air defence capability to protect ground forces and critical assets from aerial threats. It has been designed and developed indigenously by Defence Research and Development Organisation (DRDO) Research Centre Imarat (RCI). The missile and its launcher were designed by the DRDO to be portable. How many of the above statements are correct? a) Only one b) Only two c) All three d) None Correct Solution (b) Very Short-Range Air Defence System (VSHORADS) is a short-range air defence capability to protect ground forces and critical assets from aerial threats, including helicopters and low-flying aircraft. Hence statement 1 is incorrect. is a fourth-generation Man Portable Air Defence System (MANPAD) specially designed to counter low-altitude aerial threats over short distances. It has a range of up to 6 km. It has been designed and developed indigenously by Defence Research and Development Organisation (DRDO) Research Centre Imarat (RCI). Hence statement 2 is correct. The missile and its launcher were designed by the DRDO to be portable, which enables their quick deployment over difficult terrain. Hence statement 3 is correct. It has a dual-band IIR Seeker, a miniaturized Reaction Control System, and is propelled by a dual-thrust solid motor.   Incorrect Solution (b) Very Short-Range Air Defence System (VSHORADS) is a short-range air defence capability to protect ground forces and critical assets from aerial threats, including helicopters and low-flying aircraft. Hence statement 1 is incorrect. is a fourth-generation Man Portable Air Defence System (MANPAD) specially designed to counter low-altitude aerial threats over short distances. It has a range of up to 6 km. It has been designed and developed indigenously by Defence Research and Development Organisation (DRDO) Research Centre Imarat (RCI). Hence statement 2 is correct. The missile and its launcher were designed by the DRDO to be portable, which enables their quick deployment over difficult terrain. Hence statement 3 is correct. It has a dual-band IIR Seeker, a miniaturized Reaction Control System, and is propelled by a dual-thrust solid motor.   Question 28 of 35 28. Question Consider the following statements regarding Juice Jacking: It is an attack carried out by hackers through a USB charging cable. It is used by hackers to steal users’ passwords, credit card information, addresses, and other sensitive data stored on the targeted device. Choose the correct code: a) 1 only b) 2 only c) Both 1 and 2 d) Neither 1 nor 2 Correct Solution (c) Juice Jacking is an attack carried out by hackers through a USB charging cable. Hence statement 1 is correct. The term “juice jacking” was first coined in 2011 by investigative journalist Brian Krebs. It is a form of cyberattack where a public USB charging port is tampered with and infected using hardware and software changes to steal data or install malware on devices connected to it. It is used by hackers to steal users’ passwords, credit card information, addresses, and other sensitive data stored on the targeted device. Hence statement 2 is correct. This type of attack has been a growing concern, with incidents reported in various public spaces such as airports, hotels, and shopping centres. Incorrect Solution (c) Juice Jacking is an attack carried out by hackers through a USB charging cable. Hence statement 1 is correct. The term “juice jacking” was first coined in 2011 by investigative journalist Brian Krebs. It is a form of cyberattack where a public USB charging port is tampered with and infected using hardware and software changes to steal data or install malware on devices connected to it. It is used by hackers to steal users’ passwords, credit card information, addresses, and other sensitive data stored on the targeted device. Hence statement 2 is correct. This type of attack has been a growing concern, with incidents reported in various public spaces such as airports, hotels, and shopping centres. Question 29 of 35 29. Question Consider the following statements: International Big Cat Alliance is an initiative launched by India to commemorate the 50th anniversary of Project Tiger. All the seven cats recognised under the International Big Cat Alliance are found in India. Choose the correct code: a) 1 only b) 2 only c) Both 1 and 2 d) Neither 1 nor 2 Correct Solution (a) International Big Cat Alliance is an initiative launched by India to commemorate the 50th anniversary of Project Tiger. Hence statement 1 is correct. The objective of the IBCA is to ensure cooperation for the conservation of seven big cats – lion, tiger, leopard, cheetah, snow leopard, jaguar, and puma. It is open to 97 ‘range’ countries, which contain the natural habitat of these big cats, as well as other interested nations, international organizations, etc. It aims for cooperation among countries for mutual benefit in furthering the conservation agenda. It would have a multipronged approach in broad basing and establishing linkages manifold in several areas and help in knowledge sharing, capacity building, networking, advocacy, finance and resources support, research and technical support, education, and awareness. It has a General Assembly consisting of all member countries. A Council of at least seven but not more than 15 member countries elected by the General Assembly for a term of 5 years, and a Secretariat. Upon the recommendation of the Council, the General Assembly will appoint the IBCA Secretary General for a specific term. It has secured the Government of India’s initial support of Rs. 150 crore for five years (2023-24 to 2027-28). Five of the seven cats recognized under the International Big Cat Alliance are found in India. Apart from jaguar and puma, are found in India. Hence statement 2 is incorrect. Incorrect Solution (a) International Big Cat Alliance is an initiative launched by India to commemorate the 50th anniversary of Project Tiger. Hence statement 1 is correct. The objective of the IBCA is to ensure cooperation for the conservation of seven big cats – lion, tiger, leopard, cheetah, snow leopard, jaguar, and puma. It is open to 97 ‘range’ countries, which contain the natural habitat of these big cats, as well as other interested nations, international organizations, etc. It aims for cooperation among countries for mutual benefit in furthering the conservation agenda. It would have a multipronged approach in broad basing and establishing linkages manifold in several areas and help in knowledge sharing, capacity building, networking, advocacy, finance and resources support, research and technical support, education, and awareness. It has a General Assembly consisting of all member countries. A Council of at least seven but not more than 15 member countries elected by the General Assembly for a term of 5 years, and a Secretariat. Upon the recommendation of the Council, the General Assembly will appoint the IBCA Secretary General for a specific term. It has secured the Government of India’s initial support of Rs. 150 crore for five years (2023-24 to 2027-28). Five of the seven cats recognized under the International Big Cat Alliance are found in India. Apart from jaguar and puma, are found in India. Hence statement 2 is incorrect. Question 30 of 35 30. Question Consider the following statements regarding the BioTRIG: It is a waste management technology that works by sealing the waste inside an oxygen-free chamber and cooling it below 4000 degrees Celsius. It could help reduce greenhouse gas emissions from communities and could help rural Indians cut indoor air pollution. Choose the correct code: a) 1 only b) 2 only c) Both 1 and 2 d) Neither 1 nor 2 Correct Solution (b) The BioTRIG is a waste management technology that works by sealing the waste inside an oxygen-free chamber and heating it above 400 degrees Celsius based on the pyrolysis system. Hence statement 1 is incorrect. The syngas and bio-oil facilitate heat and power the pyrolysis system in future cycles and surplus electricity is utilized to power local homes and businesses. The clean-burning bio-oil replaces dirty cooking fuels in homes and uses biochar to store carbon while improving soil fertility. It could help reduce greenhouse gas emissions from communities and could help rural Indians cut indoor air pollution. Hence statement 2 is correct.   Incorrect Solution (b) The BioTRIG is a waste management technology that works by sealing the waste inside an oxygen-free chamber and heating it above 400 degrees Celsius based on the pyrolysis system. Hence statement 1 is incorrect. The syngas and bio-oil facilitate heat and power the pyrolysis system in future cycles and surplus electricity is utilized to power local homes and businesses. The clean-burning bio-oil replaces dirty cooking fuels in homes and uses biochar to store carbon while improving soil fertility. It could help reduce greenhouse gas emissions from communities and could help rural Indians cut indoor air pollution. Hence statement 2 is correct.   Question 31 of 35 31. Question Present ages of Amulya and Chandana are in the ratio of 7: 10 respectively. Five years hence, the ratio of their ages will become 8: 11 respectively. If the Sum of the present age of Amulya and Chandana is ab. then, find what the unit digit of (a^b ) is? a) 2 b) 6 c) 5 d) 8 Correct Solution (d) Let the present ages of Amulya and Chandana be 7x and 10x years respectively. Then, 7x+5/10x+5 = 8/11 77x +55 = 80x + 40 3x = 15 x = 5   So, the present ages of Amulya and Chandana are (7 × 5)and (10 × 5) respectively. The sum of their ages = (35 +50) = 85 Hence, a = 8 and b = 5 Then, a^b = 8^5 We know unit digit of 8 repeated after 4 powers.  So, unit digit of 8^5 = unit digit of 8^1 => 8   Incorrect Solution (d) Let the present ages of Amulya and Chandana be 7x and 10x years respectively. Then, 7x+5/10x+5 = 8/11 77x +55 = 80x + 40 3x = 15 x = 5   So, the present ages of Amulya and Chandana are (7 × 5)and (10 × 5) respectively. The sum of their ages = (35 +50) = 85 Hence, a = 8 and b = 5 Then, a^b = 8^5 We know unit digit of 8 repeated after 4 powers.  So, unit digit of 8^5 = unit digit of 8^1 => 8   Question 32 of 35 32. Question What is the next number in the given series: 2, 3, 4, 6, 7, 9, 10, 12, ___? a) 15 b) 13 c) 14 d) 16 Correct Solution (a) First double the numbers to get 4, 6, 8, 12, 14, 18, 20, 24. Observe that each of them is 1 greater than a prime number. Hence, on subtracting 1 from each number, the series obtained is: 3, 5, 7, 11, 13, 17, 19, 23. Hence, the next prime is 29. This first becomes 30 and is then halved to get 15. Hence, option a. Incorrect Solution (a) First double the numbers to get 4, 6, 8, 12, 14, 18, 20, 24. Observe that each of them is 1 greater than a prime number. Hence, on subtracting 1 from each number, the series obtained is: 3, 5, 7, 11, 13, 17, 19, 23. Hence, the next prime is 29. This first becomes 30 and is then halved to get 15. Hence, option a. Question 33 of 35 33. Question A 4-digit number of the form aabb is a perfect square. What is the value of a – b? a) 3 b) 2 c) 4 d) 1 Correct Solution (a) A number of the form aabb has to be a multiple of 11. So, it is the square of either 11 or 22 or 33 or…so on up to 99. 882 = 7744. This is the only solution possible. Most of these trial and error questions need to be narrowed down a little bit before we can look for the solution. That narrowing down is critical. In this case, we should look for multiples of 11. The question is “A 4-digit number of the form aabb is a perfect square. What is the value of a – b?” Hence the answer is “3” Choice A is the correct answer. Incorrect Solution (a) A number of the form aabb has to be a multiple of 11. So, it is the square of either 11 or 22 or 33 or…so on up to 99. 882 = 7744. This is the only solution possible. Most of these trial and error questions need to be narrowed down a little bit before we can look for the solution. That narrowing down is critical. In this case, we should look for multiples of 11. The question is “A 4-digit number of the form aabb is a perfect square. What is the value of a – b?” Hence the answer is “3” Choice A is the correct answer. Question 34 of 35 34. Question In a family of 12 people, the current average age is the same as it was 18 months ago because an older member died and a new member got added to the family. What is the difference between the age of the new member and the member who died? a) 24 years b) 20 years c) 18 years d) 16 years Correct Solution (c) Let the average age of the 12-member family be x years. Had the member not changed, the average age now (i.e. after 18 months) would have been (x + 1.5) years. Hence, the total age should have been (12x + 18) instead of (12x) that it is right now. This reduction of 18 years is because of the difference in age of the person who died and the person who joined. ∴ Required age difference = 18 years Hence, option c. Incorrect Solution (c) Let the average age of the 12-member family be x years. Had the member not changed, the average age now (i.e. after 18 months) would have been (x + 1.5) years. Hence, the total age should have been (12x + 18) instead of (12x) that it is right now. This reduction of 18 years is because of the difference in age of the person who died and the person who joined. ∴ Required age difference = 18 years Hence, option c. Question 35 of 35 35. Question There are 9 men and 6 women and you need to form a committee of 3 men and 2 women. In how many ways can the committee be formed? a) 1100 b) 1260 c) 630 d) 2520 Correct Solution (b) We need to select 3 men from 9 men and 2 women from 6 women Number of ways to do this = 9C3 × 6C2 = (9×8×7/3×2×1) × (6×5/2×1) = 1,260 Incorrect Solution (b) We need to select 3 men from 9 men and 2 women from 6 women Number of ways to do this = 9C3 × 6C2 = (9×8×7/3×2×1) × (6×5/2×1) = 1,260 window.wpProQuizInitList = window.wpProQuizInitList || []; window.wpProQuizInitList.push({ id: '#wpProQuiz_3688', init: { quizId: 3688, mode: 1, globalPoints: 70, timelimit: 1800, resultsGrade: [0], bo: 704, qpp: 0, catPoints: [70], formPos: 0, lbn: "Test-summary", json: {"33174":{"type":"single","id":33174,"catId":0,"points":2,"correct":[0,0,0,1]},"33176":{"type":"single","id":33176,"catId":0,"points":2,"correct":[1,0,0,0]},"33177":{"type":"single","id":33177,"catId":0,"points":2,"correct":[0,0,1,0]},"33179":{"type":"single","id":33179,"catId":0,"points":2,"correct":[0,0,1,0]},"33180":{"type":"single","id":33180,"catId":0,"points":2,"correct":[0,1,0,0]},"33181":{"type":"single","id":33181,"catId":0,"points":2,"correct":[1,0,0,0]},"33183":{"type":"single","id":33183,"catId":0,"points":2,"correct":[0,0,1,0]},"33186":{"type":"single","id":33186,"catId":0,"points":2,"correct":[0,1,0,0]},"33187":{"type":"single","id":33187,"catId":0,"points":2,"correct":[0,1,0,0]},"33190":{"type":"single","id":33190,"catId":0,"points":2,"correct":[0,0,1,0]},"33191":{"type":"single","id":33191,"catId":0,"points":2,"correct":[0,1,0,0]},"33193":{"type":"single","id":33193,"catId":0,"points":2,"correct":[0,0,1,0]},"33194":{"type":"single","id":33194,"catId":0,"points":2,"correct":[0,0,1,0]},"33197":{"type":"single","id":33197,"catId":0,"points":2,"correct":[0,0,0,1]},"33200":{"type":"single","id":33200,"catId":0,"points":2,"correct":[1,0,0,0]},"33201":{"type":"single","id":33201,"catId":0,"points":2,"correct":[0,1,0,0]},"33204":{"type":"single","id":33204,"catId":0,"points":2,"correct":[0,0,1,0]},"33207":{"type":"single","id":33207,"catId":0,"points":2,"correct":[0,1,0,0]},"33210":{"type":"single","id":33210,"catId":0,"points":2,"correct":[0,0,1,0]},"33212":{"type":"single","id":33212,"catId":0,"points":2,"correct":[0,0,1,0]},"33214":{"type":"single","id":33214,"catId":0,"points":2,"correct":[0,1,0,0]},"33217":{"type":"single","id":33217,"catId":0,"points":2,"correct":[0,0,1,0]},"33218":{"type":"single","id":33218,"catId":0,"points":2,"correct":[0,1,0,0]},"33221":{"type":"single","id":33221,"catId":0,"points":2,"correct":[0,0,0,1]},"33222":{"type":"single","id":33222,"catId":0,"points":2,"correct":[0,0,0,1]},"33224":{"type":"single","id":33224,"catId":0,"points":2,"correct":[0,1,0,0]},"33227":{"type":"single","id":33227,"catId":0,"points":2,"correct":[0,1,0,0]},"33228":{"type":"single","id":33228,"catId":0,"points":2,"correct":[0,0,1,0]},"33229":{"type":"single","id":33229,"catId":0,"points":2,"correct":[1,0,0,0]},"33230":{"type":"single","id":33230,"catId":0,"points":2,"correct":[0,1,0,0]},"33232":{"type":"single","id":33232,"catId":0,"points":2,"correct":[0,0,0,1]},"33235":{"type":"single","id":33235,"catId":0,"points":2,"correct":[1,0,0,0]},"33237":{"type":"single","id":33237,"catId":0,"points":2,"correct":[1,0,0,0]},"33239":{"type":"single","id":33239,"catId":0,"points":2,"correct":[0,0,1,0]},"33240":{"type":"single","id":33240,"catId":0,"points":2,"correct":[0,1,0,0]}} } }); All the Best IASbaba

DAILY CURRENT AFFAIRS IAS | UPSC Prelims and Mains Exam – 27th April 2024

Archives (PRELIMS & MAINS Focus)   Inheritance Tax Syllabus Prelims & Mains – Current Event , Mobilisation of resources Context: The chairman of Indian Overseas Congress, Sam Pitroda’s comments on the inheritance tax, have raised the spectre of wealth redistribution, almost four decades after the very same Congress party under Prime Minister Rajiv Gandhi had abolished it. Background:- India did have an inheritance (or death) tax once. The tax, which was known as estate duty, was introduced in 1953, and was abolished in 1985 by the government of Rajiv Gandhi. India also had a wealth tax and a gift tax, which were abolished in 2015 and 1998 respectively. Ways to tax wealth Taxes are levied on the flow of income on wealth or at the time of transfer of wealth or on the stock of wealth linked to the value of owned assets as a one-time levy. There can be a capital levy on income from wealth or ownership of assets resulting in capital gains, transfer taxes in the form of wealth tax, inheritance tax, estate tax, or gift tax at the time of transfer of wealth or assets. Taxes can also be levied on a combination of income and wealth.Biden Administration (USA) has proposed legislation for a ‘Billionaire Minimum Income Tax’ of at least 25% on their full income, including unrealised gains. The taxes in India The now abolished estate duty was inheritance tax with a threshold of Rs 1 lakh, and progressive rates from 5% to 40% on the principal value of the estate exceeding Rs 20 lakh. The Estate Duty Act, 1953 was amended in 1958 to change the definition of accountable person, lower the applicable threshold, and redefine slabs. Even after its abolition, the idea of inheritance tax remained alive and was part of official and unofficial discussions. In December 2018, then Finance Minister Arun Jaitley said at a public event that hospitals, universities, and other institutions in developed countries receive large endowments due to factors like inheritance tax, but this practice is not prevalent in India. The Narendra Modi government announced the abolition of wealth tax and its replacement with a surcharge on the super rich in the Budget for 2015-16. Jaitley said while wealth tax was 1% on assets of Rs 30 lakh and above (excluding equities, bonds, and first house), the total wealth collection in 2013-14 was only Rs 1,008 crore. An additional surcharge of 2% was levied on individuals with taxable income of Rs 1 crore and above, which was expected to bring revenues of Rs 9,000 crore. Wealthy taxpayers are seen as being sensitive to high rates of taxation, which often results in flight of capital and investment to tax havens or tax jurisdictions with favourable tax rates. Also, high tax rates do not make sense if the cost of collection and administration of these taxes are high compared to the revenues that arise. In the initial years of estate duty, collections were lower than the budgeted estimates. The high cost of collection and double taxation in various forms of wealth tax were cited as reasons for abolishing estate duty. In his Budget speech of 1985-86, then Finance Minister V P Singh said the existence of two separate laws for tax on property — wealth tax before death and estate duty after death — amounted to “procedural harassment” of taxpayers. “While the yield from estate duty is only about Rs 20 crore, its cost of administration is relatively high. I, therefore, propose to abolish the levy of estate duty in respect of estates passing on deaths occurring on or after 16th March, 1985,” Singh said. The issue with wealth tax was the same. “Should a tax which leads to high cost of collection and a low yield be continued or should it be replaced with a low cost and higher yield tax? The rich and wealthy must pay more tax than the less affluent ones. I have therefore decided to abolish the wealth tax and replace it with an additional surcharge of 2% on the super rich with a taxable income of over Rs 1 crore,” Arun Jaitley said in his Budget speech for 2015-16. The gift tax was abolished in 1998 citing lower revenues. Gift tax was, however, revived in a different form later. In his Budget speech in July 2004, then Finance Minister P Chidambaram said there was a need to plug a loophole to prevent money laundering. Thus, purported gifts from unrelated persons above the threshold of Rs 25,000 (later raised to Rs 50,000), were decided to be taxed as income. Gifts from blood relations, lineal ascendants and descendants, and gifts on occasions like marriage continued to be exempt. Source: Indian Express Womens Workforce Participation Syllabus Prelims & Mains – Rights Context: Participation of women in the workforce is a constitutional entitlement and denying mothers child care leave violates this, the Supreme Court said Monday. Background: A bench of Chief Justice of India D Y Chandrachud and J B Pardiwala was hearing a plea by an assistant professor in the Government College, Nalagarh, who was denied childcare leave (CCL) to attend to her child suffering from a genetic condition. Key Takeaways: Women government employees are eligible for two years of childcare leave (CCL) for two children till they reach the age of 18 years. The judgment highlighted that Article 15 of the Constitution not only restricts discrimination based on gender but also enables the states to make special provisions for women. In a country where care is perceived as the responsibility of women, the verdict makes it clear that the state and the employer have responsibilities to enable women to join and retain their employment. In India, women have little choice but to single-handedly manage the triple burden of housework, care work and paid work. Working women often face “marriage penalties” and “motherhood penalties” as they are often temporarily forced to withdraw from the workforce due to marriage and pregnancy. It is no surprise that the female workforce participation in India is barely 37 per cent. The Constitution enables the state to make special provisions for women and children. The sectoral labour laws that were repealed recently after the introduction of four Labour Codes mandated childcare services on worksites and paid maternity leaves for a section of workers at construction sites, beedi, cigar and other factories, plantations and migrants. These laws mandated crèches on worksites that had a stipulated number of women on the work site. The provision went through a transformation under the Labour Code on Social Protection, 2020, where crèches were made a gender-neutral entitlement. It is a significant step in the right direction. The gender-neutral provision underlined care as a “parental” responsibility. Care needs to be seen as a collective responsibility of the state, employers, and communities. Labour markets need to consider women as primary earners and enable them to take up full employment. There is evidence of high female labour force participation in countries where unpaid care work responsibilities are equally shared. Additional information As per PLFS 2022, 60 per cent women are self-employed and 53 per cent of the self-employed women work as unpaid family helpers. These are intertwined outcomes of lack of opportunities in the labour market and opting for flexible employment near or at home to balance both. A decrease in women’s unpaid care work is related to a 10-percentage point increase in women’s labour force participation rate. The IMF has predicted a possible alleviation of 27 per cent in India’s GDP through an equal participation of women in the labour force participation. Source: Indian Express GANDHI SAGAR WILDLIFE SANCTUARY Syllabus Prelims – Environment Context: South African delegation recently visited the Gandhi Sagar Wildlife Sanctuary in Madhya Pradesh to assess its readiness for the release of five to eight cheetahs later this year. Background: This visit is part of the planning for Project Cheetah, which aims to reintroduce cheetahs into the sanctuary. The focus is on breeding and favourable weather conditions.   About Gandhi Sagar Wildlife Sanctuary The Gandhi Sagar Wildlife Sanctuary is situated on the northern boundary of Mandsaur and Nimach districts in Madhya Pradesh. It shares its border with the state of Rajasthan. Much of the sanctuary consists of vast open landscapes with sparse vegetation and rocky terrain, interspersed with small patches of dense forests. The Chambal River flows through the sanctuary, dividing it into two parts. The forest of this sanctuary is part of Khathiar-Gir dry deciduous forest thus here we will find trees like Salai, Kardhai, Dhawda, Tendu, Palash etc. The sanctuary is home to various wildlife species.Some of the easily sighted animals include chinkara (Indian gazelle), Nilgai, and sambar. Additionally, the Indian leopard, langur, Indian wild dog, peacock, otter, and Mugger crocodile inhabit the region. World famous Chaturbhuj Nala rock shelters are also part of the same Gandhi Sagar wildlife sanctuary. Gandhi Sagar Wildlife Sanctuary in Madhya Pradesh, India, is being prepared as a second home for cheetahs as part of the Cheetah Reintroduction Project. The Kuno National Park in Madhya Pradesh has been chosen as the first home for the reintroduction of cheetahs. Source: Times of India MAJULI ISLAND Syllabus Prelims – Geography & Environment Context: The drying of the wetlands in Assam’s Majuli island is having a profound impact on both the local ecology and the livelihoods of its residents. Background: Wetlands play a crucial role in maintaining ecological balance, supporting biodiversity, and preserving the livelihoods of local communities. About Majuli Island: Majuli is a river island located on the Brahmaputra River in Assam, India. It is recognized as the world’s largest river island by the Guinness World Records, covering an area of 352 sq km. The island was formed due to frequent earthquakes and a catastrophic flood in 1750. Majuli is home to various tribal groups including Mising, Deoris, Ahoms, and Sonowal Kacharis. The island is known for the Ali Aye Ligang festival celebrated by the Mishing tribe and is an ancient site of Assamese neo-Vaishnavite monasteries, with around 22 Satras. Majuli faces serious ecological threats due to soil erosion and changing climatic conditions, which affect agriculture and livelihoods. Majuli has been nominated for the World Heritage Site status and is included in the tentative list by UNESCO. In 2016, Majuli became the first island to be made a district in India. Factors Contributing to the Drying Up of Majuli Island Embankments: The construction of embankments along the Brahmaputra River has disrupted the natural flow of water. These embankments prevent floodwaters from reaching the wetlands, leading to reduced water levels and drying up of the island’s beels (wetlands). Lack of Natural Floodwater Circulation: The embankments have also hindered the natural circulation of floodwaters within Majuli. As a result, stagnant water accumulates in the wetlands, affecting their health and vitality. Expansion of Agriculture and Infrastructure: Agricultural expansion and infrastructure development have encroached upon wetland areas. Land reclamation for agriculture and construction reduces the available space for wetlands, impacting their size and functionality. Erosion: Majuli faces severe erosion due to the Brahmaputra’s powerful currents. The erosion displaces soil and silt, which then choke the water bodies, further contributing to their drying up. Climate Change: The changing climate in the region exacerbates the situation. Unpredictable rainfall patterns and increased summer temperatures affect water availability, impacting the wetlands. Source: Scroll NEPHROTIC SYNDROME Syllabus Prelims – Science Context: Researchers from Kerala have reported a series of cases where the regular use of fairness creams has been linked to nephrotic syndrome. Background: These creams, often marketed for skin lightening or brightening, may contain harmful ingredients, including high levels of mercury. About NEPHROTIC SYNDROME : Nephrotic syndrome is a kidney disorder characterized by excessive protein excretion in the urine. Recently, a link has been found between fairness creams and kidney problems, specifically membranous nephropathy (MN), which damages kidney filters and causes protein leakage. Fairness Creams and Mercury: Some fairness creams contain high levels of mercury, which can be detrimental to the kidneys. Mercury is absorbed through the skin, leading to damage in kidney filters. The presence of mercury in these creams has been associated with the development of membranous nephropathy. Symptoms of Nephrotic Syndrome: Severe Swelling (Edema) especially around the eyes, ankles, and feet. Foamy urine due to excess protein. Weight gain caused by fluid retention. Fatigue and loss of appetite. Underlying Cause: Nephrotic syndrome usually occurs due to damage to the tiny blood vessels (glomeruli) in the kidneys. These glomeruli filter blood, separating essential substances from waste. Damage to glomeruli allows too much blood protein (mainly albumin) to seep into the urine, leading to nephrotic syndrome. Source: Hindu G-Securities (G-Secs) Syllabus Prelims & Mains – Economy Context: The Reserve Bank of India (RBI) has permitted the lending and borrowing of G-Securities (G-Secs) by issuing directions called RBI (Government Securities Lending) Directions, 2023. Background: This move comes even as India is set to be included in the globally tracked JP Morgan’s Government Bond Index-Emerging Markets (GBI-EM) index, starting June 28, 2024. G-Securities (G-Secs): It is a tradeable instrument issued by central or state governments. It acknowledges the government’s debt obligation. Such securities are short-term terms usually called Treasury bills (T- Bills) with maturities of less than one year (91 days, 182 days, or 364 days) or long-term called Government bonds or dated securities with maturity of one year or more (between 5 years and 40 years). In India, central government issues both T bills and bonds or dated securities while State Governments issue only bonds or dated securities, which are called State Development Loans (SDLs). G-Secs carry practically no risk of default and, hence, are called risk-free gilt-edged instruments. Other G-Sec includes Cash Management Bills (CMBs), introduced in 2010, a new short-term instrument to meet temporary cash flow mismatches of the government. CMBs have the generic character of T-bills but are issued for maturities of less than 91 days. G-Secs are issued through auctions conducted by RBI. Auctions are conducted on the electronic platform called the E-Kuber, the Core Banking Solution (CBS) platform of RBI. Concerns associated with G-Securities (G-Secs): A diversified investor base for fixed-income securities is important for ensuring high liquidity and stable demand in the market. However, currently, a large portion of G-Secs are held by captive investors such as banks, and insurance companies. RBI’s Negotiated Dealing System Order Matching (NDS-OM) platform was not able to boost retail participation as it resulted in an artificial segmentation of investors in different securities. Inflows of foreign funds via government bonds can lead to rupee appreciation. The G-sec market lacks liquidity due to the non-availability of buyers for the security in the secondary market. It can lead to distressed sales (selling at a lower price than its holding cost) causing loss to sellers. Major risks associated with holding G-Secs: Market risk arises out of adverse movement of prices of the securities due to changes in interest rates. This could lead to loss if securities are sold at adverse prices. Cash flows on a G-Sec include a coupon every half year and repayment of principal at maturity, which needs to be reinvested. However, it poses a risk for investors as they may not be able to reinvest due to a decrease in prevailing interest rates. Dated securities have a long-term maturity of 5-40 years, and thus are exposed to interest rate risk, reducing their relevance over longer tenure. Way Forward: Unifying the G-Sec and corporate bond markets would enable the seamless transmission of pricing information from G-Secs to corporate bonds. Having the same regulatory regime for trade, clearance, and settlement of corporate bonds and G-Secs will result in economies of scale and scope, leading to greater competition, efficiency, and liquidity in markets. To facilitate greater investor participation and achieve ease of doing business, G-Secs should be issued and traded through the stock exchange mechanism. The government should issue G-Secs in demat so that demat holders (currently, more than 120 million and expanding) can easily invest in G-Secs. G-Sec-based exchange-traded funds should also be developed to increase retail participation. Fiscal Responsibility and Budget Management (FRBM) legislation should highlight a fiscal path to investors highlighting the steps to reducing government debt in a transparent and accountable manner to boost investors’ confidence. Providing tax incentives in the form of no tax to be paid on interest income generated from the G-Sec can boost the demand for the G-sec in the market. Source: Hindu Businessline Practice MCQs Daily Practice MCQs Q1.) With reference to the Nephrotic syndrome, consider the following statements: Nephrotic syndrome is a liver disorder. It is characterized by excessive protein excretion in the urine. Regular use of fairness creams containing high levels of mercury is leading to nephrotic syndrome. How many of the statements given above are correct? Only one Only two Only three None Q2.) Consider the following: The construction of embankments along the Brahmaputra River. Lack of natural floodwater circulation. Agricultural expansion and infrastructure development. The changing climate in the region. How many of the above are the possible reasons for the recent drying of the wetlands in Assam’s Majuli island? Only one Only two Only three All four Q3.) With reference to the Gandhi Sagar Wildlife Sanctuary, consider the following statements: The Gandhi Sagar Wildlife Sanctuary is situated in Madhya Pradesh. The Chambal River flows through the sanctuary. Gandhi Sagar Wildlife Sanctuary is an ideal location for the reintroduction of cheetahs. How many of the statements given above are correct? Only one Only two All three None Comment the answers to the above questions in the comment section below!! ANSWERS FOR ’  27th April  2024 – Daily Practice MCQs’ will be updated along with tomorrow’s Daily Current Affairs.st ANSWERS FOR  25th April – Daily Practice MCQs Answers- Daily Practice MCQs Q.1) – b Q.2) – b Q.3) – c

[DAY 48] 60 DAY RAPID REVISION (RaRe) SERIES for UPSC Prelims 2024 – ENVIRONMENT, CURRENT AFFAIRS & CSAT TEST SERIES!

Archives Hello Friends The 60 Days Rapid Revision (RaRe) Series is IASbaba’s Flagship Initiative recommended by Toppers and loved by the aspirants’ community every year. It is the most comprehensive program which will help you complete the syllabus, revise and practice tests on a daily basis. The Programme on a daily basis includes Daily Prelims MCQs from Static (Monday – Saturday) Daily Static Quiz will cover all the topics of static subjects – Polity, History, Geography, Economics, Environment and Science and technology. 20 questions will be posted daily and these questions are framed from the topics mentioned in the schedule. It will ensure timely and streamlined revision of your static subjects. Daily Current Affairs MCQs (Monday – Saturday) Daily 5 Current Affairs questions, based on sources like ‘The Hindu’, ‘Indian Express’ and ‘PIB’, would be published from Monday to Saturday according to the schedule. Daily CSAT Quiz (Monday – Friday) CSAT has been an Achilles heel for many aspirants. Daily 5 CSAT Questions will be published. Note – Daily Test of 20 static questions, 10 current affairs, and 5 CSAT questions. (35 Prelims Questions) in QUIZ FORMAT will be updated on a daily basis. To Know More about 60 Days Rapid Revision (RaRe) Series – CLICK HERE   60 Day Rapid Revision (RaRe) Series Schedule – CLICK HERE  Important Note Comment your Scores in the Comment Section. This will keep you accountable, responsible and sincere in days to come. It will help us come out with the Cut-Off on a Daily Basis. Let us know if you enjoyed today’s test 🙂  You can post your comments in the given format  (1) Your Score (2) Matrix Meter (3) New Learning from the Test Time limit: 0 Test-summary 0 of 35 questions completed Questions: 1 2 3 4 5 6 7 8 9 10 11 12 13 14 15 16 17 18 19 20 21 22 23 24 25 26 27 28 29 30 31 32 33 34 35 Information The following Test is based on the syllabus of 60 Days Plan-2023 for UPSC IAS Prelims 2022. To view Solutions, follow these instructions: Click on – ‘Start Test’ button Solve Questions Click on ‘Test Summary’ button Click on ‘Finish Test’ button Now click on ‘View Questions’ button – here you will see solutions and links. You have already completed the test before. Hence you can not start it again. Test is loading... You must sign in or sign up to start the test. You have to finish following test, to start this test: Results 0 of 35 questions answered correctly Your time: Time has elapsed You have scored 0 points out of 0 points, (0) Average score     Your score     Categories Not categorized 0% Your result has been entered into leaderboard Loading Name: E-Mail: Captcha: maximum of 70 points Pos. Name Entered on Points Result Table is loading No data available 1 2 3 4 5 6 7 8 9 10 11 12 13 14 15 16 17 18 19 20 21 22 23 24 25 26 27 28 29 30 31 32 33 34 35 Answered Review Question 1 of 35 1. Question ‘Florianopolis Declaration’ is related to which of the following? a) Promoting commercial whaling as a necessary economic activity b) Allowing the indefinite protection of the world's whale population c) Encouraging pro-whaling states to increase their activities d) Providing binding regulations on commercial whaling activities Correct Solution (b) Florianopolis Declaration of 2018 reaffirms that the moratorium on commercial whaling, which has been in effect since 1986, has contributed to the recovery of some whale populations, and aware of cumulative effects of multiple, existing and emerging threats to whale populations such as entanglement, bycatch, underwater noise, ship strikes, marine debris and climate change. (Hence option b is correct) It Agrees that the role of the International Whaling Commission in the 21st Century includes inter alia its responsibility to ensure the recovery of whale populations to their pre-industrial levels, and in this context reaffirms the importance in maintaining the moratorium on commercial whaling. Incorrect Solution (b) Florianopolis Declaration of 2018 reaffirms that the moratorium on commercial whaling, which has been in effect since 1986, has contributed to the recovery of some whale populations, and aware of cumulative effects of multiple, existing and emerging threats to whale populations such as entanglement, bycatch, underwater noise, ship strikes, marine debris and climate change. (Hence option b is correct) It Agrees that the role of the International Whaling Commission in the 21st Century includes inter alia its responsibility to ensure the recovery of whale populations to their pre-industrial levels, and in this context reaffirms the importance in maintaining the moratorium on commercial whaling. Question 2 of 35 2. Question Eco bridges and Eco ducts were in news recently, consider following statements: It aims to enhance wildlife connectivity and create contiguous look to landscape. Uttarakhand has Eco bridge for reptiles and small animals. Tiger, leopard used Eco bridge in Pench tiger reserve. How many of the above statements are correct? a) Only one b) Only two c) All three d) None Correct Solution (c) Eco-ducts or eco-bridges aim to enhance wildlife connectivity that can be disrupted because of highways or logging. These include canopy bridges (usually for monkeys, squirrels and other arboreal species); concrete underpasses or overpass tunnels or viaducts (usually for larger animals); and amphibian tunnels or culverts. (Hence statement 1 is correct) Usually these bridges are overlaid with planting from the area to give it a contiguous look with the landscape. Ramnagar Forest Division in Nainital district, Uttarakhand, recently built its first eco-bridge for reptiles and smaller mammals. (Hence statement 2 is correct) A 2020 study by the Wildlife Institute of India (WII) noted that nearly 50,000 km of road projects have been identified for construction in India over the next five to six years, while many highways are being upgraded to four lanes. The National Tiger Conservation Authority, New Delhi, had identified three major sites that were cutting across animal corridors, including National Highway 37 through the Kaziranga-Karbi Anglong landscape in Assam, and State Highway 33 through the Nagarhole Tiger Reserve in Karnataka. Cameras captured nearly 18 species that used these underpasses, including tiger, leopard, and golden jackal. (Hence statement 3 is correct) Incorrect Solution (c) Eco-ducts or eco-bridges aim to enhance wildlife connectivity that can be disrupted because of highways or logging. These include canopy bridges (usually for monkeys, squirrels and other arboreal species); concrete underpasses or overpass tunnels or viaducts (usually for larger animals); and amphibian tunnels or culverts. (Hence statement 1 is correct) Usually these bridges are overlaid with planting from the area to give it a contiguous look with the landscape. Ramnagar Forest Division in Nainital district, Uttarakhand, recently built its first eco-bridge for reptiles and smaller mammals. (Hence statement 2 is correct) A 2020 study by the Wildlife Institute of India (WII) noted that nearly 50,000 km of road projects have been identified for construction in India over the next five to six years, while many highways are being upgraded to four lanes. The National Tiger Conservation Authority, New Delhi, had identified three major sites that were cutting across animal corridors, including National Highway 37 through the Kaziranga-Karbi Anglong landscape in Assam, and State Highway 33 through the Nagarhole Tiger Reserve in Karnataka. Cameras captured nearly 18 species that used these underpasses, including tiger, leopard, and golden jackal. (Hence statement 3 is correct) Question 3 of 35 3. Question Consider the following statements about IPCC’s AR6 Synthesis Report: Over 3 billion people in highly vulnerable areas being 15 times more likely to die from climate-related events compared to least vulnerable. Existing adaptation gaps that will continue to grow due to limited resources, low climate literacy, and a lack of political commitment. It stresses need for climate-resilient development, reducing fossil fuel use, climate investments and low-carbon lifestyles. How many of the above statements are correct? a) Only one b) Only two c) All three d) None Correct Solution (c) Vulnerable communities, historically less responsible for climate change, are disproportionately affected. It states that over three billion people in highly vulnerable areas are 15 times more likely to die from climate-related events. (Hence statement 1 is correct.) Despite some progress in curtailing greenhouse gas emissions, there are existing adaptation gaps. These gaps are attributed to factors such as limited resources, low climate literacy, and a lack of political commitment. (Hence statement 2 is correct.) It stresses need climate-resilient development, reducing fossil fuel use, increasing finance for climate investments, and facilitating low-carbon lifestyles. (Hence statement 3 is correct.) Incorrect Solution (c) Vulnerable communities, historically less responsible for climate change, are disproportionately affected. It states that over three billion people in highly vulnerable areas are 15 times more likely to die from climate-related events. (Hence statement 1 is correct.) Despite some progress in curtailing greenhouse gas emissions, there are existing adaptation gaps. These gaps are attributed to factors such as limited resources, low climate literacy, and a lack of political commitment. (Hence statement 2 is correct.) It stresses need climate-resilient development, reducing fossil fuel use, increasing finance for climate investments, and facilitating low-carbon lifestyles. (Hence statement 3 is correct.) Question 4 of 35 4. Question Consider the following statements about ‘Ecomark Scheme’: It provides accreditation and labelling for household and consumer products. It will motivate manufacturers to shift towards environmentally friendly production. The Central Pollution Control Board administers the scheme in partnership with Bureau of Indian Standards (BIS). How many of the above statements are correct? a) Only one b) Only two c) All three d) None Correct Solution (c) The Ecomark Scheme provides accreditation and labelling for household and consumer products that meet specific environmental criteria while maintaining quality standards as per Indian norms. (Hence statement 1 is correct) Products accredited under the Ecomark Scheme will adhere to specific environmental criteria, ensuring minimal environmental impact. It will build consumer awareness of environmental issues and encourage eco-conscious choices. It will also motivate manufacturers to shift towards environmentally friendly production. The scheme seeks to ensure accurate labelling and prevent misleading information about products. (Hence statement 2 is correct) The Central Pollution Control Board administers the Ecomark Scheme in partnership with Bureau of Indian Standards (BIS), which is the national body for standards and certification. (Hence statement 3 is correct) Incorrect Solution (c) The Ecomark Scheme provides accreditation and labelling for household and consumer products that meet specific environmental criteria while maintaining quality standards as per Indian norms. (Hence statement 1 is correct) Products accredited under the Ecomark Scheme will adhere to specific environmental criteria, ensuring minimal environmental impact. It will build consumer awareness of environmental issues and encourage eco-conscious choices. It will also motivate manufacturers to shift towards environmentally friendly production. The scheme seeks to ensure accurate labelling and prevent misleading information about products. (Hence statement 2 is correct) The Central Pollution Control Board administers the Ecomark Scheme in partnership with Bureau of Indian Standards (BIS), which is the national body for standards and certification. (Hence statement 3 is correct) Question 5 of 35 5. Question Consider the following statements in context of Indian Forest Act 1927: All activities are prohibited unless permitted in Protected Forests All activities are permitted unless it is prohibited in Reserve Forests State government can assign to any village community the rights of government over any land constituted as reserved forest. How many of the above statements are correct? a) Only one b) Only two c) All three d) None Correct Solution (a) State government may constitute any forest land or waste land as reserved forest which is the property of Government & may sell the produce from these forest. Before issuing official notification state government needs to appoint a Forest Settlement Officer (has power of Civil Courts) to inquire & to provide settlements to persons belonging to that area. All activities are prohibited unless permitted in reserve forests. (Hence statement 1 is incorrect) Any land not included in Reserve Forest is considered as Protected Forest. Protected forests are of 2 kinds: Demarcated protected forests and Undemarcated protected forests In protected forests, all activities are permitted unless it is prohibited. (Hence statement 2 is incorrect) State government can assign to any village community the rights of government (to or over) any land which has been constituted as reserved forest. (Hence statement 3 is correct) State government may make rules for regulating the management of village forest, prescribing conditions under which the community may be provided with timber or other forest produce or pasture. Incorrect Solution (a) State government may constitute any forest land or waste land as reserved forest which is the property of Government & may sell the produce from these forest. Before issuing official notification state government needs to appoint a Forest Settlement Officer (has power of Civil Courts) to inquire & to provide settlements to persons belonging to that area. All activities are prohibited unless permitted in reserve forests. (Hence statement 1 is incorrect) Any land not included in Reserve Forest is considered as Protected Forest. Protected forests are of 2 kinds: Demarcated protected forests and Undemarcated protected forests In protected forests, all activities are permitted unless it is prohibited. (Hence statement 2 is incorrect) State government can assign to any village community the rights of government (to or over) any land which has been constituted as reserved forest. (Hence statement 3 is correct) State government may make rules for regulating the management of village forest, prescribing conditions under which the community may be provided with timber or other forest produce or pasture. Question 6 of 35 6. Question Consider the following statements in context of “Assessment Report on Invasive Alien Species and their Control’’: It is released by IPCC-Intergovernmental Panel on Climate Change. Not all alien species establish and spread with negative impacts on biodiversity. Highest percentage of alien invertebrates are known to be invasive as per report amongst alien plants, alien microbes, alien vertebrates. Many invasive alien species have been intentionally introduced for their perceived benefits. How many of the above statements are correct? a) Only one b) Only two c) Only three d) All four Correct Solution (b) In the most extensive study on invasive species carried out till date, the Intergovernmental Platform on Biodiversity and Ecosystem Services (IPBES) in its new publication – the “Assessment Report on Invasive Alien Species and their Control’’ – has found that there are 37,000 alien species, including plants and animals, that have been introduced by many human activities to regions and biomes around the world, including more than 3,500 invasive alien species and that invasive alien species have played a key role in 60% of global plant and animal extinctions recorded.(Hence statement 1 is incorrect) Not all alien species establish and spread with negative impacts on biodiversity, local ecosystems and species, but a significant proportion do – then becoming known as invasive alien species. (Hence statement 2 is correct) About 6% of alien plants; 22% of alien invertebrates; 14% of alien vertebrates; and 11% of alien microbes are known to be invasive, posing major risks to nature and to people. (Hence statement 3 is incorrect) Many invasive alien species have been intentionally introduced for their perceived benefits, without consideration or knowledge of their negative impacts in forestry, agriculture, horticulture, aquaculture, or as pets. (Hence statement 4 is correct) Incorrect Solution (b) In the most extensive study on invasive species carried out till date, the Intergovernmental Platform on Biodiversity and Ecosystem Services (IPBES) in its new publication – the “Assessment Report on Invasive Alien Species and their Control’’ – has found that there are 37,000 alien species, including plants and animals, that have been introduced by many human activities to regions and biomes around the world, including more than 3,500 invasive alien species and that invasive alien species have played a key role in 60% of global plant and animal extinctions recorded.(Hence statement 1 is incorrect) Not all alien species establish and spread with negative impacts on biodiversity, local ecosystems and species, but a significant proportion do – then becoming known as invasive alien species. (Hence statement 2 is correct) About 6% of alien plants; 22% of alien invertebrates; 14% of alien vertebrates; and 11% of alien microbes are known to be invasive, posing major risks to nature and to people. (Hence statement 3 is incorrect) Many invasive alien species have been intentionally introduced for their perceived benefits, without consideration or knowledge of their negative impacts in forestry, agriculture, horticulture, aquaculture, or as pets. (Hence statement 4 is correct) Question 7 of 35 7. Question Consider the following statements about illegal wildlife trade of ‘Sturgeon fish’: Sturgeon a fish native to Bulgaria, Romania, Serbia and Ukraine, produces caviar. Volga is the last river body with functional populations of beluga, Russian, stellate and sterlet sturgeons. The Convention on International Trade in Endangered Species of Wild Fauna and Flora (CITES) in listed the species as endangered. IUCN imposed an international labelling system for all caviar products to curb illegal trade. How many of the above statements are correct? a) Only one b) Only two c) Only three d) All four Correct Solution (b) Sturgeon, a fish native to countries such as Bulgaria, Romania, Serbia and Ukraine that produces caviar, has been fished illegally, the researchers found. (Hence statement 1 is correct) Danube is the last river body with functional populations of beluga (Huso huso), Russian (Acipenser gueldenstaedtii), stellate (Acipenser stellatus) and sterlet (Acipenser ruthenus) sturgeons, according to the authors of the study. (Hence statement 2 is incorrect) The Convention on International Trade in Endangered Species of Wild Fauna and Flora (CITES) in 1998 listed the species as endangered and put restrictions on fishing of sturgeons from the Danube and Black Sea. (Hence statement 3 is correct) In 2000, CITES even imposed an international labelling system for all caviar products to curb illegal trade. (Hence statement 4 is incorrect) Incorrect Solution (b) Sturgeon, a fish native to countries such as Bulgaria, Romania, Serbia and Ukraine that produces caviar, has been fished illegally, the researchers found. (Hence statement 1 is correct) Danube is the last river body with functional populations of beluga (Huso huso), Russian (Acipenser gueldenstaedtii), stellate (Acipenser stellatus) and sterlet (Acipenser ruthenus) sturgeons, according to the authors of the study. (Hence statement 2 is incorrect) The Convention on International Trade in Endangered Species of Wild Fauna and Flora (CITES) in 1998 listed the species as endangered and put restrictions on fishing of sturgeons from the Danube and Black Sea. (Hence statement 3 is correct) In 2000, CITES even imposed an international labelling system for all caviar products to curb illegal trade. (Hence statement 4 is incorrect) Question 8 of 35 8. Question Consider the following statements about ‘Hope Spot Network’: It is a joint initiative of Conservation International and IUCN. A hope spot is an area of high species richness at least 1500 species of vascular plants and high degree of species endemism in terrestrial region. Hope spots can only be Marine Protected Areas (MPAs). How many of the above statements are correct? a) Only one b) Only two c) All three d) None Correct Solution (d) Hope Spot Network is a joint initiative of Mission Blue & IUCN. (Hence statement 1 is incorrect) A hope spot is an area of an ocean that needs special protection because of its wildlife & significant underwater habitats. They are chosen for their contributions to biodiversity, the carbon sink, and important habitat. (Hence statement 2 is incorrect) They can be Marine Protected Area’s (MPA – defined by IUCN) that need attention or any new sites. (Hence statement 3 is incorrect) Incorrect Solution (d) Hope Spot Network is a joint initiative of Mission Blue & IUCN. (Hence statement 1 is incorrect) A hope spot is an area of an ocean that needs special protection because of its wildlife & significant underwater habitats. They are chosen for their contributions to biodiversity, the carbon sink, and important habitat. (Hence statement 2 is incorrect) They can be Marine Protected Area’s (MPA – defined by IUCN) that need attention or any new sites. (Hence statement 3 is incorrect) Question 9 of 35 9. Question Consider the following statements about ‘Composite Index of Agricultural Sustainability (CIAS)’: NITI Aayog has prepared a Composite Index of Agricultural Sustainability (CIAS). It shows that Indian agriculture practices are moderately sustainable. At present, most sustainable agriculture is practiced in Mizoram. How many of the above statements are correct? a) Only one b) Only two c) All three d) None Correct Solution (b) Indian Council for Agricultural Research have prepared a Composite Index of Agricultural Sustainability (CIAS). (Hence statement 1 is incorrect) Index shows that Indian agriculture practices are moderately sustainable. (Hence statement 2 is correct) At present, the most sustainable agriculture is practised in Mizoram, Kerala, Andhra Pradesh, Madhya Pradesh and West Bengal while agriculture in arid Rajasthan is the least sustainable. (Hence statement 3 is correct) Incorrect Solution (b) Indian Council for Agricultural Research have prepared a Composite Index of Agricultural Sustainability (CIAS). (Hence statement 1 is incorrect) Index shows that Indian agriculture practices are moderately sustainable. (Hence statement 2 is correct) At present, the most sustainable agriculture is practised in Mizoram, Kerala, Andhra Pradesh, Madhya Pradesh and West Bengal while agriculture in arid Rajasthan is the least sustainable. (Hence statement 3 is correct) Question 10 of 35 10. Question Consider the following statements in context of ‘Global Green Growth Institute (GGGI)’: The Global Green Growth Institute (GGGI) is a treaty-based international, inter-governmental organization. It is headquartered in Seoul, South Korea. It is dedicated to promoting green growth with its Green Growth Index. Global Green Growth Institute and Energy Efficiency Services Ltd (EESL) will collaborate to implement the Asia Low Carbon Buildings Transition (ALCBT) project. How many of the above statements are correct? a) Only one b) Only two c) Only three d) All four Correct Solution (d) The Global Green Growth Institute (GGGI) is a treaty-based international, inter-governmental organization dedicated to supporting and promoting strong, inclusive and sustainable economic growth in developing countries and emerging economies. (Hence statement 1 is correct) It is based in Seoul, South Korea. (Hence statement 2 is correct) GGGI aims for global transition toward a model of green growth While this will be differentiated at the country level, at its core will be strategies that simultaneously achieve poverty reduction, social inclusion, environmental sustainability, and economic growth. Ensuring the world’s progress towards a green transformation requires comprehensive measurement and tracking systems. With this report, GGGI is presenting a new platform for measuring and tracking the green growth performance of countries worldwide through the Green Growth Index. (Hence statement 3 is correct) Energy Efficiency Services Limited (EESL) in collaboration with the Global Green Growth Institute (GGGI), an international intergovernmental organization, announced the launch of the Asia Low Carbon Buildings Transition (ALCBT) Project in India. (Hence statement 4 is correct) Incorrect Solution (d) The Global Green Growth Institute (GGGI) is a treaty-based international, inter-governmental organization dedicated to supporting and promoting strong, inclusive and sustainable economic growth in developing countries and emerging economies. (Hence statement 1 is correct) It is based in Seoul, South Korea. (Hence statement 2 is correct) GGGI aims for global transition toward a model of green growth While this will be differentiated at the country level, at its core will be strategies that simultaneously achieve poverty reduction, social inclusion, environmental sustainability, and economic growth. Ensuring the world’s progress towards a green transformation requires comprehensive measurement and tracking systems. With this report, GGGI is presenting a new platform for measuring and tracking the green growth performance of countries worldwide through the Green Growth Index. (Hence statement 3 is correct) Energy Efficiency Services Limited (EESL) in collaboration with the Global Green Growth Institute (GGGI), an international intergovernmental organization, announced the launch of the Asia Low Carbon Buildings Transition (ALCBT) Project in India. (Hence statement 4 is correct) Question 11 of 35 11. Question Consider the following statements related to the United Nations Convention on the Law of the Seas (UNCLOS): UNCLOS exclusively governs the management of maritime territories and exclusive economic zones of individual countries. The international ocean floor is not subject to UNCLOS unless a country has signed or ratified the convention. The “common heritage of mankind” principle applies to the seabed and its mineral resources as per UNCLOS. How many of the above statements are correct? a) Only one b) Only two c) All three d) None Correct Solution (a) Countries manage their own maritime territory and exclusive economic zones, while the high seas and the international ocean floor are governed by the United Nations Convention on the Law of the Seas. (Hence, statement 1 is incorrect) It is considered to apply to states regardless of whether or not they have signed or ratified it. (Hence, statement 2 is incorrect) Under the treaty, the seabed and its mineral resources are considered the “common heritage of mankind” that must be managed in a way that protects the interests of humanity through the sharing of economic benefits, support for marine scientific research, and protecting marine environments. (Hence, statement 3 is correct) Incorrect Solution (a) Countries manage their own maritime territory and exclusive economic zones, while the high seas and the international ocean floor are governed by the United Nations Convention on the Law of the Seas. (Hence, statement 1 is incorrect) It is considered to apply to states regardless of whether or not they have signed or ratified it. (Hence, statement 2 is incorrect) Under the treaty, the seabed and its mineral resources are considered the “common heritage of mankind” that must be managed in a way that protects the interests of humanity through the sharing of economic benefits, support for marine scientific research, and protecting marine environments. (Hence, statement 3 is correct) Question 12 of 35 12. Question With reference to The Wildlife Trade Monitoring Network (TRAFFIC), consider the following statements: Its mission is to ensure that trade in wild plants and animals is not a threat to the conservation of nature. It is a collaborative effort of Coalition Against Wildlife Trafficking (CAWT) and World Wildlife Fund (WWF). Choose the correct code a) 1 only b) 2 only c) Both 1 and 2 d) Neither 1 nor 2 Correct Solution (a) Statement 1 Statement 2 Correct Incorrect TRAFFIC aims to ensure that trade in wild plants and animals is not a threat to the conservation of nature. It is a leading non-governmental organisation working globally on trade in wild animals and plants in the context of both biodiversity conservation and sustainable development. TRAFFIC is a joint conservation programme of WWF and IUCN. It was established in 1976 by the Species Survival Commission of IUCN, principally as a response to the entry into force during the previous year of the Convention on International Trade in Endangered Species of Wild Fauna and Flora (CITES). Note: The Wildlife Trade Monitoring Network (TRAFFIC): The TRAFFIC, the Wildlife Trade Monitoring Network, is a leading non-governmental organisation working on wildlife trade in the context of both biodiversity conservation and sustainable development. It is a joint program of World Wildlife Fund (WWF) and the International Union for Conservation of Nature (IUCN). It aims to ensure that trade in wild plants and animals is not a threat to the conservation of nature. It was established in 1976 and has developed into a global network, research-driven and action-oriented, committed to delivering innovative and practical conservation solutions. Headquarters: Cambridge, United Kingdom Illegal wildlife trade is one of the main reasons that many species are endangered. The TRAFFIC is governed by the TRAFFIC Committee, a steering group composed of members of TRAFFIC’s partner organizations, WWF and IUCN. TRAFFIC also works in close co-operation with the Secretariat of the Convention on International Trade in Endangered Species of Wild Fauna and Flora (CITES). Its staff includes experts from a plethora of backgrounds: biologists, conservationists, academics, researchers, communicators or investigators, etc. Since its establishment, it has helped in the evolution of the international wildlife trade treaties. It focuses on leveraging resources, expertise and awareness of the latest globally urgent species trade issues such as tiger parts, elephant ivory and rhino horn. Large scale commercial trade in commodities like timber and fisheries products are also addressed and linked to work on developing rapid results and policy improvements. Incorrect Solution (a) Statement 1 Statement 2 Correct Incorrect TRAFFIC aims to ensure that trade in wild plants and animals is not a threat to the conservation of nature. It is a leading non-governmental organisation working globally on trade in wild animals and plants in the context of both biodiversity conservation and sustainable development. TRAFFIC is a joint conservation programme of WWF and IUCN. It was established in 1976 by the Species Survival Commission of IUCN, principally as a response to the entry into force during the previous year of the Convention on International Trade in Endangered Species of Wild Fauna and Flora (CITES). Note: The Wildlife Trade Monitoring Network (TRAFFIC): The TRAFFIC, the Wildlife Trade Monitoring Network, is a leading non-governmental organisation working on wildlife trade in the context of both biodiversity conservation and sustainable development. It is a joint program of World Wildlife Fund (WWF) and the International Union for Conservation of Nature (IUCN). It aims to ensure that trade in wild plants and animals is not a threat to the conservation of nature. It was established in 1976 and has developed into a global network, research-driven and action-oriented, committed to delivering innovative and practical conservation solutions. Headquarters: Cambridge, United Kingdom Illegal wildlife trade is one of the main reasons that many species are endangered. The TRAFFIC is governed by the TRAFFIC Committee, a steering group composed of members of TRAFFIC’s partner organizations, WWF and IUCN. TRAFFIC also works in close co-operation with the Secretariat of the Convention on International Trade in Endangered Species of Wild Fauna and Flora (CITES). Its staff includes experts from a plethora of backgrounds: biologists, conservationists, academics, researchers, communicators or investigators, etc. Since its establishment, it has helped in the evolution of the international wildlife trade treaties. It focuses on leveraging resources, expertise and awareness of the latest globally urgent species trade issues such as tiger parts, elephant ivory and rhino horn. Large scale commercial trade in commodities like timber and fisheries products are also addressed and linked to work on developing rapid results and policy improvements. Question 13 of 35 13. Question With respect to the Kunming-Montreal Global Biodiversity Framework (GBF), consider the following statements: It aims to restore 50% degraded ecosystems globally by 2050. Conserve and manage 30% areas by 2050. Reduce risk from pesticides by at least 50% by 2030. Reduce nutrients lost to the environment by at least 50% by 2030. How many of the above statements are correct? a) Only one b) Only two c) Only three d) All four Correct Solution (b) Statement 1 Statement 2 Statement 3 Statement 4 Incorrect Incorrect Correct Correct Restore 30% degraded ecosystems globally (on land and sea) by 2030. Conserve and manage 30% areas (terrestrial, inland water, and coastal and marine) by 2030. Reduce risk from pesticides by at least 50% by 2030. Reduce nutrients lost to the environment by at least 50% by 2030. Note: At the 15th Conference of Parties (COP15) to the UN Convention on Biological Diversity ‘Kunming-Montreal Global Biodiversity Framework’ (GBF) was adopted. Key Targets of the GBF: 30×30 Deal: Restore 30% degraded ecosystems globally (on land and sea) by 2030. Conserve and manage 30% areas (terrestrial, inland water, and coastal and marine) by 2030. Stop the extinction of known species, and by 2050 reduce tenfold the extinction risk and rate of all species (including unknown). Reduce risk from pesticides by at least 50% by 2030. Reduce nutrients lost to the environment by at least 50% by 2030. Reduce pollution risks and negative impacts of pollution from all sources by 2030 to levels that are not harmful to biodiversity and ecosystem functions. Reduce global footprint of consumption by 2030, including through significantly reducing overconsumption and waste generation and halving food waste. Sustainably manage areas under agriculture, aquaculture, fisheries, and forestry and substantially increase agroecology and other biodiversity-friendly practices. Tackle climate change through nature-based solutions. Reduce the rate of introduction and establishment of invasive alien species by at least 50% by 2030. Secure the safe, legal and sustainable use and trade of wild species by 2030. Green up urban spaces. Incorrect Solution (b) Statement 1 Statement 2 Statement 3 Statement 4 Incorrect Incorrect Correct Correct Restore 30% degraded ecosystems globally (on land and sea) by 2030. Conserve and manage 30% areas (terrestrial, inland water, and coastal and marine) by 2030. Reduce risk from pesticides by at least 50% by 2030. Reduce nutrients lost to the environment by at least 50% by 2030. Note: At the 15th Conference of Parties (COP15) to the UN Convention on Biological Diversity ‘Kunming-Montreal Global Biodiversity Framework’ (GBF) was adopted. Key Targets of the GBF: 30×30 Deal: Restore 30% degraded ecosystems globally (on land and sea) by 2030. Conserve and manage 30% areas (terrestrial, inland water, and coastal and marine) by 2030. Stop the extinction of known species, and by 2050 reduce tenfold the extinction risk and rate of all species (including unknown). Reduce risk from pesticides by at least 50% by 2030. Reduce nutrients lost to the environment by at least 50% by 2030. Reduce pollution risks and negative impacts of pollution from all sources by 2030 to levels that are not harmful to biodiversity and ecosystem functions. Reduce global footprint of consumption by 2030, including through significantly reducing overconsumption and waste generation and halving food waste. Sustainably manage areas under agriculture, aquaculture, fisheries, and forestry and substantially increase agroecology and other biodiversity-friendly practices. Tackle climate change through nature-based solutions. Reduce the rate of introduction and establishment of invasive alien species by at least 50% by 2030. Secure the safe, legal and sustainable use and trade of wild species by 2030. Green up urban spaces. Question 14 of 35 14. Question Which of the following statements is/are correct regarding to the Green Climate Fund (GCF)? It was set up by the United Nations Framework Convention on Climate Change. It is being managed by the World Bank. It engages directly with the private sector through its Private Sector Facility. Select the correct answer using the code given below: a) 1 and 2 only b) 1 and 3 only c) 3 only d) 1, 2 and 3 Correct Solution (b) Statement 1 Statement 2 Statement 3 Correct Incorrect Correct The Green Climate Fund (GCF) was set up by the United Nations Framework Convention on Climate Change (UNFCCC) in 2010. It is not managed by the World Bank. The Green Climate fund has its own administrative set up with Headquarters at Songdo International Business District, Incheon, South Korea. It engages directly with the private sector through its Private Sector Facility. Note: Green Climate Fund (GCF): The GCF was set up in 2010 under the UNFCCC’s financial mechanism to channel funding from developed countries to developing countries to allow them to mitigate climate change and also adapt to disruptions arising from a changing climate. The Green Climate Fund will support projects, programmes, policies and other activities in developing country Parties using thematic funding windows. It is intended to be the centerpiece of efforts to raise Climate Finance of $100 billion a year by 2020. It engages directly with the private sector through its Private Sector Facility. The Fund will promote the paradigm shift towards low-emission and climate-resilient development pathways by providing support to developing countries to limit or reduce their greenhouse gas emissions and to adapt to the impacts of climate change, taking into account the needs of those developing countries particularly vulnerable to the adverse effects of climate change. The Fund will strive to maximize the impact of its funding for adaptation and mitigation, and seek a balance between the two, while promoting environmental, social, economic and development co-benefits and taking a gender-sensitive approach. The Fund is governed and supervised by a Board that will have full responsibility for funding decisions and that receives the guidance of the Conference of Parties (COP). The Fund is accountable to, and functions under the guidance of, the COP. The Green Climate fund has its own administrative set up with Headquarters at Songdo International Business District, Incheon, South Korea. Incorrect Solution (b) Statement 1 Statement 2 Statement 3 Correct Incorrect Correct The Green Climate Fund (GCF) was set up by the United Nations Framework Convention on Climate Change (UNFCCC) in 2010. It is not managed by the World Bank. The Green Climate fund has its own administrative set up with Headquarters at Songdo International Business District, Incheon, South Korea. It engages directly with the private sector through its Private Sector Facility. Note: Green Climate Fund (GCF): The GCF was set up in 2010 under the UNFCCC’s financial mechanism to channel funding from developed countries to developing countries to allow them to mitigate climate change and also adapt to disruptions arising from a changing climate. The Green Climate Fund will support projects, programmes, policies and other activities in developing country Parties using thematic funding windows. It is intended to be the centerpiece of efforts to raise Climate Finance of $100 billion a year by 2020. It engages directly with the private sector through its Private Sector Facility. The Fund will promote the paradigm shift towards low-emission and climate-resilient development pathways by providing support to developing countries to limit or reduce their greenhouse gas emissions and to adapt to the impacts of climate change, taking into account the needs of those developing countries particularly vulnerable to the adverse effects of climate change. The Fund will strive to maximize the impact of its funding for adaptation and mitigation, and seek a balance between the two, while promoting environmental, social, economic and development co-benefits and taking a gender-sensitive approach. The Fund is governed and supervised by a Board that will have full responsibility for funding decisions and that receives the guidance of the Conference of Parties (COP). The Fund is accountable to, and functions under the guidance of, the COP. The Green Climate fund has its own administrative set up with Headquarters at Songdo International Business District, Incheon, South Korea. Question 15 of 35 15. Question Consider the following statements regarding World Wide Fund (WWF) for Nature: WWF for Nature is an international non-governmental organization. It publishes ‘The living planet report’ every three years. It is working for the reduction of humanity’s footprint on the environment. How many of the above statements are correct? a) Only one b) Only two c) All three d) None Correct Solution (b) Statement 1 Statement 2 Statement 3 Correct Incorrect Correct WWF for Nature is an international non-governmental organization founded in 1961. It publishes ‘The Living Planet Report’ every two years. It is working for the reduction of humanity’s footprint on the environment.   Note: World Wide Fund (WWF) for Nature: The World Wide Fund (WWF) for Nature was founded on 29th April 1961. Its precursor organisation was the Conservation Foundation. Its original name was World Wildlife Fund. WWF Headquarters – Gland, Switzerland. It is an international non-profit organization dedicated to the preservation and conservation of nature and its different species. It is working for the reduction of humanity’s footprint on the environment. It is the world’s biggest conservation organization. It supports more than 1000 projects all over the world. WWF works in the areas of climate, food, forests, freshwater, oceans and wildlife primarily. It runs several projects in various fields in partnership with people, bodies and governments worldwide. In species conservation, they focus on tigers, elephants, gorillas, giant pandas, sea turtles, polar bears, rhinos and whales. Through its various projects, the organisation aims to check the degradation of the earth’s natural environment and create a future in which human beings live in harmony with nature. Campaigns launched by WWF include Earth Hour and Debt-for-Nature Swap. The famous ‘Living Planet Report’ (started in 1998) is published by the WWF every two years in which the health of the planet and the impact of human activities on nature are talked about. It is based on the Living Planet Index and the calculations of ecological footprints. What is the living planet index- It is a measure of the state of the world’s biological diversity based on population trends of vertebrate species in terrestrial, freshwater, and marine habitats. Incorrect Solution (b) Statement 1 Statement 2 Statement 3 Correct Incorrect Correct WWF for Nature is an international non-governmental organization founded in 1961. It publishes ‘The Living Planet Report’ every two years. It is working for the reduction of humanity’s footprint on the environment.   Note: World Wide Fund (WWF) for Nature: The World Wide Fund (WWF) for Nature was founded on 29th April 1961. Its precursor organisation was the Conservation Foundation. Its original name was World Wildlife Fund. WWF Headquarters – Gland, Switzerland. It is an international non-profit organization dedicated to the preservation and conservation of nature and its different species. It is working for the reduction of humanity’s footprint on the environment. It is the world’s biggest conservation organization. It supports more than 1000 projects all over the world. WWF works in the areas of climate, food, forests, freshwater, oceans and wildlife primarily. It runs several projects in various fields in partnership with people, bodies and governments worldwide. In species conservation, they focus on tigers, elephants, gorillas, giant pandas, sea turtles, polar bears, rhinos and whales. Through its various projects, the organisation aims to check the degradation of the earth’s natural environment and create a future in which human beings live in harmony with nature. Campaigns launched by WWF include Earth Hour and Debt-for-Nature Swap. The famous ‘Living Planet Report’ (started in 1998) is published by the WWF every two years in which the health of the planet and the impact of human activities on nature are talked about. It is based on the Living Planet Index and the calculations of ecological footprints. What is the living planet index- It is a measure of the state of the world’s biological diversity based on population trends of vertebrate species in terrestrial, freshwater, and marine habitats. Question 16 of 35 16. Question Consider the following statements about Man and Biosphere Programme: It was launched by IUCN and WWF. It aims to establish a scientific basis for the improvement of relationships between people and their environments. Biosphere Reserves are nominated by national government for inclusion in the world network of Biosphere Reserves. How many of the above statements are correct? a) Only one b) Only two c) All three d) None Correct Solution (b) Statement 1 Statement 2 Statement 3 Incorrect Correct Correct Man and Biosphere Programme was launched by UNESCO. It aims to establish a scientific basis for the improvement of relationships between people and their environments. Biosphere Reserves are nominated by national government for inclusion in the world network of Biosphere Reserves. Incorrect Solution (b) Statement 1 Statement 2 Statement 3 Incorrect Correct Correct Man and Biosphere Programme was launched by UNESCO. It aims to establish a scientific basis for the improvement of relationships between people and their environments. Biosphere Reserves are nominated by national government for inclusion in the world network of Biosphere Reserves. Question 17 of 35 17. Question With reference to the Cartagena Protocol, consider the following statements: It was adopted as a supplementary agreement to the Convention on Biological Diversity at the Rio Summit, 1992. It seeks to protect biological diversity from the potential risks posed by the transboundary movements of the living modified organisms resulting from modern biotechnology. It establishes an advance informed agreement (AIA) procedure for ensuring that countries are provided with the information necessary to make informed decisions before agreeing to the import of such organisms into their territory. It is a legally binding protocol even altering the rights and obligations of governments under the World Trade Organization. How many of the above statements are correct? a) Only one b) Only two c) Only three d) All four Correct Solution (b) Statement 1 Statement 2 Statement 3 Statement 4 Incorrect Correct Correct Incorrect As a supplementary agreement to the Convention on Biological Diversity, it was adopted in 2000 and entered into force in 2003. It seeks to protect biological diversity from the potential risks posed by the transboundary movements of the living modified organisms resulting from modern biotechnology. It establishes an advance informed agreement (AIA) procedure for ensuring that countries are provided with the information necessary to make informed decisions before agreeing to the import of such organisms into their territory. Although it is a legally binding protocol, the agreement does not alter the rights and obligations of governments under the World Trade Organization (WTO) or other existing international agreements. Incorrect Solution (b) Statement 1 Statement 2 Statement 3 Statement 4 Incorrect Correct Correct Incorrect As a supplementary agreement to the Convention on Biological Diversity, it was adopted in 2000 and entered into force in 2003. It seeks to protect biological diversity from the potential risks posed by the transboundary movements of the living modified organisms resulting from modern biotechnology. It establishes an advance informed agreement (AIA) procedure for ensuring that countries are provided with the information necessary to make informed decisions before agreeing to the import of such organisms into their territory. Although it is a legally binding protocol, the agreement does not alter the rights and obligations of governments under the World Trade Organization (WTO) or other existing international agreements. Question 18 of 35 18. Question With reference to Earth Summit, consider the following statements: It is the United Nations Conference on Environment and Development that took place in 1992 in Rio de Janeiro. Agenda 21, a non-binding action plan, was an outcome of this summit. Which of the statements given above is/are correct? a) 1 only b) 2 only c) Both 1 and 2 d) Neither 1 nor 2 Correct Solution (c) Statement 1 Statement 2 Correct Correct Earth Summit is the United Nations Conference on Environment and Development that took place in 1992 in Rio de Janeiro. Agenda 21, a non-binding action plan, was an outcome of this summit. Since 2015, Sustainable Development Goals are included in the newer Agenda 2030. Note: Earth Summit: The United Nations Conference on Environment and Development (UNCED), also known as the Rio de Janeiro Earth Summit, Rio Summit, Rio Conference, and Earth Summit, was a major United Nations conference held in Rio de Janeiro (Brazil) from 3 to 14 June 1992. This summit led to the development of the following documents: Rio Declaration on Environment and Development- Rio Declaration, in short, contained 27 principles that were supposed to guide countries in future sustainable development. Agenda 21- Agenda 21 is an action plan concerning sustainable development, but it is non-binding. Forest Principles- The Forest Principles is formally called ‘Non-Legally Binding Authoritative Statement of Principles for a Global Consensus on the Management, Conservation and Sustainable Development of All Types of Forests’. It makes many recommendations for conservation and sustainable development forestry and is non-binding. Incorrect Solution (c) Statement 1 Statement 2 Correct Correct Earth Summit is the United Nations Conference on Environment and Development that took place in 1992 in Rio de Janeiro. Agenda 21, a non-binding action plan, was an outcome of this summit. Since 2015, Sustainable Development Goals are included in the newer Agenda 2030. Note: Earth Summit: The United Nations Conference on Environment and Development (UNCED), also known as the Rio de Janeiro Earth Summit, Rio Summit, Rio Conference, and Earth Summit, was a major United Nations conference held in Rio de Janeiro (Brazil) from 3 to 14 June 1992. This summit led to the development of the following documents: Rio Declaration on Environment and Development- Rio Declaration, in short, contained 27 principles that were supposed to guide countries in future sustainable development. Agenda 21- Agenda 21 is an action plan concerning sustainable development, but it is non-binding. Forest Principles- The Forest Principles is formally called ‘Non-Legally Binding Authoritative Statement of Principles for a Global Consensus on the Management, Conservation and Sustainable Development of All Types of Forests’. It makes many recommendations for conservation and sustainable development forestry and is non-binding. Question 19 of 35 19. Question Consider the following statements with reference to the ‘World Water Council’: It is an inter-governmental organization under the aegis of United Nations. The Council focuses on the political dimensions of water security, adaptation and sustainability. Which of the statements given above is/are correct? a) 1 only b) 2 only c) Both 1 and 2 d) Neither 1 nor 2 Correct Solution (b) Statement 1 Statement 2 Incorrect Correct The World Water Council is a non-profit international multi-stakeholder platform. It was established in 1996. The Council focuses on the political dimensions of water security, adaptation and sustainability. Note: World Water Council: The World Water Council (WWC) is an international think tank. It was founded in 1996, with its headquarters in Marseille, France. It has 358 members (as of February 2020) which encompass organisations from the UN and intergovernmental organizations, the private sector (construction, engineering and manufacturing companies), governments and ministries, academic institutions, international organizations, local governments and civil society groups. Its stated mission is “to promote awareness, build political commitment and trigger action on critical water issues at all levels, including the highest decision-making level, to facilitate the efficient conservation, protection, development, planning, management, and use of water in all its dimensions on an environmentally sustainable basis for the benefit of all life on earth.” The Council also focuses on the political dimensions of water security, adaptation and sustainability. Incorrect Solution (b) Statement 1 Statement 2 Incorrect Correct The World Water Council is a non-profit international multi-stakeholder platform. It was established in 1996. The Council focuses on the political dimensions of water security, adaptation and sustainability. Note: World Water Council: The World Water Council (WWC) is an international think tank. It was founded in 1996, with its headquarters in Marseille, France. It has 358 members (as of February 2020) which encompass organisations from the UN and intergovernmental organizations, the private sector (construction, engineering and manufacturing companies), governments and ministries, academic institutions, international organizations, local governments and civil society groups. Its stated mission is “to promote awareness, build political commitment and trigger action on critical water issues at all levels, including the highest decision-making level, to facilitate the efficient conservation, protection, development, planning, management, and use of water in all its dimensions on an environmentally sustainable basis for the benefit of all life on earth.” The Council also focuses on the political dimensions of water security, adaptation and sustainability. Question 20 of 35 20. Question Consider the following statements with reference to the Bonn Convention: It is the only global convention specializing in the conservation of migratory species. It is an environmental treaty under the aegis of UNEP. Which of the statements given above is/are correct? a) 1 only b) 2 only c) Both 1 and 2 d) Neither 1 nor 2 Correct Solution (c) Statement 1 Statement 2 Correct Correct It is the only global convention specializing in the conservation of migratory species, their habitats and migration routes. It is an environmental treaty under the aegis of UNEP. Note: Bonn Convention: The Convention on the Conservation of Migratory Species of Wild Animals (also known as CMS or the Bonn Convention, not to be confused with the Bonn Agreement) aims to conserve terrestrial, marine and avian migratory species throughout their range. It is an intergovernmental treaty, concluded under the aegis of the United Nations Environment Programme, associated with the conservation of wildlife and habitats on a global scale. Since the Convention’s entry into force, its membership has grown steadily to include over 100 Parties from Africa, Central and South America, Asia, Europe and Oceania. The Convention was signed in 1979 in Bonn (hence the name) and entered into force in 1983. Incorrect Solution (c) Statement 1 Statement 2 Correct Correct It is the only global convention specializing in the conservation of migratory species, their habitats and migration routes. It is an environmental treaty under the aegis of UNEP. Note: Bonn Convention: The Convention on the Conservation of Migratory Species of Wild Animals (also known as CMS or the Bonn Convention, not to be confused with the Bonn Agreement) aims to conserve terrestrial, marine and avian migratory species throughout their range. It is an intergovernmental treaty, concluded under the aegis of the United Nations Environment Programme, associated with the conservation of wildlife and habitats on a global scale. Since the Convention’s entry into force, its membership has grown steadily to include over 100 Parties from Africa, Central and South America, Asia, Europe and Oceania. The Convention was signed in 1979 in Bonn (hence the name) and entered into force in 1983. Question 21 of 35 21. Question Which of the following statements about Down syndrome is true? a) It is caused by a mutation in the X chromosome. b) It is typically inherited from one's parents. c) It can only be diagnosed after birth through physical characteristics. d) It is a genetic disorder caused by an extra copy of chromosome 21. Correct Solution (d) Down syndrome, also known as trisomy 21, is caused by the presence of an extra copy of chromosome 21. Hence option d is correct. This additional genetic material alters the course of development and causes the characteristics associated with Down syndrome. It is not caused by a mutation in the X chromosome. While Down syndrome is a genetic disorder, it is not typically inherited from one’s parents, as it is usually the result of a random error in cell division during the formation of either the sperm or the egg. Down syndrome can be diagnosed before birth through prenatal screening tests such as amniocentesis or chorionic villus sampling, and through genetic testing. Incorrect Solution (d) Down syndrome, also known as trisomy 21, is caused by the presence of an extra copy of chromosome 21. Hence option d is correct. This additional genetic material alters the course of development and causes the characteristics associated with Down syndrome. It is not caused by a mutation in the X chromosome. While Down syndrome is a genetic disorder, it is not typically inherited from one’s parents, as it is usually the result of a random error in cell division during the formation of either the sperm or the egg. Down syndrome can be diagnosed before birth through prenatal screening tests such as amniocentesis or chorionic villus sampling, and through genetic testing. Question 22 of 35 22. Question Which of the following can be the advantages of Genome Mapping of a country? Identifying the genetic basis for various diseases. Understanding of population health trends Accurate prediction of individual life events. Targeted treatment of rare diseases arising from genetic anomalies. How many of the above statements are correct? a) Only one b) Only two c) Only three d) All four Correct Solution (c) Studying the Genetic makeup of a country can help in: It can identify the genetic basis or genetic risk factors for various diseases. Hence statement 1 is correct. It can provide insights into health trends and patterns within that population. It can be a useful tool in public health planning, preventive measures, and the allocation of resources. Hence statement 2 is correct. Genome mapping cannot accurately predict individual life events. Accurate prediction of an individual’s life events is influenced by various factors beyond genetics. Hence statement 3 is incorrect. In providing targeted treatments, especially for rare diseases that usually arise from genetic anomalies. Hence statement 4 is correct. Incorrect Solution (c) Studying the Genetic makeup of a country can help in: It can identify the genetic basis or genetic risk factors for various diseases. Hence statement 1 is correct. It can provide insights into health trends and patterns within that population. It can be a useful tool in public health planning, preventive measures, and the allocation of resources. Hence statement 2 is correct. Genome mapping cannot accurately predict individual life events. Accurate prediction of an individual’s life events is influenced by various factors beyond genetics. Hence statement 3 is incorrect. In providing targeted treatments, especially for rare diseases that usually arise from genetic anomalies. Hence statement 4 is correct. Question 23 of 35 23. Question Consider the following statements regarding the World Trade Organisation (WTO): It is the outcome of the Marrakesh Agreement signed in 1994. It is a UN specialised agency dealing with the rules of trade between nations. The Ministerial Conference is the highest decision-making body of the WTO. How many of the above statements are correct? a) Only one b) Only two c) All three d) None Correct Solution (b) The World Trade Organisation (WTO) is the outcome of the Marrakesh Agreement signed in 1994. Hence statement 1 is correct. The Marrakesh Agreement signed in 1994 marked the culmination of the 8-year-long Uruguay Round and established the World Trade Organization (WTO), which officially came into being on January 1, 1995. It is not a UN specialised agency dealing with the rules of trade between nations. Hence statement 2 is incorrect. However, it has maintained strong relations with the United Nations and its agencies since its establishment. The Ministerial Conference is the highest decision-making body of the WTO. Hence statement 3 is correct. The Ministerial Conference usually meets every two years. It brings together all members of the WTO, all of which are countries or customs unions. The Ministerial Conference can make decisions on all matters under any of the multilateral trade agreements.   Incorrect Solution (b) The World Trade Organisation (WTO) is the outcome of the Marrakesh Agreement signed in 1994. Hence statement 1 is correct. The Marrakesh Agreement signed in 1994 marked the culmination of the 8-year-long Uruguay Round and established the World Trade Organization (WTO), which officially came into being on January 1, 1995. It is not a UN specialised agency dealing with the rules of trade between nations. Hence statement 2 is incorrect. However, it has maintained strong relations with the United Nations and its agencies since its establishment. The Ministerial Conference is the highest decision-making body of the WTO. Hence statement 3 is correct. The Ministerial Conference usually meets every two years. It brings together all members of the WTO, all of which are countries or customs unions. The Ministerial Conference can make decisions on all matters under any of the multilateral trade agreements.   Question 24 of 35 24. Question Which of the following are covered under the adjudicating power of the Central Administrative Tribunal (CAT)? Members of the defence forces Officers and servants of the Supreme Court All-India services Civilian employees of defence services Choose the correct code: a) Only one b) Only two c) Only three d) All four Correct Solution (b) The original Constitution did not contain provisions concerning tribunals. The 42nd Amendment Act of 1976 added a new Part XIV-A to the Constitution. This part is entitled ‘Tribunals’ and consists of only two Articles–Article 323 A dealing with administrative tribunals and Article 323 B dealing with tribunals for other matters. The Central Administrative Tribunal is established under the Tribunals Act 1985 following Article 323-A of the constitution. The CAT exercises original jurisdiction concerning recruitment and all service matters of public servants covered by it. Its jurisdiction extends to the All-India services The Central Civil Service, Civilian employees of defence services Civil posts under the Centre and State governments. The members of the defence forces, officers and servants of the Supreme Court, and the secretarial staff of the Parliament are not covered by it. Hence option b is correct. Incorrect Solution (b) The original Constitution did not contain provisions concerning tribunals. The 42nd Amendment Act of 1976 added a new Part XIV-A to the Constitution. This part is entitled ‘Tribunals’ and consists of only two Articles–Article 323 A dealing with administrative tribunals and Article 323 B dealing with tribunals for other matters. The Central Administrative Tribunal is established under the Tribunals Act 1985 following Article 323-A of the constitution. The CAT exercises original jurisdiction concerning recruitment and all service matters of public servants covered by it. Its jurisdiction extends to the All-India services The Central Civil Service, Civilian employees of defence services Civil posts under the Centre and State governments. The members of the defence forces, officers and servants of the Supreme Court, and the secretarial staff of the Parliament are not covered by it. Hence option b is correct. Question 25 of 35 25. Question Consider the following statements regarding the Bochasanwasi Akshar Purushottam Swaminarayan Sanstha Mandir: It is built in the traditional Dravida style of architecture. It has been built by using fly ash instead of cement. It is surrounded by ‘Dome of Harmony’ and ‘Dome of Peace.’ How many of the above statements are correct? a) Only one b) Only two c) All three d) None Correct Solution (b) The Bochasanwasi Akshar Purushottam Swaminarayan Sanstha Mandir is built in the traditional Nagara style of architecture. Hence statement 1 is incorrect. It has been built by using fly ash instead of cement which helps to reduce its carbon footprint. Hence statement 2 is correct. Nano tiles have been installed with designs to keep visitors comfortable even in extreme heat. It has not used any ferrous material. The temple houses seven shrines, each dedicated to different deities hailing from the North, East, West, and South parts of India. It is surrounded by ‘Dome of Harmony’ and ‘Dome of Peace.’ Hence statement 3 is correct. It symbolizes unity and coexistence among different religions and cultures. Incorrect Solution (b) The Bochasanwasi Akshar Purushottam Swaminarayan Sanstha Mandir is built in the traditional Nagara style of architecture. Hence statement 1 is incorrect. It has been built by using fly ash instead of cement which helps to reduce its carbon footprint. Hence statement 2 is correct. Nano tiles have been installed with designs to keep visitors comfortable even in extreme heat. It has not used any ferrous material. The temple houses seven shrines, each dedicated to different deities hailing from the North, East, West, and South parts of India. It is surrounded by ‘Dome of Harmony’ and ‘Dome of Peace.’ Hence statement 3 is correct. It symbolizes unity and coexistence among different religions and cultures. Question 26 of 35 26. Question Consider the following statements: H. Marigowda is considered the Father of Indian Horticulture. India is the 2nd largest producer and exporter of fruits and vegetables. The productivity of horticulture crops is much higher compared to the productivity of food grains. How many of the above statements are correct? a) Only one b) Only two c) All three d) None Correct Solution (b) Horticulture is the branch of agriculture concerned with intensively cultured plants directly used by humansfor food, medicinal purposes, and aesthetic gratification. It is the cultivation, production, and sale of vegetables, fruits, flowers, herbs, ornamental or exotic plants. H. Bailey is considered the Father of American Horticulture. H. Marigowdais considered the Father of Indian Horticulture. Hence statement 1 is correct. India is the 2nd largest producerof fruits and vegetables. In terms of exports, India is ranked 14th in vegetables and 23rd in fruits, and its share in the global horticultural market is a mere 1%. Hence statement 2 is incorrect. The productivity of horticulture crops is much higher compared to the productivity of food grains. Hence statement 3 is correct. Incorrect Solution (b) Horticulture is the branch of agriculture concerned with intensively cultured plants directly used by humansfor food, medicinal purposes, and aesthetic gratification. It is the cultivation, production, and sale of vegetables, fruits, flowers, herbs, ornamental or exotic plants. H. Bailey is considered the Father of American Horticulture. H. Marigowdais considered the Father of Indian Horticulture. Hence statement 1 is correct. India is the 2nd largest producerof fruits and vegetables. In terms of exports, India is ranked 14th in vegetables and 23rd in fruits, and its share in the global horticultural market is a mere 1%. Hence statement 2 is incorrect. The productivity of horticulture crops is much higher compared to the productivity of food grains. Hence statement 3 is correct. Question 27 of 35 27. Question Consider the following statements regarding the Commission for Agricultural Costs and Prices (CACP): Minimum Support Price (MSP) is based on the recommendations of the CACP. It is an attached office of the Ministry of Agriculture and Farmers Welfare. The CACP recommends MSPs for 22 mandated crops and fair and remunerative prices (FRP) for sugarcane. How many of the above statements are correct? a) Only one b) Only two c) All three d) None Correct Solution (c) Minimum Support Price (MSP)is the guaranteed amount paid to farmers when the government buys their produce. MSP is based on the recommendations of the Commission for Agricultural Costs and Prices (CACP), which considers various factors such as cost of production, demand and supply, market price trends, inter-crop price parity, etc. Hence statement 1 is correct. It recommends MSP based on the A2+FL formula,which considers only paid-out costs and the imputed value of family labour. CACP is an attached office of the Ministry of Agriculture and Farmers Welfare.Hence statement 2 is correct. It came into existence in January 1965. The CACP recommends MSPs for 22 mandated cropsand fair and remunerative prices (FRP) for sugarcane. Hence statement 3 is correct.   Incorrect Solution (c) Minimum Support Price (MSP)is the guaranteed amount paid to farmers when the government buys their produce. MSP is based on the recommendations of the Commission for Agricultural Costs and Prices (CACP), which considers various factors such as cost of production, demand and supply, market price trends, inter-crop price parity, etc. Hence statement 1 is correct. It recommends MSP based on the A2+FL formula,which considers only paid-out costs and the imputed value of family labour. CACP is an attached office of the Ministry of Agriculture and Farmers Welfare.Hence statement 2 is correct. It came into existence in January 1965. The CACP recommends MSPs for 22 mandated cropsand fair and remunerative prices (FRP) for sugarcane. Hence statement 3 is correct.   Question 28 of 35 28. Question Consider the following statements regarding the Digital Public Infrastructure (DPI): India became the first country to develop all three foundational DPIs: Digital identity (Aadhar), Real-time fast payment (UPI), and Account Aggregator built on the Data Empowerment Protection Architecture (DEPA). The Data Empowerment and Protection Architecture (DEPA) establishes a digital framework enabling users to share their data solely under the terms defined by a third-party entity. Choose the correct code: a) 1 only b) 2 only c) Both 1 and 2 d) Neither 1 nor 2 Correct Solution (a) Digital Public Infrastructure (DPI) refers to blocks or platforms such as digital identification, payment infrastructure, and data exchange solutions that help countries deliver essential services to their people, empowering citizens and improving lives by enabling digital inclusion. DPIs mediate the flow of people, money, and information. These three sets become the foundation for developing an effective DPI ecosystem: First, the flow of people through a digital ID System. Second, the flow of money through a real-time fast payment system. And third, the flow of personal information through a consent-based data-sharing system to actualise the benefits of DPIs and to empower the citizen with a real ability to control data. IndiaStack is a set of APIs (Application programming interfaces) that allows governments, businesses, startups, and developers to utilise a unique digital Infrastructure to solve India’s hard problems towards presence-less, paperless, and cashless service delivery. India, through India Stack, became the first country to develop all three foundational DPIs, Digital identity (Aadhar), Real-time fast payment (UPI), and Account Aggregator built on the Data Empowerment Protection Architecture (DEPA). Hence statement 1 is correct. DEPA creates a digital framework that allows users to share their data on their terms through a third-party entity, known as Consent Manager. Hence statement 2 is incorrect. Incorrect Solution (a) Digital Public Infrastructure (DPI) refers to blocks or platforms such as digital identification, payment infrastructure, and data exchange solutions that help countries deliver essential services to their people, empowering citizens and improving lives by enabling digital inclusion. DPIs mediate the flow of people, money, and information. These three sets become the foundation for developing an effective DPI ecosystem: First, the flow of people through a digital ID System. Second, the flow of money through a real-time fast payment system. And third, the flow of personal information through a consent-based data-sharing system to actualise the benefits of DPIs and to empower the citizen with a real ability to control data. IndiaStack is a set of APIs (Application programming interfaces) that allows governments, businesses, startups, and developers to utilise a unique digital Infrastructure to solve India’s hard problems towards presence-less, paperless, and cashless service delivery. India, through India Stack, became the first country to develop all three foundational DPIs, Digital identity (Aadhar), Real-time fast payment (UPI), and Account Aggregator built on the Data Empowerment Protection Architecture (DEPA). Hence statement 1 is correct. DEPA creates a digital framework that allows users to share their data on their terms through a third-party entity, known as Consent Manager. Hence statement 2 is incorrect. Question 29 of 35 29. Question Consider the following statements regarding the Global Pulses Conference 2024: It was organized by the Ministry of Agriculture and Farmer Welfare and NITI Aayog. It is the industry’s largest global event, bringing together key players from more than 50 countries. Choose the correct code: a) 1 only b) 2 only c) Both 1 and 2 d) Neither 1 nor 2 Correct Solution (b) Global Pulses Conference 2024 is an annual gatheringof pulses producers, processors, and traders, which was recently jointly organized by the Global Pulse Confederation and the National Agricultural Cooperative Marketing Federation of India Ltd. (NAFED). Hence statement 1 is incorrect. India is the largest producer (25% of global production), consumer (27% of world consumption), and importer (14%)of pulses in the world. Pulses account for around 20% of the area under food grains and contribute around 7-10% of the total food grain production in the country. The Global Pulses Conference is the industry’s largest global event, bringing together key players from more than 50 countries. Hence statement 2 is correct. India is aiming toachieve self-sufficiency in pulses by 2027, focusing on expanding cultivation and supplying new varieties of seeds. Incorrect Solution (b) Global Pulses Conference 2024 is an annual gatheringof pulses producers, processors, and traders, which was recently jointly organized by the Global Pulse Confederation and the National Agricultural Cooperative Marketing Federation of India Ltd. (NAFED). Hence statement 1 is incorrect. India is the largest producer (25% of global production), consumer (27% of world consumption), and importer (14%)of pulses in the world. Pulses account for around 20% of the area under food grains and contribute around 7-10% of the total food grain production in the country. The Global Pulses Conference is the industry’s largest global event, bringing together key players from more than 50 countries. Hence statement 2 is correct. India is aiming toachieve self-sufficiency in pulses by 2027, focusing on expanding cultivation and supplying new varieties of seeds. Question 30 of 35 30. Question Consider the following statements about Purple Fest 2024: It aims to raise awareness about different disabilities and their impact on people’s lives. It is organized by the Department of Empowerment of Persons with Disabilities, Ministry of Social Justice and Empowerment. Choose the correct code: a) 1 only b) 2 only c) Both 1 and 2 d) Neither 1 nor 2 Correct Solution (c) Purple Fest 2024 aims to raise awareness about different disabilities and their impact on people’s lives. Hence statement 1 is correct. Purple Fest 2024 was organized at Rashtrapati Bhawan, celebrating diversity and unity. It promotes understanding, acceptance, and inclusion of persons with disabilities within society. It is organized by the Department of Empowerment of Persons with Disabilities, Ministry of Social Justice and Empowerment. Hence statement 2 is correct.   Incorrect Solution (c) Purple Fest 2024 aims to raise awareness about different disabilities and their impact on people’s lives. Hence statement 1 is correct. Purple Fest 2024 was organized at Rashtrapati Bhawan, celebrating diversity and unity. It promotes understanding, acceptance, and inclusion of persons with disabilities within society. It is organized by the Department of Empowerment of Persons with Disabilities, Ministry of Social Justice and Empowerment. Hence statement 2 is correct.   Question 31 of 35 31. Question From a group of 9 men and 6 women, eight persons are to be selected to form a committee so that at least 5 men are there on the committee. In how many ways can it be done? a) 5064 b) 4650 c) 4005 d) 5004 Correct Solution (c) We may have (5 men and 3 women) or (6 men and 2 women) or (7 men or 1 women) or (8 men only) Required number of ways = ( 9C5 * 6C3) + ( 9C6 * 6C2 ) + ( 9C7 * 6C1 ) + ( 9C8) Hence  nCr = nCn-r = 9C4 * 6C3 + 9C3 * 6C2 + 9C2 * 6C5 + 9C1 = [ 9 * 8 * 7 * 6/ 4 * 3 * 2 * 1] * [ 6 * 5 * 4 / 3 * 2 * 1] + 9C3 * 6C2 + 9C2 * 6C5 + 9C1 = 2520 + [ (9 * 8 * 7 / 3 * 2 * 1) * (6 * 5/ 2 * 1) + 9C3 * 6C2 + 9C2 * 6C5 + 9C1 = 2520 + 1260 + [ (9 * 8 / 2 * 1) * (6 * 1)) + 9C1 = 2520 + 1260 + 216 + 9 = 4005 Incorrect Solution (c) We may have (5 men and 3 women) or (6 men and 2 women) or (7 men or 1 women) or (8 men only) Required number of ways = ( 9C5 * 6C3) + ( 9C6 * 6C2 ) + ( 9C7 * 6C1 ) + ( 9C8) Hence  nCr = nCn-r = 9C4 * 6C3 + 9C3 * 6C2 + 9C2 * 6C5 + 9C1 = [ 9 * 8 * 7 * 6/ 4 * 3 * 2 * 1] * [ 6 * 5 * 4 / 3 * 2 * 1] + 9C3 * 6C2 + 9C2 * 6C5 + 9C1 = 2520 + [ (9 * 8 * 7 / 3 * 2 * 1) * (6 * 5/ 2 * 1) + 9C3 * 6C2 + 9C2 * 6C5 + 9C1 = 2520 + 1260 + [ (9 * 8 / 2 * 1) * (6 * 1)) + 9C1 = 2520 + 1260 + 216 + 9 = 4005 Question 32 of 35 32. Question If “PAVNPA” is the code for MUMBAI then the code for DELHI is a) GHOKL b) GNMKL c) GHUTL d) GKUTX Correct Solution (d) According to the same pattern, DELHI is written as 4 5 12 8 9 D E L H I   7 11 21 20 24 G K U T X   Incorrect Solution (d) According to the same pattern, DELHI is written as 4 5 12 8 9 D E L H I   7 11 21 20 24 G K U T X   Question 33 of 35 33. Question Let N be the greatest number that will divide 6360, 7420 and 9540, leaving the same remainder in each case. Then sum of the digits in N is: a) 7 b) 5 c) 6 d) 8 Correct Solution (a) N = H.C.F. of (9540 – 6360), (9540 – 7420) and (7420 – 6360) H.C.F. of 3180, 2120 and 1060 = 1060. Sum of digits in N = (1 + 0 + 6 + 0) = 7.   Incorrect Solution (a) N = H.C.F. of (9540 – 6360), (9540 – 7420) and (7420 – 6360) H.C.F. of 3180, 2120 and 1060 = 1060. Sum of digits in N = (1 + 0 + 6 + 0) = 7.   Question 34 of 35 34. Question If we listed all numbers from 100 to 10,000, how many times would the digit 5 is printed? a) 3980 b) 3700 c) 3840 d) 3780 Correct Solution (a) We need to consider all three digit and all 4-digit numbers. Three-digit numbers: A B C. 5 can be printed in the 100’s place or 10’s place or units place. => 100’s place: 5 B C. B can take values 0 to 9, C can take values 0 to 9. So, 5 gets printed in the 100’s place 100 times   => 10’s place: A 5 C. A can take values 1 to 9, C can take values 0 to 9. So, 5 gets printed in the 10’s place 90 times => Unit’s place: A B 5. A can take values 1 to 9, B can take values 0 to 9. So, 5 gets printed in the unit’s place 90 times So, 5 gets printed 280 times in 3-digit numbers   Four-digit numbers: A B C D. 5 can be printed in the 1000’s place, 100’s place or 10’s place or units place. => 1000’s place: 5 B C D. B can take values 0 to 9, C can take values 0 to 9, D can take values 0 to 9. So, 5 get printed in the 100’s place 1000 times. => 100’s place: A 5 C D. A can take values 1 to 9, C & D can take values 0 to 9. So, 5 gets printed in the 100’s place 900 times. => 10’s place: A B 5 D. A can take values 1 to 9, B & D can take values 0 to 9. So, 5 gets printed in the 10’s place 900 times. => Unit’s place: A B C 5. A can take values 1 to 9, B & C can take values 0 to 9. So, 5 gets printed in the unit’s place 900 times. 5 get printed 3700 times in 4-digit numbers. So, there are totally 3700 + 280 = 3980 numbers. Incorrect Solution (a) We need to consider all three digit and all 4-digit numbers. Three-digit numbers: A B C. 5 can be printed in the 100’s place or 10’s place or units place. => 100’s place: 5 B C. B can take values 0 to 9, C can take values 0 to 9. So, 5 gets printed in the 100’s place 100 times   => 10’s place: A 5 C. A can take values 1 to 9, C can take values 0 to 9. So, 5 gets printed in the 10’s place 90 times => Unit’s place: A B 5. A can take values 1 to 9, B can take values 0 to 9. So, 5 gets printed in the unit’s place 90 times So, 5 gets printed 280 times in 3-digit numbers   Four-digit numbers: A B C D. 5 can be printed in the 1000’s place, 100’s place or 10’s place or units place. => 1000’s place: 5 B C D. B can take values 0 to 9, C can take values 0 to 9, D can take values 0 to 9. So, 5 get printed in the 100’s place 1000 times. => 100’s place: A 5 C D. A can take values 1 to 9, C & D can take values 0 to 9. So, 5 gets printed in the 100’s place 900 times. => 10’s place: A B 5 D. A can take values 1 to 9, B & D can take values 0 to 9. So, 5 gets printed in the 10’s place 900 times. => Unit’s place: A B C 5. A can take values 1 to 9, B & C can take values 0 to 9. So, 5 gets printed in the unit’s place 900 times. 5 get printed 3700 times in 4-digit numbers. So, there are totally 3700 + 280 = 3980 numbers. Question 35 of 35 35. Question A cube has all its faces painted in different colors. It is cut into smaller cubes of equal sizes such that the side of the small cube is one-fourth the big cube. The number of small cubes with none of the sides painted is: a) 64 b) 8 c) 16 d) Not even one Correct Solution (b) Total number of cubes after cutting = n^3 = (4)^3 = 64 (n = number of pieces) Cube with 3-sides painted = 8 (for a proper cube number of the cube at corner is always 8) Cube with 2-sides painted = 12(n − 2) = 12(4 − 2) = 24 Cube with 1 side painted = 6(n − 2) 2 = 6(4 − 2) 2 = 24 Cube with no side painted = (Total number of smaller cubes − cubes with at least one side painted) [64− (8 + 24+ 24)] = 8­­­­­­­­ Incorrect Solution (b) Total number of cubes after cutting = n^3 = (4)^3 = 64 (n = number of pieces) Cube with 3-sides painted = 8 (for a proper cube number of the cube at corner is always 8) Cube with 2-sides painted = 12(n − 2) = 12(4 − 2) = 24 Cube with 1 side painted = 6(n − 2) 2 = 6(4 − 2) 2 = 24 Cube with no side painted = (Total number of smaller cubes − cubes with at least one side painted) [64− (8 + 24+ 24)] = 8­­­­­­­­ window.wpProQuizInitList = window.wpProQuizInitList || []; window.wpProQuizInitList.push({ id: '#wpProQuiz_3685', init: { quizId: 3685, mode: 1, globalPoints: 70, timelimit: 1800, resultsGrade: [0], bo: 704, qpp: 0, catPoints: [70], formPos: 0, lbn: "Test-summary", json: {"33104":{"type":"single","id":33104,"catId":0,"points":2,"correct":[0,1,0,0]},"33105":{"type":"single","id":33105,"catId":0,"points":2,"correct":[0,0,1,0]},"33108":{"type":"single","id":33108,"catId":0,"points":2,"correct":[0,0,1,0]},"33111":{"type":"single","id":33111,"catId":0,"points":2,"correct":[0,0,1,0]},"33113":{"type":"single","id":33113,"catId":0,"points":2,"correct":[1,0,0,0]},"33116":{"type":"single","id":33116,"catId":0,"points":2,"correct":[0,1,0,0]},"33119":{"type":"single","id":33119,"catId":0,"points":2,"correct":[0,1,0,0]},"33120":{"type":"single","id":33120,"catId":0,"points":2,"correct":[0,0,0,1]},"33121":{"type":"single","id":33121,"catId":0,"points":2,"correct":[0,1,0,0]},"33123":{"type":"single","id":33123,"catId":0,"points":2,"correct":[0,0,0,1]},"33125":{"type":"single","id":33125,"catId":0,"points":2,"correct":[1,0,0,0]},"33127":{"type":"single","id":33127,"catId":0,"points":2,"correct":[1,0,0,0]},"33130":{"type":"single","id":33130,"catId":0,"points":2,"correct":[0,1,0,0]},"33131":{"type":"single","id":33131,"catId":0,"points":2,"correct":[0,1,0,0]},"33132":{"type":"single","id":33132,"catId":0,"points":2,"correct":[0,1,0,0]},"33134":{"type":"single","id":33134,"catId":0,"points":2,"correct":[0,1,0,0]},"33137":{"type":"single","id":33137,"catId":0,"points":2,"correct":[0,1,0,0]},"33138":{"type":"single","id":33138,"catId":0,"points":2,"correct":[0,0,1,0]},"33140":{"type":"single","id":33140,"catId":0,"points":2,"correct":[0,1,0,0]},"33142":{"type":"single","id":33142,"catId":0,"points":2,"correct":[0,0,1,0]},"33144":{"type":"single","id":33144,"catId":0,"points":2,"correct":[0,0,0,1]},"33147":{"type":"single","id":33147,"catId":0,"points":2,"correct":[0,0,1,0]},"33150":{"type":"single","id":33150,"catId":0,"points":2,"correct":[0,1,0,0]},"33151":{"type":"single","id":33151,"catId":0,"points":2,"correct":[0,1,0,0]},"33153":{"type":"single","id":33153,"catId":0,"points":2,"correct":[0,1,0,0]},"33156":{"type":"single","id":33156,"catId":0,"points":2,"correct":[0,1,0,0]},"33157":{"type":"single","id":33157,"catId":0,"points":2,"correct":[0,0,1,0]},"33159":{"type":"single","id":33159,"catId":0,"points":2,"correct":[1,0,0,0]},"33162":{"type":"single","id":33162,"catId":0,"points":2,"correct":[0,1,0,0]},"33164":{"type":"single","id":33164,"catId":0,"points":2,"correct":[0,0,1,0]},"33165":{"type":"single","id":33165,"catId":0,"points":2,"correct":[0,0,1,0]},"33167":{"type":"single","id":33167,"catId":0,"points":2,"correct":[0,0,0,1]},"33170":{"type":"single","id":33170,"catId":0,"points":2,"correct":[1,0,0,0]},"33171":{"type":"single","id":33171,"catId":0,"points":2,"correct":[1,0,0,0]},"33172":{"type":"single","id":33172,"catId":0,"points":2,"correct":[0,1,0,0]}} } }); All the Best IASbaba

Daily Prelims CA Quiz

UPSC Quiz – 2024 : IASbaba’s Daily Current Affairs Quiz 27th April 2024

The Current Affairs questions are based on sources like ‘The Hindu’, ‘Indian Express’ and ‘PIB’, which are very important sources for UPSC Prelims Exam. The questions are focused on both the concepts and facts. The topics covered here are generally different from what is being covered under ‘Daily Current Affairs/Daily News Analysis (DNA) and Daily Static Quiz’ to avoid duplication. The questions would be published from Monday to Saturday before 2 PM. One should not spend more than 10 minutes on this initiative. Gear up and Make the Best Use of this initiative. Do remember that, “the difference between Ordinary and EXTRA-Ordinary is PRACTICE!!” Important Note: Don’t forget to post your marks in the comment section. Also, let us know if you enjoyed today’s test 🙂After completing the 5 questions, click on ‘View Questions’ to check your score, time taken, and solutions.To take the Test Click Here

[DAY 47] 60 DAY RAPID REVISION (RaRe) SERIES for UPSC Prelims 2024 – SCIENCE AND TECHNOLOGY, CURRENT AFFAIRS & CSAT TEST SERIES!

Archives Hello Friends The 60 Days Rapid Revision (RaRe) Series is IASbaba’s Flagship Initiative recommended by Toppers and loved by the aspirants’ community every year. It is the most comprehensive program which will help you complete the syllabus, revise and practice tests on a daily basis. The Programme on a daily basis includes Daily Prelims MCQs from Static (Monday – Saturday) Daily Static Quiz will cover all the topics of static subjects – Polity, History, Geography, Economics, Environment and Science and technology. 20 questions will be posted daily and these questions are framed from the topics mentioned in the schedule. It will ensure timely and streamlined revision of your static subjects. Daily Current Affairs MCQs (Monday – Saturday) Daily 5 Current Affairs questions, based on sources like ‘The Hindu’, ‘Indian Express’ and ‘PIB’, would be published from Monday to Saturday according to the schedule. Daily CSAT Quiz (Monday – Friday) CSAT has been an Achilles heel for many aspirants. Daily 5 CSAT Questions will be published. Note – Daily Test of 20 static questions, 10 current affairs, and 5 CSAT questions. (35 Prelims Questions) in QUIZ FORMAT will be updated on a daily basis. To Know More about 60 Days Rapid Revision (RaRe) Series – CLICK HERE   60 Day Rapid Revision (RaRe) Series Schedule – CLICK HERE  Important Note Comment your Scores in the Comment Section. This will keep you accountable, responsible and sincere in days to come. It will help us come out with the Cut-Off on a Daily Basis. Let us know if you enjoyed today’s test 🙂  You can post your comments in the given format  (1) Your Score (2) Matrix Meter (3) New Learning from the Test Time limit: 0 Test-summary 0 of 35 questions completed Questions: 1 2 3 4 5 6 7 8 9 10 11 12 13 14 15 16 17 18 19 20 21 22 23 24 25 26 27 28 29 30 31 32 33 34 35 Information The following Test is based on the syllabus of 60 Days Plan-2023 for UPSC IAS Prelims 2022. To view Solutions, follow these instructions: Click on – ‘Start Test’ button Solve Questions Click on ‘Test Summary’ button Click on ‘Finish Test’ button Now click on ‘View Questions’ button – here you will see solutions and links. You have already completed the test before. Hence you can not start it again. Test is loading... You must sign in or sign up to start the test. You have to finish following test, to start this test: Results 0 of 35 questions answered correctly Your time: Time has elapsed You have scored 0 points out of 0 points, (0) Average score     Your score     Categories Not categorized 0% Your result has been entered into leaderboard Loading Name: E-Mail: Captcha: maximum of 70 points Pos. Name Entered on Points Result Table is loading No data available 1 2 3 4 5 6 7 8 9 10 11 12 13 14 15 16 17 18 19 20 21 22 23 24 25 26 27 28 29 30 31 32 33 34 35 Answered Review Question 1 of 35 1. Question The compound used in black and white photography is? a) AgBr b) MgSO4 c) CaCO3 d) NaCl Correct Solution (a) Silver bromide (AgBr) is a soft, pale-yellow, water-insoluble salt well known (along with other silver halides) for its unusual sensitivity to light. This property has allowed silver halides to become the basis of modern photographic materials. AgBr is widely used in photographic film. Incorrect Solution (a) Silver bromide (AgBr) is a soft, pale-yellow, water-insoluble salt well known (along with other silver halides) for its unusual sensitivity to light. This property has allowed silver halides to become the basis of modern photographic materials. AgBr is widely used in photographic film. Question 2 of 35 2. Question Bisphenol A (BPA), a cause of concern, is a structural/key component in the manufacture of which of the following kinds of plastics? a) Polycarbonate b) Low-density polyethylene c) Polyvinylchloride d) Polyethylene Terephthalate Correct Solution (a) Bisphenol A (BPA) is a chemical compound primarily used in the manufacturing of various plastics. It is a colourless solid which is soluble in most common organic solvents, but has very poor solubility in water. BPA’s largest single application is as a co-monomer in the production of polycarbonates, which accounts for 65-70% of all BPA production. Incorrect Solution (a) Bisphenol A (BPA) is a chemical compound primarily used in the manufacturing of various plastics. It is a colourless solid which is soluble in most common organic solvents, but has very poor solubility in water. BPA’s largest single application is as a co-monomer in the production of polycarbonates, which accounts for 65-70% of all BPA production. Question 3 of 35 3. Question Which one among the following is a chemical process? a) Distillation of sea water b) Crystallization of impure salt c) Production of iodine from seaweeds d) Sublimation of iodine Correct Solution (c) All the given processes are physical processes except production of iodine from seaweeds. Incorrect Solution (c) All the given processes are physical processes except production of iodine from seaweeds. Question 4 of 35 4. Question With respect to Ethylene Glycol, consider the following statements: Ethylene Glycol is used as an anti-freezing agent in automobiles. Ethylene Glycol is highly poisonous in nature. Ethylene Glycol is an important precursor to polyester fibers and resins How many of the above statements are correct? a) Only one b) Only two c) All three d) None Correct Solution (c) Ethylene Glycol, also called ethane-1,2-diol, the simplest member of the glycol family of organic compounds. Ethylene glycol is a clear, sweet, slightly viscous liquid that boils at 198 °C (388.4 °F). Its most common use is as an automotive antifreeze. In addition to its use in antifreeze, ethylene glycol is used as an ingredient in hydraulic fluids, printing inks, and paint solvents. It is also used as a reagent in making polyesters, explosives, alkyd resins, and synthetic waxes. In the plastic industry, ethylene glycol is an important precursor to polyester fibers and resins. Polyethylene terephthalate, used to make plastic bottles for soft drinks, is prepared from ethylene glycol. Incorrect Solution (c) Ethylene Glycol, also called ethane-1,2-diol, the simplest member of the glycol family of organic compounds. Ethylene glycol is a clear, sweet, slightly viscous liquid that boils at 198 °C (388.4 °F). Its most common use is as an automotive antifreeze. In addition to its use in antifreeze, ethylene glycol is used as an ingredient in hydraulic fluids, printing inks, and paint solvents. It is also used as a reagent in making polyesters, explosives, alkyd resins, and synthetic waxes. In the plastic industry, ethylene glycol is an important precursor to polyester fibers and resins. Polyethylene terephthalate, used to make plastic bottles for soft drinks, is prepared from ethylene glycol. Question 5 of 35 5. Question Consider the following statements with respect to ‘Alloys’: Alloys are defined as mixtures of only two or more metals. They can be separated into their components by physical methods. Which of the statement/s given above is/are correct? a) 1 only b) 2 only c) Both 1 and 2 d) Neither 1 nor 2 Correct Solution (d) Statement 1 Statement 2 Incorrect Incorrect Alloys are mixtures of two or more metals or a metal and a non-metal. For example, brass is a mixture of approximately 30% zinc and 70% copper. They cannot be separated into their components by physical methods. But still, an alloy is considered as a mixture because it shows the properties of its constituents and can have variable composition. Incorrect Solution (d) Statement 1 Statement 2 Incorrect Incorrect Alloys are mixtures of two or more metals or a metal and a non-metal. For example, brass is a mixture of approximately 30% zinc and 70% copper. They cannot be separated into their components by physical methods. But still, an alloy is considered as a mixture because it shows the properties of its constituents and can have variable composition. Question 6 of 35 6. Question Consider the following statements with reference to properties of a colloid: A colloid is a homogenous mixture. The size of particles of a colloid are big enough to be individually seen by naked eyes. Colloids do not settle down when left undisturbed. How many of the above statements are correct? a) Only one b) Only two c) All three d) None Correct Solution (a) Statement 1 Statement 2 Statement 3 Incorrect Incorrect Correct The particles of a colloid are uniformly spread throughout the solution. Due to the relatively smaller size of particles, as compared to that of a suspension, the mixture appears to be homogeneous. But actually, a colloidal solution is a heterogeneous mixture, for example, milk. Because of the small size of colloidal particles, we cannot see them with naked eyes. But these particles can easily scatter a beam of visible light. Thus, the size of particles of a colloid is too small to be individually seen by naked eyes.   They do not settle down when left undisturbed, that is, a colloid is quite stable. They cannot be separated from the mixture by the process of filtration. But, a special technique of separation known as centrifugation, can be used to separate the colloidal particles. Incorrect Solution (a) Statement 1 Statement 2 Statement 3 Incorrect Incorrect Correct The particles of a colloid are uniformly spread throughout the solution. Due to the relatively smaller size of particles, as compared to that of a suspension, the mixture appears to be homogeneous. But actually, a colloidal solution is a heterogeneous mixture, for example, milk. Because of the small size of colloidal particles, we cannot see them with naked eyes. But these particles can easily scatter a beam of visible light. Thus, the size of particles of a colloid is too small to be individually seen by naked eyes.   They do not settle down when left undisturbed, that is, a colloid is quite stable. They cannot be separated from the mixture by the process of filtration. But, a special technique of separation known as centrifugation, can be used to separate the colloidal particles. Question 7 of 35 7. Question What do you understand by the term ‘Chromatography’? a) The study of the absorption and emission of light. b) The process of separation of components of a mixture. c) The process in which one or more substances are converted to one or more different substances. d) The study of physical interaction that occurs between electrically charged particles. Correct Solution (b) Statement 1 Statement 2 Statement 3 Statement 4 Incorrect Correct Incorrect Incorrect Spectroscopy is the study of the absorption and emission of light and other radiation by matter. It involves the splitting of light (or more precisely electromagnetic radiation) into its constituent wavelengths (a spectrum), which is done in much the same way as a prism splits light into a rainbow of colours. In chemical analysis, chromatography is a laboratory technique for the separation of a mixture into its components. The mixture is dissolved in a fluid solvent called the mobile phase, which carries it through a system on which a material called the stationary phase is fixed. Chemical reaction is a process in which one or more substances, the reactants, are converted to one or more different substances, the products. Substances are either chemical elements or compounds. A chemical reaction rearranges the constituent atoms of the reactants to create different substances as products. Electromagnetism is a branch of physics involving the study of the electromagnetic force, a type of physical interaction that occurs between electrically charged particles. The electromagnetic force is carried by electromagnetic fields composed of electric fields and magnetic fields, and it is responsible for electromagnetic radiation such as light. Incorrect Solution (b) Statement 1 Statement 2 Statement 3 Statement 4 Incorrect Correct Incorrect Incorrect Spectroscopy is the study of the absorption and emission of light and other radiation by matter. It involves the splitting of light (or more precisely electromagnetic radiation) into its constituent wavelengths (a spectrum), which is done in much the same way as a prism splits light into a rainbow of colours. In chemical analysis, chromatography is a laboratory technique for the separation of a mixture into its components. The mixture is dissolved in a fluid solvent called the mobile phase, which carries it through a system on which a material called the stationary phase is fixed. Chemical reaction is a process in which one or more substances, the reactants, are converted to one or more different substances, the products. Substances are either chemical elements or compounds. A chemical reaction rearranges the constituent atoms of the reactants to create different substances as products. Electromagnetism is a branch of physics involving the study of the electromagnetic force, a type of physical interaction that occurs between electrically charged particles. The electromagnetic force is carried by electromagnetic fields composed of electric fields and magnetic fields, and it is responsible for electromagnetic radiation such as light. Question 8 of 35 8. Question It was known by 1900 that the atom was indivisible particle but contained at least one sub-atomic particle i.e., the electron. This was identified by? a) J.J. Thomson b) E. Goldstein c) Ernest Rutherford d) John Dalton Correct Solution (a) Statement 1 Statement 2 Statement 3 Statement 4 Correct Incorrect Incorrect Incorrect J.J. Thomson (1856-1940), a British physicist, was born in Cheetham Hill, a suburb of Manchester, on 18 December 1856. He was awarded the Nobel prize in Physics in 1906 for his work on the discovery of electrons. He directed the Cavendish Laboratory at Cambridge for 35 years and 7 of his research assistants subsequently won Nobel prizes. Eugen Goldstein was a German physicist. He was an early investigator of discharge tubes, the discoverer of anode rays or canal rays, later identified as positive ions in the gas phase including the hydrogen ion. These are positively charged ions that are accelerated toward and through a perforated cathode in an evacuated tube. Ernest Rutherford was a New Zealand physicist who came to be known as the father of nuclear physics. Encyclopedia Britannica considers him to be the greatest experimentalist since Michael Faraday. He discovered the nucleus of the atom in 1911. John Dalton FRS was an English chemist, physicist and meteorologist. He is best known for introducing the atomic theory into chemistry, and for his research into colour blindness, sometimes referred to as Daltonism in his honour. Incorrect Solution (a) Statement 1 Statement 2 Statement 3 Statement 4 Correct Incorrect Incorrect Incorrect J.J. Thomson (1856-1940), a British physicist, was born in Cheetham Hill, a suburb of Manchester, on 18 December 1856. He was awarded the Nobel prize in Physics in 1906 for his work on the discovery of electrons. He directed the Cavendish Laboratory at Cambridge for 35 years and 7 of his research assistants subsequently won Nobel prizes. Eugen Goldstein was a German physicist. He was an early investigator of discharge tubes, the discoverer of anode rays or canal rays, later identified as positive ions in the gas phase including the hydrogen ion. These are positively charged ions that are accelerated toward and through a perforated cathode in an evacuated tube. Ernest Rutherford was a New Zealand physicist who came to be known as the father of nuclear physics. Encyclopedia Britannica considers him to be the greatest experimentalist since Michael Faraday. He discovered the nucleus of the atom in 1911. John Dalton FRS was an English chemist, physicist and meteorologist. He is best known for introducing the atomic theory into chemistry, and for his research into colour blindness, sometimes referred to as Daltonism in his honour. Question 9 of 35 9. Question Steel is an alloy of? a) Iron and Chromium b) Carbon and Nickel c) Iron and Carbon d) Carbon and Aluminium Correct Solution (c) Steel is an alloy made up of iron with typically a few tenths of a percent of carbon to improve its strength and fracture resistance compared to other forms of iron. Incorrect Solution (c) Steel is an alloy made up of iron with typically a few tenths of a percent of carbon to improve its strength and fracture resistance compared to other forms of iron. Question 10 of 35 10. Question With reference to the Acids and its sources in our daily life, select the correct pair: Name of acid                  Material found in Acetic acid –                  Vinegar Formic acid –                  Curd Oxalic acid –                 Spinach Select the answer: a) 1 only b) 1 and 2 only c) 2 and 3 only d) 1 and 3 only Correct Solution (d) Statement 1 Statement 2 Statement 3 Correct Incorrect Correct Acetic acid, systematically named ethanoic acid, is an acidic, colourless liquid and organic compound with the chemical formula CH₃COOH. Vinegar is at least 4% acetic acid by volume, making acetic acid the main component of vinegar apart from water and other trace elements. Formic acid is the simplest carboxylic acid, containing a single carbon. Occurs naturally in various sources including the venom of bee and ant stings, and is a useful organic synthetic reagent. Principally used as a preservative and antibacterial agent in livestock feed. Oxalic acid is an organic compound found in many plants, including leafy greens, vegetables, fruits, cocoa, nuts, seeds and spinach. In plants, it’s usually bound to minerals, forming oxalate. The terms “oxalic acid” and “oxalate” are used interchangeably in nutrition science. Incorrect Solution (d) Statement 1 Statement 2 Statement 3 Correct Incorrect Correct Acetic acid, systematically named ethanoic acid, is an acidic, colourless liquid and organic compound with the chemical formula CH₃COOH. Vinegar is at least 4% acetic acid by volume, making acetic acid the main component of vinegar apart from water and other trace elements. Formic acid is the simplest carboxylic acid, containing a single carbon. Occurs naturally in various sources including the venom of bee and ant stings, and is a useful organic synthetic reagent. Principally used as a preservative and antibacterial agent in livestock feed. Oxalic acid is an organic compound found in many plants, including leafy greens, vegetables, fruits, cocoa, nuts, seeds and spinach. In plants, it’s usually bound to minerals, forming oxalate. The terms “oxalic acid” and “oxalate” are used interchangeably in nutrition science. Question 11 of 35 11. Question Turmeric stains on the shirt turn red when washed with soap because? a) Soap water reacts with cloth of shirt to make it red. b) Soap solution is basic in nature which changes the yellow colour of turmeric into red. c) Material of shirt is exposed to liquid medium, which makes it change the colour. d) Turmeric has inherent property to turn red when it comes in contact with any solution. Correct Solution (b) Turmeric stains on the shirt turn red when washed with soap because the soap solution is basic in nature which changes the yellow colour of Turmeric into red colour. It is because turmeric contains tartaric acid, while the soap contains a sodium hydroxide base. Incorrect Solution (b) Turmeric stains on the shirt turn red when washed with soap because the soap solution is basic in nature which changes the yellow colour of Turmeric into red colour. It is because turmeric contains tartaric acid, while the soap contains a sodium hydroxide base. Question 12 of 35 12. Question With reference to classification of ‘Carbohydrates’, consider the following statements: Carbohydrates which yield a large number of monosaccharide units on hydrolysis are called oligosaccharides. Carbohydrates that yield two to ten monosaccharide units, on hydrolysis, are called polysaccharides. From the above given statements, identify the incorrect one/s: a) 1 only b) 2 only c) Both 1 and 2 d) Neither 1 nor 2 Correct Solution (c) Statement 1 Statement 2 Incorrect Incorrect A carbohydrate that cannot be hydrolysed further to give simpler unit of polyhydroxy aldehyde or ketone is called a monosaccharide. Carbohydrates that yield two to ten monosaccharide units, on hydrolysis, are called oligosaccharides. Carbohydrates which yield a large number of monosaccharide units on hydrolysis are called polysaccharides. Some common examples are starch, cellulose, glycogen, gums, etc. Polysaccharides are not sweet in taste, hence they are also called non-sugars. Incorrect Solution (c) Statement 1 Statement 2 Incorrect Incorrect A carbohydrate that cannot be hydrolysed further to give simpler unit of polyhydroxy aldehyde or ketone is called a monosaccharide. Carbohydrates that yield two to ten monosaccharide units, on hydrolysis, are called oligosaccharides. Carbohydrates which yield a large number of monosaccharide units on hydrolysis are called polysaccharides. Some common examples are starch, cellulose, glycogen, gums, etc. Polysaccharides are not sweet in taste, hence they are also called non-sugars. Question 13 of 35 13. Question Consider the following statements: Aqua regia is a mixture of nitric acid and hydrochloric acid. Aqua regia is a highly corrosive, fuming liquid. It is able to dissolve gold and platinum. How many of the above statements are correct? a) Only one b) Only two c) All three d) None Correct Solution (c) Statement 1 Statement 2 Statement 3 Correct Correct Correct Aqua regia, (Latin for ‘royal water’) is a freshly prepared mixture of concentrated hydrochloric acid and concentrated nitric acid in the ratio of 3:1. Aqua regia is a highly corrosive, fuming liquid. It can dissolve gold, even though neither of these acids can do so alone. It is one of the few reagents that is able to dissolve gold and platinum. Aqua regia is a mixture of nitric acid and hydrochloric acid, optimally in a molar ratio of 1:3. Aqua regia is a fuming liquid.   Hydrogen gas is not evolved when a metal reacts with nitric acid. It is because HNO3 is a strong oxidising agent. It oxidises the H2 produced to water and itself gets reduced to any of the nitrogen oxides (N2O, NO, NO2). But magnesium (Mg) and manganese (Mn) react with very dilute HNO3 to evolve H2 gas. Incorrect Solution (c) Statement 1 Statement 2 Statement 3 Correct Correct Correct Aqua regia, (Latin for ‘royal water’) is a freshly prepared mixture of concentrated hydrochloric acid and concentrated nitric acid in the ratio of 3:1. Aqua regia is a highly corrosive, fuming liquid. It can dissolve gold, even though neither of these acids can do so alone. It is one of the few reagents that is able to dissolve gold and platinum. Aqua regia is a mixture of nitric acid and hydrochloric acid, optimally in a molar ratio of 1:3. Aqua regia is a fuming liquid.   Hydrogen gas is not evolved when a metal reacts with nitric acid. It is because HNO3 is a strong oxidising agent. It oxidises the H2 produced to water and itself gets reduced to any of the nitrogen oxides (N2O, NO, NO2). But magnesium (Mg) and manganese (Mn) react with very dilute HNO3 to evolve H2 gas. Question 14 of 35 14. Question Consider the following statements: The chemical name of the compound ‘Baking Soda’ is sodium chloride. Carbon dioxide is produced when baking powder is heated or mixed in water. Which of the given statement/s is/are correct? a) 1 only b) 2 only c) Both 1 and 2 d) Neither 1 nor 2 Correct Solution (b) Statement 1 Statement 2 Incorrect Correct The baking soda is commonly used in the kitchen for making tasty crispy pakoras, etc. Sometimes it is added for faster cooking. The chemical name of the compound is sodium hydrogencarbonate (NaHCO3). It is produced using sodium chloride as one of the raw materials. Sodium Chloride (NaCl) is an ingredient in common salt. For making baking powder, which is a mixture of baking soda (sodium hydrogencarbonate) and a mild edible acid such as tartaric acid. When baking powder is heated or mixed in water, the following reaction takes place – NaHCO3 + H+ = CO2 + H2O + Sodium salt of acid Carbon dioxide produced during the reaction can cause bread or cake to rise making them soft and spongy. Incorrect Solution (b) Statement 1 Statement 2 Incorrect Correct The baking soda is commonly used in the kitchen for making tasty crispy pakoras, etc. Sometimes it is added for faster cooking. The chemical name of the compound is sodium hydrogencarbonate (NaHCO3). It is produced using sodium chloride as one of the raw materials. Sodium Chloride (NaCl) is an ingredient in common salt. For making baking powder, which is a mixture of baking soda (sodium hydrogencarbonate) and a mild edible acid such as tartaric acid. When baking powder is heated or mixed in water, the following reaction takes place – NaHCO3 + H+ = CO2 + H2O + Sodium salt of acid Carbon dioxide produced during the reaction can cause bread or cake to rise making them soft and spongy. Question 15 of 35 15. Question The p in pH stands for ‘potenz’ in German, meaning power. In this regard, consider the following statements with reference to ‘pH’: The pH of human stomach is acidic. Tooth decay starts when the pH of the mouth is lower than 5.5. Human body works within the pH range of 7.0 to 7.8. How many of the above statements are correct? a) Only one b) Only two c) All three d) None Correct Solution (c) Statement 1 Statement 2 Statement 3 Correct Correct Correct The pH of our stomach varies, but its natural state is between 1.5 and 3.5. This level rises when food enters the stomach; it can reach up to six, but it lowers again throughout digestion as stomach acid is secreted. It is very interesting to note that our stomach produces hydrochloric acid. It helps in the digestion of food without harming the stomach. Tooth decay starts when the pH of the mouth is lower than 5.5. Tooth enamel, made up of calcium hydroxyapatite (a crystalline form of calcium phosphate) is the hardest substance in the body. It does not dissolve in water, but is corroded when the pH in the mouth is below 5.5. Bacteria present in the mouth produce acids by degradation of sugar and food particles remaining in the mouth after eating. Human body works within the pH range of 7.0 to 7.8. Living organisms can survive only in a narrow range of pH change. When pH of rain water is less than 5.6, it is called acid rain. When acid rain flows into the rivers, it lowers the pH of the river water. The survival of aquatic life in such rivers becomes difficult. Incorrect Solution (c) Statement 1 Statement 2 Statement 3 Correct Correct Correct The pH of our stomach varies, but its natural state is between 1.5 and 3.5. This level rises when food enters the stomach; it can reach up to six, but it lowers again throughout digestion as stomach acid is secreted. It is very interesting to note that our stomach produces hydrochloric acid. It helps in the digestion of food without harming the stomach. Tooth decay starts when the pH of the mouth is lower than 5.5. Tooth enamel, made up of calcium hydroxyapatite (a crystalline form of calcium phosphate) is the hardest substance in the body. It does not dissolve in water, but is corroded when the pH in the mouth is below 5.5. Bacteria present in the mouth produce acids by degradation of sugar and food particles remaining in the mouth after eating. Human body works within the pH range of 7.0 to 7.8. Living organisms can survive only in a narrow range of pH change. When pH of rain water is less than 5.6, it is called acid rain. When acid rain flows into the rivers, it lowers the pH of the river water. The survival of aquatic life in such rivers becomes difficult. Question 16 of 35 16. Question Consider the following statements: Diamond and Graphite are crystalline forms of carbon. Crystallization is a technique used to purify solid compounds. Crystallization is based on the principles of solubility How many of the above statements are correct? a) Only one b) Only two c) All three d) None Correct Solution (c) Statement 1 Statement 2 Statement 3 Correct Correct Correct The crystalline forms of carbon are diamond, graphite and fullerene. Crystallization is the process by which a solid form, where the atoms or molecules are highly organized into a structure known as a crystal.   Crystallization is a technique which chemists use to purify solid compounds. Crystallization is based on the principles of solubility: compounds (solutes) tend to be more soluble in hot liquids (solvents) than they are in cold liquids.   If a saturated hot solution is allowed to cool, the solute is no longer soluble in the solvent and forms crystals of pure compound.   Impurities are excluded from the growing crystals and the pure solid crystals can be separated from the dissolved impurities by filtration.   Incorrect Solution (c) Statement 1 Statement 2 Statement 3 Correct Correct Correct The crystalline forms of carbon are diamond, graphite and fullerene. Crystallization is the process by which a solid form, where the atoms or molecules are highly organized into a structure known as a crystal.   Crystallization is a technique which chemists use to purify solid compounds. Crystallization is based on the principles of solubility: compounds (solutes) tend to be more soluble in hot liquids (solvents) than they are in cold liquids.   If a saturated hot solution is allowed to cool, the solute is no longer soluble in the solvent and forms crystals of pure compound.   Impurities are excluded from the growing crystals and the pure solid crystals can be separated from the dissolved impurities by filtration.   Question 17 of 35 17. Question Which one of the following is not a semiconductor? a) Germanium b) Silicon c) Quartz d) Gallium Arsenide Correct Solution (c) A semiconductor material has an electrical conductivity value falling between that of a conductor, such as metallic copper, and an insulator, such as glass. Its resistivity falls as its temperature rises; metals behave in the opposite way. Some examples of semiconductors are silicon, germanium and gallium arsenide. After silicon, gallium arsenide is the second-most common semiconductor and is used in laser diodes, solar cells, microwave-frequency integrated circuits, and others. Silicon is a critical element for fabricating most electronic circuits. Incorrect Solution (c) A semiconductor material has an electrical conductivity value falling between that of a conductor, such as metallic copper, and an insulator, such as glass. Its resistivity falls as its temperature rises; metals behave in the opposite way. Some examples of semiconductors are silicon, germanium and gallium arsenide. After silicon, gallium arsenide is the second-most common semiconductor and is used in laser diodes, solar cells, microwave-frequency integrated circuits, and others. Silicon is a critical element for fabricating most electronic circuits. Question 18 of 35 18. Question Which one of the following polymers is widely used for making bullet proof material? a) Polycarbonate b) Polyvinyl Chloride c) Polyethylene d) Polyamides Correct Solution (a) Polycarbonates (PC) are a group of thermoplastic polymers containing carbonate groups in their chemical structures. Polycarbonates used in engineering are strong, tough materials, and some grades are optically transparent. Much of the bullet-proof and shatter-proof glass that you see and experience in real life is actually made of bullet-resistant polycarbonate sheets. Incorrect Solution (a) Polycarbonates (PC) are a group of thermoplastic polymers containing carbonate groups in their chemical structures. Polycarbonates used in engineering are strong, tough materials, and some grades are optically transparent. Much of the bullet-proof and shatter-proof glass that you see and experience in real life is actually made of bullet-resistant polycarbonate sheets. Question 19 of 35 19. Question Magnetite is a mineral whose primary component is an: a) Iron Oxide b) Copper Oxide c) Aluminium Oxide d) Magnesium Oxide Correct Solution (a) Magnetite is a mineral whose primary component is an iron oxide that contains equal amounts of iron (II) and iron (III). Its empirical formula is Fe3O4, and it is often expressed as iron (II, III) oxide. Magnetite is found in igneous, metamorphic, and sedimentary rocks. As its name implies, it is magnetic; it and other inherently magnetic iron-containing minerals are described as being ferrimagnetic. Incorrect Solution (a) Magnetite is a mineral whose primary component is an iron oxide that contains equal amounts of iron (II) and iron (III). Its empirical formula is Fe3O4, and it is often expressed as iron (II, III) oxide. Magnetite is found in igneous, metamorphic, and sedimentary rocks. As its name implies, it is magnetic; it and other inherently magnetic iron-containing minerals are described as being ferrimagnetic. Question 20 of 35 20. Question Who of the following was the founder of the ‘Madras school’ of conformational analysis of biopolymers? a) Ramakrishnan Nagaraj b) G.N. Ramachandran c) Salim Ali d) CV Raman Correct Solution (b) G.N. Ramachandran, an outstanding figure in the field of protein structure, was the founder of the ‘Madras school’ of conformational analysis of biopolymers. His discovery of the triple helical structure of collagen in 1955 and his analysis of the allowed conformations of proteins through the use of the ‘Ramachandran plot’ ranks among the most outstanding contributions in structural biology. Ramakrishnan Nagaraj (born 1953) is an Indian biochemist, molecular biologist and the leader of a team of scientists working in the field of peptide biochemistry at Centre for Cellular and Molecular Biology (CCMB). He is known for his studies on hemolytic and antibacterial properties in synthetic analogs of bacterial toxins. Sálim Moizuddin Abdul Ali (12 November 1896 – 20 June 1987) was an Indian ornithologist and naturalist. Sometimes referred to as the “Birdman of India”, Salim Ali was the first Indian to conduct systematic bird surveys across India and wrote several bird books that popularized ornithology in India. He became a key figure behind the Bombay Natural History Society. Chandrasekhara Venkata Raman won the Nobel Prize for Physics in 1930 for his pioneering work on scattering of light. Born in Tiruchirapalli on November 7, 1888, he was the first Asian and first non-White to receive any Nobel Prize in the sciences. Incorrect Solution (b) G.N. Ramachandran, an outstanding figure in the field of protein structure, was the founder of the ‘Madras school’ of conformational analysis of biopolymers. His discovery of the triple helical structure of collagen in 1955 and his analysis of the allowed conformations of proteins through the use of the ‘Ramachandran plot’ ranks among the most outstanding contributions in structural biology. Ramakrishnan Nagaraj (born 1953) is an Indian biochemist, molecular biologist and the leader of a team of scientists working in the field of peptide biochemistry at Centre for Cellular and Molecular Biology (CCMB). He is known for his studies on hemolytic and antibacterial properties in synthetic analogs of bacterial toxins. Sálim Moizuddin Abdul Ali (12 November 1896 – 20 June 1987) was an Indian ornithologist and naturalist. Sometimes referred to as the “Birdman of India”, Salim Ali was the first Indian to conduct systematic bird surveys across India and wrote several bird books that popularized ornithology in India. He became a key figure behind the Bombay Natural History Society. Chandrasekhara Venkata Raman won the Nobel Prize for Physics in 1930 for his pioneering work on scattering of light. Born in Tiruchirapalli on November 7, 1888, he was the first Asian and first non-White to receive any Nobel Prize in the sciences. Question 21 of 35 21. Question “Sagar Aankalan Guidelines” recently in the news is? a) These guidelines aim to promote deep-sea exploration and resource extraction. b) These guidelines focus on developing sustainable fishing practices and conserving marine resources. c) These guidelines aim to map and benchmark the logistics performance of Indian ports. d) These guidelines aim for information sharing and cooperation between maritime agencies. Correct Solution (c) The Sagar Aankalan Guidelines were launched by the Indian government to establish a comprehensive framework for assessing and improving the performance of Indian ports. Hence option c is correct. It focuses on the National Benchmarking of Indian Ports’ Performance to enhance the efficiency and effectiveness of seaports across India. It would apply to all Indian seaports. It will transform the performance assessment of Indian ports. It will help in fulfilling “Maritime Amrit Kaal Vision 2047”. It will help in port development, modernization, green hydrogen and ammonia, port-led development, the cruise sector, business and commerce, shipbuilding, and knowledge sharing. Incorrect Solution (c) The Sagar Aankalan Guidelines were launched by the Indian government to establish a comprehensive framework for assessing and improving the performance of Indian ports. Hence option c is correct. It focuses on the National Benchmarking of Indian Ports’ Performance to enhance the efficiency and effectiveness of seaports across India. It would apply to all Indian seaports. It will transform the performance assessment of Indian ports. It will help in fulfilling “Maritime Amrit Kaal Vision 2047”. It will help in port development, modernization, green hydrogen and ammonia, port-led development, the cruise sector, business and commerce, shipbuilding, and knowledge sharing. Question 22 of 35 22. Question Consider the following statements regarding the Central Electricity Regulatory Commission (CERC): It is a statutory body established under the provisions of the Electricity Regulatory Commissions Act, of 1998. The Chairperson of the Central Electricity Authority is an ex-officio member of the CERC. It regulates the tariff of generating companies owned or controlled by the central government. It regulates the inter-state transmission of electricity and determines tariffs for inter-state transmission of electricity. How many of the above statements are correct? a) Only one b) Only two c) Only three d) All four Correct Solution (d) The Central Electricity Regulatory Commission (CERC) is a statutory body established under the provisions of the Electricity Regulatory Commissions Act, of 1998. Hence statement 1 is correct. It is the central commission for the purposes of the Electricity Act, of 2003, which has repealed the ERC Act, of 1998. The Commission consists of a Chairperson and four other members including the Chairperson, Central Electricity Authority, who is an ex-officio Member of the Commission. The Chairperson of the Central Electricity Authority is an ex-officio member of the CERC. Hence statement 2 is correct. It intends to promote competition, efficiency, and economy in bulk power markets, improve the quality of supply, promote investments, and advise the government on the removal of institutional barriers to bridge the demand-supply gap and thus foster the interests of consumers. It regulates the tariff of generating companies owned or controlled by the central government. Hence statement 3 is correct. It regulates the inter-state transmission of electricity and determines tariffs for inter-state transmission of electricity. Hence statement 4 is correct. It regulates the tariff of generating companies other than those owned or controlled by the central government specified in clause (a) if such generating companies enter into or otherwise have a composite scheme for the generation and sale of electricity in more than one state. It issues licences to persons to function as transmission licensees and electricity traders with respect to their interstate operations. Incorrect Solution (d) The Central Electricity Regulatory Commission (CERC) is a statutory body established under the provisions of the Electricity Regulatory Commissions Act, of 1998. Hence statement 1 is correct. It is the central commission for the purposes of the Electricity Act, of 2003, which has repealed the ERC Act, of 1998. The Commission consists of a Chairperson and four other members including the Chairperson, Central Electricity Authority, who is an ex-officio Member of the Commission. The Chairperson of the Central Electricity Authority is an ex-officio member of the CERC. Hence statement 2 is correct. It intends to promote competition, efficiency, and economy in bulk power markets, improve the quality of supply, promote investments, and advise the government on the removal of institutional barriers to bridge the demand-supply gap and thus foster the interests of consumers. It regulates the tariff of generating companies owned or controlled by the central government. Hence statement 3 is correct. It regulates the inter-state transmission of electricity and determines tariffs for inter-state transmission of electricity. Hence statement 4 is correct. It regulates the tariff of generating companies other than those owned or controlled by the central government specified in clause (a) if such generating companies enter into or otherwise have a composite scheme for the generation and sale of electricity in more than one state. It issues licences to persons to function as transmission licensees and electricity traders with respect to their interstate operations. Question 23 of 35 23. Question Consider the following statements: The 14th Conference of the Parties to the Convention on the United Nations Convention on Biological Diversity (UNCBD) adopted the Single Species Action Plan for the conservation of the Hawksbill Turtle. The International Single Species Action Plan is developed under the Agreement on the Conservation of African-Eurasian Migratory Waterbirds (AEWA). AEWA is an intergovernmental treaty dedicated to the conservation of migratory waterbirds and their habitats across Africa, Europe, the Middle East, Central Asia, Greenland, and the Canadian Archipelago. How many of the above statements are correct? a) Only one b) Only two c) All three d) None Correct Solution (a) The 14th Conference of the Parties to the Convention on the Conservation of Migratory Species of Wild Animals (CMS COP14) adopted the Single Species Action Plan for the conservation of the Hawksbill Turtle. Hence statement 1 is incorrect. The International Single Species Action Plan is developed under the Agreement on the Conservation of African-Eurasian Migratory Waterbirds (AEWA). Hence statement 2 is correct. It is to implement coordinated measures to restore migratory waterbird species to favourable conservation status. AEWA is an intergovernmental treaty dedicated to the conservation of migratory waterbirds and their habitats across Africa, Europe, the Middle East, Central Asia, Greenland, and the Canadian Archipelago. Hence statement 3 is correct. It was developed under the framework of the Convention on Migratory Species (CMS) and administered by the United Nations Environment Programme (UNEP). It brings together countries and the wider international conservation community to establish coordinated conservation and management of migratory waterbirds throughout their entire migratory range. Incorrect Solution (a) The 14th Conference of the Parties to the Convention on the Conservation of Migratory Species of Wild Animals (CMS COP14) adopted the Single Species Action Plan for the conservation of the Hawksbill Turtle. Hence statement 1 is incorrect. The International Single Species Action Plan is developed under the Agreement on the Conservation of African-Eurasian Migratory Waterbirds (AEWA). Hence statement 2 is correct. It is to implement coordinated measures to restore migratory waterbird species to favourable conservation status. AEWA is an intergovernmental treaty dedicated to the conservation of migratory waterbirds and their habitats across Africa, Europe, the Middle East, Central Asia, Greenland, and the Canadian Archipelago. Hence statement 3 is correct. It was developed under the framework of the Convention on Migratory Species (CMS) and administered by the United Nations Environment Programme (UNEP). It brings together countries and the wider international conservation community to establish coordinated conservation and management of migratory waterbirds throughout their entire migratory range. Question 24 of 35 24. Question Consider the following statements regarding Papua New Guinea: It is an island country that lies in the south-western Atlantic. It has Indonesia to the west, Australia to the south, and the Solomon Islands to the southeast. It is the world’s most linguistically diverse country, with more than 800 languages. It is a constitutional monarchy and a member of the Commonwealth. How many of the above statements are correct? a) Only one b) Only two c) Only three d) All four Correct Solution (c) Papua New Guinea is an island country that lies in the south-western Pacific. Hence statement 1 is incorrect. Its capital is Port Moresby. It has Indonesia to the west, Australia to the south, and the Solomon Islands to the southeast. Hence statement 2 is correct. It is mainly mountainous but has low-lying plains in southern New Guinea. The islands that constitute Papua New Guinea were settled over a period of 40,000 years by a mixture of peoples who are generally referred to as Melanesians. It is the world’s most linguistically diverse country, with more than 800 languages. Hence statement 3 is correct. English is the main language of government and commerce. In most everyday contexts, the most widely spoken language is Tok Pisin. The majority of Papua New Guinea’s people are at least nominally Christian. It is a constitutional monarchy and a member of the Commonwealth. Hence statement 4 is correct. Papua New Guinea became self-governing on December 1, 1973, and achieved independence on September 16, 1975. Incorrect Solution (c) Papua New Guinea is an island country that lies in the south-western Pacific. Hence statement 1 is incorrect. Its capital is Port Moresby. It has Indonesia to the west, Australia to the south, and the Solomon Islands to the southeast. Hence statement 2 is correct. It is mainly mountainous but has low-lying plains in southern New Guinea. The islands that constitute Papua New Guinea were settled over a period of 40,000 years by a mixture of peoples who are generally referred to as Melanesians. It is the world’s most linguistically diverse country, with more than 800 languages. Hence statement 3 is correct. English is the main language of government and commerce. In most everyday contexts, the most widely spoken language is Tok Pisin. The majority of Papua New Guinea’s people are at least nominally Christian. It is a constitutional monarchy and a member of the Commonwealth. Hence statement 4 is correct. Papua New Guinea became self-governing on December 1, 1973, and achieved independence on September 16, 1975. Question 25 of 35 25. Question Consider the following statements regarding the Director General of Civil Aviation (DGCA): Its primary function is to ensure the safety of passengers and crew members on all flights operating in India. It is responsible for the regulation of air transport services to/from/within India and for the enforcement of civil air regulations, air safety, and airworthiness standards. It investigates any incidents or accidents that occur within the Indian airspace and takes appropriate action to prevent similar incidents from happening in the future. How many of the above statements are correct? a) Only one b) Only two c) All three d) None Correct Solution (c) The Director General of Civil Aviation (DGCA) is the regulatory body in the field of civil aviation primarily dealing with safety issues. It is an attached office of the Ministry of Civil Aviation. Its primary function is to ensure the safety of passengers and crew members on all flights operating in India. Hence statement 1 is correct. In addition to safety, the DGCA also plays a crucial role in the growth and development of the Indian aviation industry. The organization works closely with airlines and airport operators to promote the industry’s growth and improve the overall travel experience for passengers. It also coordinates all regulatory functions with the International Civil Aviation Organisation. It is responsible for the regulation of air transport services to/from/within India and for the enforcement of civil air regulations, air safety, and airworthiness standards. Hence statement 2 is correct. It plays a crucial role in developing new airports and modernizing existing facilities to ensure that they can meet the growing demands of the aviation industry. It investigates any incidents or accidents that occur within the Indian airspace and takes appropriate action to prevent similar incidents from happening in the future. Hence statement 3 is correct.   Incorrect Solution (c) The Director General of Civil Aviation (DGCA) is the regulatory body in the field of civil aviation primarily dealing with safety issues. It is an attached office of the Ministry of Civil Aviation. Its primary function is to ensure the safety of passengers and crew members on all flights operating in India. Hence statement 1 is correct. In addition to safety, the DGCA also plays a crucial role in the growth and development of the Indian aviation industry. The organization works closely with airlines and airport operators to promote the industry’s growth and improve the overall travel experience for passengers. It also coordinates all regulatory functions with the International Civil Aviation Organisation. It is responsible for the regulation of air transport services to/from/within India and for the enforcement of civil air regulations, air safety, and airworthiness standards. Hence statement 2 is correct. It plays a crucial role in developing new airports and modernizing existing facilities to ensure that they can meet the growing demands of the aviation industry. It investigates any incidents or accidents that occur within the Indian airspace and takes appropriate action to prevent similar incidents from happening in the future. Hence statement 3 is correct.   Question 26 of 35 26. Question Consider the following statements regarding the Melghat Tiger Reserve: It is a critical tiger habitat located in Vindhya Hill in northern Madhya Pradesh. It is a catchment area for the Khandu, Khapra, Sipna, Gadga, and Dolar, all of which are tributaries of the River Tapti. The Korkus are the largest tribal community in Melghat. How many of the above statements are correct? a) Only one b) Only two c) All three d) None Correct Solution (b) The Melghat Tiger Reserve is located in the Amaravati district of Maharashtra. Hence statement 1 is incorrect. It is located on the southern offshoot of the Satpura Hill Range in Central India, called Gavilgarh Hill. It was established as a wildlife sanctuary in 1967 and was declared a tiger reserve in 1974. It was the first tiger reserve in Maharashtra. It was among the first nine tiger reserves notified in 1973-74 under Project Tiger. The name ‘Melghat’ means the confluence of various ‘ghats’ or valleys, as is typical of the landscape of this Tiger Reserve. It is a catchment area for the Khandu, Khapra, Sipna, Gadga, and Dolar, all of which are tributaries of the River Tapti. Hence statement 2 is correct. The forest is tropical dry deciduous, and dominated by teak. The Tapti River and the Gawilgadh ridge of the Satpura Range form the boundaries of the reserve. The Korkus are the largest tribal community in Melghat. Hence statement 3 is correct. Other communities include the Gawli community, the Gond tribe, and several other smaller tribal communities. Incorrect Solution (b) The Melghat Tiger Reserve is located in the Amaravati district of Maharashtra. Hence statement 1 is incorrect. It is located on the southern offshoot of the Satpura Hill Range in Central India, called Gavilgarh Hill. It was established as a wildlife sanctuary in 1967 and was declared a tiger reserve in 1974. It was the first tiger reserve in Maharashtra. It was among the first nine tiger reserves notified in 1973-74 under Project Tiger. The name ‘Melghat’ means the confluence of various ‘ghats’ or valleys, as is typical of the landscape of this Tiger Reserve. It is a catchment area for the Khandu, Khapra, Sipna, Gadga, and Dolar, all of which are tributaries of the River Tapti. Hence statement 2 is correct. The forest is tropical dry deciduous, and dominated by teak. The Tapti River and the Gawilgadh ridge of the Satpura Range form the boundaries of the reserve. The Korkus are the largest tribal community in Melghat. Hence statement 3 is correct. Other communities include the Gawli community, the Gond tribe, and several other smaller tribal communities. Question 27 of 35 27. Question Consider the following statements: National Electronic Funds Transfer (NEFT) is suitable for both small and large-value transactions. Real Time Gross Settlement (RTGS) system does not operate on weekends. Under Immediate Payment Service (IMPS), a minimum transaction amount is ₹2 lakhs. How many of the above statements are correct? a) Only one b) Only two c) All three d) None Correct Solution (a) National Electronic Funds Transfer (NEFT) is suitable for both small and large-value transactions. It allows individuals to transfer any amount to the recipient’s account on a one-on-one transfer basis without a maximum limit for funds that can be transferred in a single day. Hence statement 1 is correct. The Real Time Gross Settlement (RTGS) system is available 24 hours a day, 7 days a week, including weekends and bank holidays. Hence statement 2 is incorrect. Immediate Payment Service allows for both peer-to-peer transfers and merchant payments. IMPS is available 24/7 throughout the year, including bank holidays, and enables real-time instant inter-bank funds to transfer with no upper limit for transactions. The upper limit usually varies from bank to bank. There is also no lower limit or minimum transaction amount specified for IMPS. Hence statement 3 is incorrect. Incorrect Solution (a) National Electronic Funds Transfer (NEFT) is suitable for both small and large-value transactions. It allows individuals to transfer any amount to the recipient’s account on a one-on-one transfer basis without a maximum limit for funds that can be transferred in a single day. Hence statement 1 is correct. The Real Time Gross Settlement (RTGS) system is available 24 hours a day, 7 days a week, including weekends and bank holidays. Hence statement 2 is incorrect. Immediate Payment Service allows for both peer-to-peer transfers and merchant payments. IMPS is available 24/7 throughout the year, including bank holidays, and enables real-time instant inter-bank funds to transfer with no upper limit for transactions. The upper limit usually varies from bank to bank. There is also no lower limit or minimum transaction amount specified for IMPS. Hence statement 3 is incorrect. Question 28 of 35 28. Question Consider the following statements: Cryptocurrencies in India fall under the Virtual Digital Assets (VDAs) category and are not subject to taxation. Bitcoin is an open-source digital currency, that facilitates instant payments without central authority issuance. Choose the correct code: a) 1 only b) 2 only c) Both 1 and 2 d) Neither 1 nor 2 Correct Solution (b) Cryptocurrencies in India fall under the virtual digital assets (VDAs) category and are subject to taxation. Hence statement 1 is incorrect. The profits generated from cryptocurrency trading are taxed at a rate of 30%, with an additional 4% cess (Union budget 2022-23). In 2022, the RBI launched its own Central Bank Digital Currency (CBDC) known as the e-Rupee which is based on blockchain technology. Bitcoin is an open-source digital currency, that facilitates instant payments without central authority issuance. Hence statement 2 is correct. Note: A bitcoin halving is an event where the reward for mining new blocks is halved, meaning miners receive 50% fewer bitcoins for verifying transactions. Bitcoin halving is scheduled to occur once every 210,000 blocks – roughly every four years – until the maximum supply of 21 million bitcoins has been generated by the network.   Incorrect Solution (b) Cryptocurrencies in India fall under the virtual digital assets (VDAs) category and are subject to taxation. Hence statement 1 is incorrect. The profits generated from cryptocurrency trading are taxed at a rate of 30%, with an additional 4% cess (Union budget 2022-23). In 2022, the RBI launched its own Central Bank Digital Currency (CBDC) known as the e-Rupee which is based on blockchain technology. Bitcoin is an open-source digital currency, that facilitates instant payments without central authority issuance. Hence statement 2 is correct. Note: A bitcoin halving is an event where the reward for mining new blocks is halved, meaning miners receive 50% fewer bitcoins for verifying transactions. Bitcoin halving is scheduled to occur once every 210,000 blocks – roughly every four years – until the maximum supply of 21 million bitcoins has been generated by the network.   Question 29 of 35 29. Question Consider the following statements about Google DeepMind’s Genie: It is the first generative interactive environment that has been trained in an unsupervised manner from unlabelled internet videos. It can generate an endless variety of playable (action-controllable) worlds from synthetic images, photographs, and even sketches. Choose the correct code: a) 1 only b) 2 only c) Both 1 and 2 d) Neither 1 nor 2 Correct Solution (c) Google DeepMind is a British-American AI research laboratory that is a subsidiary of Google. DeepMind is based in London and has research centres in Canada, France, Germany, and the US. It has introduced Genie AI (Artificial Intelligence), a new model that can generate interactive video games from just a text or image prompt. It is the first generative interactive environment that has been trained in an unsupervised manner from unlabelled internet videos. Hence statement 1 is correct. Generative Interactive Environments (Genie) is a foundation world model that is trained on videos sourced from the Internet. The model can “generate an endless variety of playable (action-controllable) worlds from synthetic images, photographs, and even sketches”. Hence statement 2 is correct. Incorrect Solution (c) Google DeepMind is a British-American AI research laboratory that is a subsidiary of Google. DeepMind is based in London and has research centres in Canada, France, Germany, and the US. It has introduced Genie AI (Artificial Intelligence), a new model that can generate interactive video games from just a text or image prompt. It is the first generative interactive environment that has been trained in an unsupervised manner from unlabelled internet videos. Hence statement 1 is correct. Generative Interactive Environments (Genie) is a foundation world model that is trained on videos sourced from the Internet. The model can “generate an endless variety of playable (action-controllable) worlds from synthetic images, photographs, and even sketches”. Hence statement 2 is correct. Question 30 of 35 30. Question Consider the following statements about Uyghurs: They are a predominantly Muslim minority Turkic ethnic group, whose origins can be traced to Central and East Asia. Currently, the largest population of the Uighur ethnic community lives in the Xinjiang region of Turkey. Choose the correct code: a) 1 only b) 2 only c) Both 1 and 2 d) Neither 1 nor 2 Correct Solution (a) Uyghurs are a predominantly Muslim minority Turkic ethnic group, whose origins can be traced to Central and East Asia. Hence statement 1 is correct. The Uighurs speak their own language, similar to Turkish, and see themselves as culturally and ethnically close to Central Asian nations. The Uighurs are considered to be one of the 55 officially recognized ethnic minority communities in China. However, China recognises the community only as a regional minority and rejects that they are an indigenous group. Currently, the largest population of the Uighur ethnic community lives in the Xinjiang region of China. Hence statement 2 is incorrect. Incorrect Solution (a) Uyghurs are a predominantly Muslim minority Turkic ethnic group, whose origins can be traced to Central and East Asia. Hence statement 1 is correct. The Uighurs speak their own language, similar to Turkish, and see themselves as culturally and ethnically close to Central Asian nations. The Uighurs are considered to be one of the 55 officially recognized ethnic minority communities in China. However, China recognises the community only as a regional minority and rejects that they are an indigenous group. Currently, the largest population of the Uighur ethnic community lives in the Xinjiang region of China. Hence statement 2 is incorrect. Question 31 of 35 31. Question At 5’O clock, clock ticks 5 times. The time between first and last tick was 36sec. How much time it takes at 11’O clock? a) 60 seconds b) 90 seconds c) 120 seconds d) 30 seconds Correct Solution (b) At 5’o clock it ticks 5 times so there are total 4-time intervals. Total time taken = 36 seconds Hence, the time taken between any two consecutive ticks = 36/4 seconds = 9 seconds Similarly, clock will tick 11 times at 11 o’ clock. At 11o’clock there will be 10 intervals. Hence total time needed at 11 o’clock = 10 × 9 = 90 seconds. Incorrect Solution (b) At 5’o clock it ticks 5 times so there are total 4-time intervals. Total time taken = 36 seconds Hence, the time taken between any two consecutive ticks = 36/4 seconds = 9 seconds Similarly, clock will tick 11 times at 11 o’ clock. At 11o’clock there will be 10 intervals. Hence total time needed at 11 o’clock = 10 × 9 = 90 seconds. Question 32 of 35 32. Question Out of the four annual examinations, each with a total of 200 marks, a student secured average marks of 45%, 50% and 55% in the first, second and third annual examinations. To have an overall average of 60%, how many marks does the student need to secure in the fourth annual examination? a) 160 b) 170 c) 180 d) 190 Correct Solution (c)              Let the average marks in the third Annual examination be x. Total marks = (Marks in first + second + third + forth) Annual examination 4(60)( 200/100) = (45/100) (200) + ( 50/100) (200) + ( 55/100) (200) + ( x/100) (200) 4(60) = 45 + 50 +55 + x x = 90 So, the student must score 90% in the fourth annual examination to secure 60% overall average. ∴ Average marks in the fourth annual examination (90/100) × 200 = 180 marks. Incorrect Solution (c)              Let the average marks in the third Annual examination be x. Total marks = (Marks in first + second + third + forth) Annual examination 4(60)( 200/100) = (45/100) (200) + ( 50/100) (200) + ( 55/100) (200) + ( x/100) (200) 4(60) = 45 + 50 +55 + x x = 90 So, the student must score 90% in the fourth annual examination to secure 60% overall average. ∴ Average marks in the fourth annual examination (90/100) × 200 = 180 marks. Question 33 of 35 33. Question The sum of 3 numbers is 85. If third number be one fifth of the first and the first number be twice the second, find the second number. a) 20 b) 25 c) 10 d) 15 Correct Solution (b) From the details given in the question, the Ratio of the numbers = 1: 1/2: 1/5 = 10: 5: 2 Second number = 5/17 × 85 = 25. Incorrect Solution (b) From the details given in the question, the Ratio of the numbers = 1: 1/2: 1/5 = 10: 5: 2 Second number = 5/17 × 85 = 25. Question 34 of 35 34. Question 3ab0 is a four-digit number divisible by 25. If the number formed from the two digits ab is a multiple of 17, then ab a) 51 b) 34 c) 85 d) 17 Correct Solution (c) We have given that the number 3ab0 is divisible by 25. Any number divisible by 25 ends with the last two digits 00, 25, 50, or 75. So, b0 should equal 00 or 50. Hence, b=0 or 5. Since a is now free to take any digit from 0 through 9, ab can have multiple values. We also have that ab is divisible by 17. The multiples of 17 are 17, 34, 51, 68, 85, 102, 119, 136, 153 and 170. Among these, the only number ending with 0 or 5 is 85. Hence, ab=85   Incorrect Solution (c) We have given that the number 3ab0 is divisible by 25. Any number divisible by 25 ends with the last two digits 00, 25, 50, or 75. So, b0 should equal 00 or 50. Hence, b=0 or 5. Since a is now free to take any digit from 0 through 9, ab can have multiple values. We also have that ab is divisible by 17. The multiples of 17 are 17, 34, 51, 68, 85, 102, 119, 136, 153 and 170. Among these, the only number ending with 0 or 5 is 85. Hence, ab=85   Question 35 of 35 35. Question A 40 litre mixture of oil and water contains oil and water in the ratio 3 : 2. Now, 10 litres of the mixture is removed and replaced with pure oil. This operation is done one more time. What is the ratio of water and oil in the resultant mixture? a) 12 : 31 b) 7 : 19 c) 9 : 31 d) 11 : 31 Correct Solution (c) Original quantity of oil and water is 24 litres and 16 litres. Let the final quantity of water be a. 10 litres is being removed from 40 litres twice. ∴ a/16 = [1 – (10/40)]^2 = 9/16 ∴ a = 9 Hence, the final solution has 9 litres of water and 31 litres of oil. ∴ Required ratio = 9 : 31 Hence, option c. Incorrect Solution (c) Original quantity of oil and water is 24 litres and 16 litres. Let the final quantity of water be a. 10 litres is being removed from 40 litres twice. ∴ a/16 = [1 – (10/40)]^2 = 9/16 ∴ a = 9 Hence, the final solution has 9 litres of water and 31 litres of oil. ∴ Required ratio = 9 : 31 Hence, option c. window.wpProQuizInitList = window.wpProQuizInitList || []; window.wpProQuizInitList.push({ id: '#wpProQuiz_3678', init: { quizId: 3678, mode: 1, globalPoints: 70, timelimit: 1800, resultsGrade: [0], bo: 704, qpp: 0, catPoints: [70], formPos: 0, lbn: "Test-summary", json: {"33017":{"type":"single","id":33017,"catId":0,"points":2,"correct":[1,0,0,0]},"33018":{"type":"single","id":33018,"catId":0,"points":2,"correct":[1,0,0,0]},"33019":{"type":"single","id":33019,"catId":0,"points":2,"correct":[0,0,1,0]},"33022":{"type":"single","id":33022,"catId":0,"points":2,"correct":[0,0,1,0]},"33024":{"type":"single","id":33024,"catId":0,"points":2,"correct":[0,0,0,1]},"33027":{"type":"single","id":33027,"catId":0,"points":2,"correct":[1,0,0,0]},"33029":{"type":"single","id":33029,"catId":0,"points":2,"correct":[0,1,0,0]},"33030":{"type":"single","id":33030,"catId":0,"points":2,"correct":[1,0,0,0]},"33032":{"type":"single","id":33032,"catId":0,"points":2,"correct":[0,0,1,0]},"33034":{"type":"single","id":33034,"catId":0,"points":2,"correct":[0,0,0,1]},"33037":{"type":"single","id":33037,"catId":0,"points":2,"correct":[0,1,0,0]},"33039":{"type":"single","id":33039,"catId":0,"points":2,"correct":[0,0,1,0]},"33042":{"type":"single","id":33042,"catId":0,"points":2,"correct":[0,0,1,0]},"33044":{"type":"single","id":33044,"catId":0,"points":2,"correct":[0,1,0,0]},"33045":{"type":"single","id":33045,"catId":0,"points":2,"correct":[0,0,1,0]},"33047":{"type":"single","id":33047,"catId":0,"points":2,"correct":[0,0,1,0]},"33049":{"type":"single","id":33049,"catId":0,"points":2,"correct":[0,0,1,0]},"33051":{"type":"single","id":33051,"catId":0,"points":2,"correct":[1,0,0,0]},"33053":{"type":"single","id":33053,"catId":0,"points":2,"correct":[1,0,0,0]},"33055":{"type":"single","id":33055,"catId":0,"points":2,"correct":[0,1,0,0]},"33057":{"type":"single","id":33057,"catId":0,"points":2,"correct":[0,0,1,0]},"33059":{"type":"single","id":33059,"catId":0,"points":2,"correct":[0,0,0,1]},"33061":{"type":"single","id":33061,"catId":0,"points":2,"correct":[1,0,0,0]},"33062":{"type":"single","id":33062,"catId":0,"points":2,"correct":[0,0,1,0]},"33063":{"type":"single","id":33063,"catId":0,"points":2,"correct":[0,0,1,0]},"33064":{"type":"single","id":33064,"catId":0,"points":2,"correct":[0,1,0,0]},"33065":{"type":"single","id":33065,"catId":0,"points":2,"correct":[1,0,0,0]},"33068":{"type":"single","id":33068,"catId":0,"points":2,"correct":[0,1,0,0]},"33070":{"type":"single","id":33070,"catId":0,"points":2,"correct":[0,0,1,0]},"33073":{"type":"single","id":33073,"catId":0,"points":2,"correct":[1,0,0,0]},"33076":{"type":"single","id":33076,"catId":0,"points":2,"correct":[0,1,0,0]},"33077":{"type":"single","id":33077,"catId":0,"points":2,"correct":[0,0,1,0]},"33078":{"type":"single","id":33078,"catId":0,"points":2,"correct":[0,1,0,0]},"33079":{"type":"single","id":33079,"catId":0,"points":2,"correct":[0,0,1,0]},"33081":{"type":"single","id":33081,"catId":0,"points":2,"correct":[0,0,1,0]}} } }); All the Best IASbaba

DAILY CURRENT AFFAIRS IAS | UPSC Prelims and Mains Exam – 25th April 2024

Archives (PRELIMS & MAINS Focus)   Ethylene Oxide Syllabus Prelims – Current Event Context: Food Safety and Standards Authority of India (FSSAI) will conduct checks on products of spice brands MDH and Everest Group after authorities in Hong Kong and Singapore last week determined that four of their spice mixes contained high levels of carcinogen ethylene oxide. Background:- FSSAI doesn’t permit use of ethylene oxide in any food product. About Ethylene Oxide Ethylene oxide is a pesticide that has been classified as a Group 1 carcinogen by the International Agency for Research on Cancer, meaning there is sufficient evidence from human studies that it can cause cancer. It is used by the spice industry as a fumigant to reduce microbial contamination, such as E. coli and Salmonella. It is a colourless, highly flammable and very reactive gas that kills bacteria, viruses and fungi. It is an industrial chemical. Ethylene oxide can damage DNA during sterilisation procedures. While the risk from occasional, low-level exposure may be minimal, spices and spice blends like those flagged are commonly used in household cooking across multiple dishes. This can lead to chronic, persistent exposure over time, which has been linked to an increased risk of cancers like leukemia, stomach cancer and breast cancer. It can even lead to respiratory irritation and lung injury, headache, nausea, vomiting, diarrhoea and shortness of breath. Ethylene oxide was banned by the European Union (EU) in 2011 for fumigation of food and animal feed during transport and storage. The use of ethylene oxide is now only permitted in disinfection and sterilisation of medical devices. Additional information Escherichia coli (E.coli) is a bacterial strain that is commonly found in the intestines of people and animals, fecal waste of cattle and humans. Salmonella is a group of bacteria that can cause food-borne illnesses known as salmonellosis. The World Health Organisation (WHO) identifies Salmonella as one of four key global causes of diarrhoeal diseases.Individuals who develop salmonellosis may show symptoms such as nausea, diarrhoea, fever, and abdominal cramps 12-72 hours after contracting the infection. Source: Indian Express Ongoing Israel - Palestine Conflict Syllabus Prelims & Mains – International Event Context: Six months after the October 7 attack, Israel’s war to eliminate Hamas has resulted in the near-complete destruction of Gaza and a dire humanitarian crisis with the area on the verge of famine.Iran and its allies have militarily united behind the Palestinian cause and expanded the confrontation to other parts of the region. Background: The dynamics of war have drastically changed following the massive Iranian drone and missile attacks on military targets in Israel on April 14. This was in retaliation to Israeli aerial strikes on an Iranian diplomatic compound in Damascus on April 1 in which top Iranian generals were killed. Israel had not seen direct attacks by a state on its soil since 1991 when Saddam Hussein fired 42 Scud missiles. A volatile Middle East: The geopolitics of the region has already undergone a sea change since October 7 in four substantive ways: First, thawing of the hitherto icy Shia-Sunni relationship, with Hezbollah, Houthis and Iran (all Shia entities) extending open support to “Sunni” Palestinians, presenting regional unity via the Axis of Resistance, opposing US-Israel domination. The first signs of the Resistance surfaced early on in the war, with conflict with Hezbollah in northern Israel leading to the displacement of thousands of Israeli civilians who are yet to return. This active front is likely to simmer. Second, the war has expanded across the region, pushing the US to become the fulcrum of efforts to contain expansion and making it a primary target. Third, the Saudi-aligned Sunni Arab States have adopted the political and diplomatic route, advocating a ceasefire, supporting humanitarian assistance, and calling for a two-state solution while remaining in line with US-led initiatives. Fourth, the Yemen-based Houthis — a non-state actor devoid of a navy — have managed to successfully disrupt a critical sea lane of communication for five months, impairing trade substantively. Additional Information The expansion of the war has already caused economic distress to the Israeli economy .The Israeli economy has reportedly shrunk by 20 per cent in the last quarter of 2023. Two crucial Israeli ports, Eilat on the Red Sea and Haifa on the eastern Mediterranean have been sporadically attacked.  The Eilat port is said to be down to a quarter of its business, while Israeli media reports an acute shortage of labour in Israel, primarily on account of military conscription and volunteerism. Source: Indian Express GREEN FINANCING Syllabus Prelims & Mains – Economy & Environment Context: National Bank for Agriculture and Rural Development (NABARD) recently unveiled its Climate Strategy 2030 document which aims to address India’s need for green financing. Background: NABARD’s Climate Strategy 2030 is structured around four key pillars which include accelerating green lending across sectors, playing a broader market-making role, internal green transformation and strategic resource mobilisation. What is Green financing Green financing refers to any structured financial activity – a product or service – that has been designed to ensure a better environmental outcome. It encompasses a variety of loans, debt mechanisms, and investments aimed at promoting the development of green projects or minimizing the climate impact of regular projects. Purpose and Importance: Green finance aims to align financial activities with environmental goals. It plays a crucial role in achieving several of the United Nations’ Sustainable Development Goals by supporting sustainable development priorities. Examples of Green Finance Projects: Renewable Energy and Energy Efficiency: Funding projects related to clean energy sources and energy-saving technologies. Pollution Prevention and Control: Financing initiatives that reduce pollution and promote cleaner practices. Biodiversity Conservation: Supporting efforts to protect and preserve biodiversity. Circular Economy Initiatives: Investing in projects that promote resource efficiency and waste reduction. Sustainable Use of Natural Resources and Land: Financing projects that balance economic development with environmental conservation. Green Bonds: Green bonds are a common green finance instrument. They adhere to specific criteria, including the use of proceeds, project evaluation, proper management, and detailed reporting. The United States, China, and France are the three largest issuers of green bonds. Source: Hindu ENGLISH CHANNEL Syllabus Prelims – Geography Context: Recently, five asylum seekers died while crossing the English Channel from France to Britain in an overcrowded small boat. Background: In recent years, the English Channel has become a focal point for migrants and asylum seekers attempting to reach the UK. Many undertake dangerous journeys in small boats, hoping to find safety and better opportunities.   About English Channel: The English Channel, also known as “La Manche,” is an arm of the Atlantic Ocean that separates Southern England from northern France. It links to the southern part of the North Sea via the Strait of Dover at its northeastern end. Notably, it is the busiest shipping area in the world. Historically, the channel played a pivotal role in Britain’s naval supremacy and served as a natural defence mechanism against invasions during significant events like the Napoleonic Wars and World War II. Source: Reuters VOYAGER 1 SPACECRAFT Syllabus Prelims – Science Context: NASA Voyager 1 spacecraft has begun sending readable communications again after months of transmitting gibberish back to Earth. Background: NASA’s Voyager flight team detected a glitch in November and worked to fix the problem via code, leading to the successful return of readable data. About VOYAGER 1 SPACECRAFT : Voyager 1 was launched by NASA on September 5, 1977. Voyager 1 is part of the Voyager program, which also includes Voyager 2. Mission Objective: To study the outer Solar System and beyond, including flybys of Jupiter and Saturn. Discoveries: It discovered a thin ring around Jupiter and two new Jovian moons: Thebe and Metis. At Saturn, it found five new moons and a new ring called the G-ring. Interstellar Achievement: Voyager 1 was the first human-made object to cross the heliosphere and enter interstellar space in August 2012. Current Status: It is the most distant human-made object from Earth and is expected to send data until at least 2025. Golden Record: Carries a golden record with sounds and images representing life and culture on Earth. Source: Live Science Climate Change and Human Rights Syllabus Prelims – GS 2 & GS 3 Context: The Supreme Court of India acknowledged the right to protection from climate change impacts as part of the fundamental rights to life (Article 21) and equality (Article 19) enshrined in the Indian Constitution. Background: The arguments were a part of a verdict by a three-judge bench headed by the Chief Justice of India DY Chandrachud hearing a case on the conservation of the great India bustard and the lesser florican. The intersection of climate change and human rights: Climate change can directly affect people’s right to life by causing extreme weather events like hurricanes or floods, which can lead to loss of life and property. For example, in low-lying coastal areas, rising sea levels due to climate change can threaten people’s homes and livelihoods, forcing them to relocate. Climate change can impact water sources, leading to water scarcity or contamination. This affects people’s right to clean water and sanitation. In regions where droughts are becoming more frequent due to climate change, communities may struggle to access safe drinking water, leading to health issues. Climate change can exacerbate health problems, especially for vulnerable populations. For instance, increased heat waves can lead to heat-related illnesses and deaths, affecting the right to health. Climate change-induced events such as sea-level rise, extreme weather events, or desertification can force people to migrate or be displaced from their homes. This intersects with human rights, particularly the right to residence and the right to seek asylum. Climate change can disproportionately affect indigenous communities that rely heavily on natural resources for their livelihoods and cultural practices. For instance, changes in ecosystems due to climate change can threaten traditional livelihoods like farming or fishing, impacting indigenous peoples’ rights to land, resources, and cultural heritage. The Supreme Court’s interpretation of constitutional provisions concerning climate change: Article 48A which mandates environmental protection and Article 51A(g) which promotes wildlife conservation, implicitly guarantee a right to be safeguarded from climate change. Article 21 recognises the right to life and personal liberty while Article 14 indicates that all persons shall have equality before the law and the equal protection of laws. These articles are important sources of the right to a clean environment and the right against the adverse effects of climate change. In MC Mehta vs Kamal Nath Case, 2000, the Supreme Court stated that the right to a clean environment is an extension of the right to life. Issues/Challenges in balancing climate change mitigation with human rights protection: Some climate mitigation measures may conflict with human rights, such as restrictions on land use for conservation projects or displacement due to renewable energy infrastructure development. Finding solutions that minimise negative impacts while maximising benefits is challenging. Climate actions like transitioning to renewable energy or implementing carbon pricing can impact access to essential resources like energy, water, and food, especially for marginalised communities. Climate-induced migration can strain social systems and lead to conflicts over resources and rights in host communities. Managing migration flows in a way that respects the rights of both migrants and host populations is a multifaceted challenge. Balancing efforts to reduce greenhouse gas emissions (mitigation) with investments in adaptation to climate impacts can be challenging. Prioritising one over the other can have implications for human rights, particularly for communities already facing climate-related risks. Climate change is a global issue requiring international cooperation. Balancing national climate goals with global responsibilities and ensuring that climate actions do not undermine the rights of vulnerable communities across borders is a complex task. Source: Down To Earth Practice MCQs Daily Practice MCQs Q1.) With reference to the Voyager 1 spacecraft, consider the following statements: Voyager 1 was launched by ISRO. The primary mission of Voyager 1 was to explore the outer planets of solar system. Which of the statements given above is/are correct? 1 only 2 only Both 1 and 2 Neither 1 nor 2 Q2.) English Channel separates England from Spain France Germany Poland Q3.) With reference to the Green Financing, consider the following statements: Green financing aims to align financial activities with environmental goals. It encompasses a variety of debt mechanisms and investments aimed at promoting the development of green projects. Green bonds are a common green finance instrument. Which of the statements given above are correct? Only one Only two All three None Comment the answers to the above questions in the comment section below!! ANSWERS FOR ’  25th April  2024 – Daily Practice MCQs’ will be updated along with tomorrow’s Daily Current Affairs.st ANSWERS FOR  24th April – Daily Practice MCQs Answers- Daily Practice MCQs Q.1) – b Q.2) – c Q.3) – d